Hematology

144
CHAPTER Hematology Outline 230 >• Fundamental Hematology Principles >• Hematopoiesis >• Granulocytes >• Monocytes and Macrophages >• Lymphocytes and Plasma Cells >• Malignant Leukocyte Disorders > Erythrocytes >• Hemoglobin >• Anemias >• Hemoglobinopathies >• Thalassemias >• Hematology Tests Review Questions 288 Answers & Rationales 323 References 372

description

Hematology

Transcript of Hematology

Page 1: Hematology

CHAPTER

Hematology

Outline 230

>• Fundamental Hematology Principles

>• Hematopoiesis

>• Granulocytes

>• Monocytes and Macrophages

>• Lymphocytes and Plasma Cells

>• Malignant Leukocyte Disorders

> Erythrocytes

>• Hemoglobin

>• Anemias

>• Hemoglobinopathies

>• Thalassemias

>• Hematology Tests

Review Questions 288

Answers & Rationales 323

References 372

Page 2: Hematology

230 • CHAPTER 2: HEMATOLOGY

I. FUNDAMENTAL HEMATOLOGY PRINCIPLES

A. Blood Composition1 . Whole blood includes erythrocytes, leukocytes, platelets, and plasma. When

a specimen is centrifuged, leukocytes and platelets make up the buffy coat (smallwhite layer of cells lying between the packed red blood cells and the plasma).

2. Plasma is the liquid portion of unclotted blood. Serum is the fluid thatremains after coagulation has occurred and a clot has formed.a. Plasma is composed of 90% water and contains proteins, enzymes,

hormones, lipids, and salts.b. Plasma normally appears hazy and pale yellow (contains all coagulation

proteins), and serum normally appears clear and straw colored (lacksfibrinogen group coagulation proteins).

B. Basic Hematology Terminology

a-

-blast

-chromic

-cyte

dys-

-emia

ferro-

hyper-

hypo-

iso-

macro-

mega-

micro-

myelo-

normo-

-oid

-osis

pan-

-penia

-plasia

-poiesis

poly-

pro-

thrombo-

without

youngest/nucleated

colored

cell

abnormal

in the blood

iron

increased

decreased

equal

large

very large/huge

small

marrow

normal

like

increased

all

decreased

formation

cell production

many

before

clot

Page 3: Hematology

FUNDAMENTAL HEMATOLOGY PRINCIPLES • 231

C. Formed Elements and Sizes

Formed Element

1. Thrombocytes (platelets)

2. Erythrocytes (RBCs)

3. Normal lymphocytes

4. Reactive lymphocytes

5. Basophils

6. Segmented neutrophils

7. Band neutrophils

8. Eosinophils

9. Monocytes

Size

2-4 |j,m

6-8 (jLm

6-9 (Jim

10-22 (jum

10-1 5 (xm

10-15 (Jim

10-15 |jim

12-16 |j,m

12-20 (Jim

D. Basic HomeostasisI . Homeostasis is the body's tendency to move toward physiological stability.

In vitro testing of blood and other body fluids must replicate exactenvironmental body conditions. These conditions should include the following:a. Osmotic concentration is the body/cellular water concentration,

composed of 0.85% sodium chloride. This normal osmotic concentrationis termed isotonic. In a hypotonic solution (greater amount of H2O inrelationship to lesser amount of solutes), water enters the cell; the cellswells and may lyse. In a hypertonic solution (lesser amount of H2O inrelationship to greater amount of solutes), water leaves the cell; the cellmay crenate.

b. pH reference range: Venous blood range 7.36-7.41; arterial blood range7.38-7.44

c. Temperature: Normal body temperature is 37.0°C. Blood specimensshould be analyzed as soon as possible to prevent cellular breakdown(refer to individual tests for specimen collection requirements, stabilitytimes, and storage temperature).

E. RBC Indices

1 . MCV (mean corpuscular volume): Reference range (Si/conventional units)is 80-100 femtoliters (fL), and it is an indicator of the average/mean

Page 4: Hematology

• CHAPTER 2: HEMATOLOGY

volume of erythrocytes (RBCs). Calculate using the hematocrit (Hct) andRBC count:

Hct (%) X 10MCV (fL) =

RBC count (X 1012/L)

a. Increased in megaloblastic anemia, hemolytic anemia withreticulocytosis, liver disease, and normal newborn

b. Decreased in iron deficiency anemia, thalassemia, sideroblastic anemia,and lead poisoning

2. MCH (mean corpuscular hemoglobin): Reference range (Si/conventionalunits) is 26-34 picograms (pg), and it is an indicator of the average weight ofhemoglobin in individual RBCs. Calculate using the hemoglobin (Hgb) andRBC count:

Hemoglobin (g/dL) X 10MCH (pg) = - - - ̂ - Wi -

F5 RBC count (X 1012/L)

a. Increased in macrocytic anemiab. Decreased in microcytic, hypochromic anemia

3. MCHC (mean corpuscular hemoglobin concentration): Reference range(conventional units) is 32-37 g/dL (SI units 320-370 g/L), and it is a measureof the average concentration of hemoglobin in grams per deciliter. Calculateusing the hemoglobin and hematocrit values:

Hemoglobin (g/dL)MCHC (g/dL) - - - - - - - X 100

Hct

a. 32-37 g/dL MCHC indicates normochromic RBCs.b. Lesser than (<) 32 g/dL MCHC indicates hypochromic RBCs, which is

seen in iron deficiency and thalassemia.c. Greater than (>) 37 g/dL MCHC indicates a possible error in RBC or

hemoglobin measurement, or the presence of spherocytes.

F. Other RBC Parameters1 . RDW (RBC distribution width): Reference range (conventional units) is

11.5-14.5%.a. Determined from the RBC histogramb. Increased proportional to the degree of anisocytosis (variation in size);

coefficient of variation of the mean corpuscular volumec. High RDW: Seen post-transfusion, post-treatment (e.g., iron, B12, or folic

acid therapy), idiopathic sideroblastic anemia, in the presence of twoconcurrent deficiencies (iron and folic acid deficiencies)

Page 5: Hematology

FUNDAMENTAL HEMATOLOGY PRINCIPLES • 233

2. Hct (Hematocrit): Reference range for males (conventional units) is 41-53%(SI units 0.41-0.53 L/L). Reference range for females (conventional units) is36-46% (SI units 0.36-0.46 L/L). Reference range for hematocrit is age andsex dependent. Hematocrit is the percentage of RBCs in a given volume ofwhole blood.a. Spun microhematocrit is the reference manual method.b. The buffy coat layer of leukocytes and platelets, not included in the

measurement, can be seen between plasma (upper) and RBC (lower)layers.

c. Hematocrit is calculated by many automated cell counters using the MCVand RBC count:

MCV (fL) X RBC count (X 1012/L)Hc,% =

3. Hgb (Hemoglobin): Reference range for males (conventional units) is13.5-17.5 g/dL (SI units 135-175 g/L). Reference range for females(conventional units) is 12.0-16.0 g/dL (SI units 120-160 g/L). Referencerange for hemoglobin is age and sex dependent.

G. Platelets

1. PLT (Platelets): Reference range (SI units) is 150-450 X 109/L (conventionalunits 150,000-450,000/^L).

2. MPV (mean platelet volume): Reference range (Si/conventional units) is6.8-10.2 fL. MPV is analogous to the MCV for erythrocytes.

H. Relative and Absolute Blood Cell Counts

1. Relative count is the amount of a cell type in relation to other bloodcomponents. Relative lymphocytosis is an increase in the percentage oflymphocytes; this is frequently associated with neutropenia. In relativepolycythemia, RBCs appear increased due to a decreased plasma volume.

2. Absolute count is the actual number of each cell type without respect to otherblood components. Absolute lymphocytosis is a true increase in the numberof lymphocytes. Absolute polycythemia is a true increase in red cell mass.

I. Hematology Stains

1 . Nonvital (dead cell) polychrome stain (Romanowsky)a. Most commonly used routine peripheral blood smear stain is Wright's

stain.b. Wright's stain contains methylene blue, a basic dye, which stains acidic

cellular components (DNA and RNA) blue, and eosin, an acidic dye,which stains basic components (hemoglobin and eosinophilic cytoplasmicgranules) red-orange.

Page 6: Hematology

234 • CHAPTER 2: HEMATOLOGY

c. Methanol fixative is used in the staining process to fix the cells to the slide.d. Staining does not begin until a phosphate buffer (pH between 6.4 and

6.8) is added.e. Causes of RBCs too red and WBC nuclei poorly stained: Buffer or stain

below pH 6.4, excess buffer, decreased staining time, increased washingtime, thin smear, expired stains

f. Causes of RBCs and WBC nuclei too blue: Buffer or stain above pH 6.8,too little buffer, increased staining time, poor washing, thick smear,increased protein, heparinized blood sample

g. Examples of polychrome stains include: Wright, Giemsa, Leishman,Jenner, May-Grimwald, and various combinations of them

2. Nonvital monochrome staina. Stains specific cellular componentsb. Prussian blue stain is an example.

1) Contains potassium ferrocyanide, HC1, and a safranin counterstain2) Used to visualize iron granules in RBCs (siderotic iron granules),

histiocytes, and urine epithelial cells3. Supravital (living cell) monochrome stain

a. Used to stain specific cellular componentsb. No fixatives are used in the staining process.c. Includes:

1) New methylene blue used to precipitate RNA in reticulocytes;measure of bone marrow erythropoiesis

2) Neutral red with brilliant cresyl green as a counterstain is used tovisualize Heinz bodies; clinical disorders associated with Heinz bodiesinclude G6PD deficiency and other unstable hemoglobin disorders.

II. HEMATOPOIESIS

A. Hematopoiesis

1. Production and differentiation of blood cells2. Blood cell production, maturation, and death occur in organs of the

reticuloendo-thelial system (RES).a. RES includes bone marrow, spleen, liver, thymus, lymph nodes.b. RES functions in hematopoiesis, phagocytosis, and immune defense.

3. Intrauterine hematopoiesis includes three phases:a. Mesoblastic (yolk sac) phase begins at -19 days gestation. The yolk sac

is located outside the developing embryo. The first cell to be produced is aprimitive nucleated erythroblast. This cell produces embryonichemoglobins: Portland, Gower I and Gower II. Alpha-globin chainproduction begins at this phase and continues throughout life.

b. Hepatic (liver) phase begins at 6 weeks gestation with production ofmainly red blood cells, but also granulocytes, monocytes, and

Page 7: Hematology

HEMATOPOIESIS • 235

megakaryocytes. Alpha- and gamma-globin chain productionpredominates forming Hgb F; detectable Hgb A and A2 are also present.

c. Myeloid/medullary phase begins around the fifth month of gestation,with the bone marrow producing mainly granulocytes. The M:E(myeloid:erythroid) ratio approaches the adult level of 3:1. Alpha- andgamma-globin chain production predominates at birth, forming Hgb F;Hgb A and A.2 are also present. Hgb A will not predominate until6 months of age when the gamma-beta globin chain switch is complete.

4. At birth, the bone marrow is very cellular with mainly red marrow, indicatingvery active blood cell production. Red marrow is gradually replaced byinactive yellow marrow composed of fat. Under physiological stress, yellowmarrow may revert to active red marrow.

B. Pediatric and Adult Hematopoiesis1 . Bone marrow

a. Newborn: 80-90% of bone marrow is active red marrow.b. Young adult (age 20): 60% of bone marrow is active. Hematopoiesis

is confined to the proximal ends of large flat bones, pelvis,and sternum.

c. Older adult (age 55): 40% of bone marrow is active; 60% is fat.d. Cellularity is the ratio of marrow cells to fat (red marrow/yellow marrow)

and is described in adults as:1) Normocellular— Marrow has 30-70% hematopoietic cells.2) Hypercellular/hyperplastic—Marrow has >70% hematopoietic cells.3) Hypocellular/hypoplastic—Marrow has <30% hematopoietic cells.4) Aplastic—Marrow has few or no hematopoietic cells.

e. M:E (myeloid:erythroid) ratio is the ratio of granulocytes and theirprecursors to nucleated erythroid precursors. A normal ratio is between3:1 and 4:1. Granulocytes are more numerous because of their shortsurvival (1-2 days) as compared to erythrocytes with a 120-day life span.Lymphocytes and monocytes are excluded from the M:E ratio.

f. Stem cell theory1) Hematopoiesis involves the production of pluripotential stem cells

that develop into committed progenitor cells (lymphoid or myeloid)and finally mature blood cells,a) Progenitor cells

i) Lymphoid: Differentiate into either B or T lymphocytes inresponse to cytokines/lymphokines/interleukins/CSFs/growthfactors

ii) Myeloid: Gives rise to the multipotential progenitor CFU-GEMM (colony-forming-unit-granulocyte-erythrocyte-macrophage-megakaryocyte), which will differentiate intocommitted progenitor cells and finally mature blood cells in

Page 8: Hematology

CHAPTER 2: HEMATOLOGY

response to cytokines/interleukins/colony stimulatingfactors/growth factors:

Committed Pr

CFU-MEG

CFU-GM

CFU-GM

BFU-E

CFU-Eo

CFU-Ba

ogenitor Cell

CFU-M

CFU-G

CFU-E

Growth Factors/ Interleukins

Thrombopoietin, GM-CSF

GM-CSF, M-CSF, IL-3

GM-CSF, G-CSF, IL-3

Erythropoietin, GM-CSF, IL-3

GM-CSF, IL-3, IL-5

IL-3, IL-4

Mature Cell

Thrombocytes

Monocytes

Neutrophils

Erythrocytes

Eosinophils

Basophils

2. Lymphoid tissuea. Primary lymphoid tissue

1) Bone marrow: Site of pre-B cell differentiation2) Thymus: Site of pre-T cell differentiation3) This is antigen-independent lymphopoiesis.

b. Secondary lymphoid tissue1) B and T lymphocytes enter the blood and populate secondary lymphoid

tissue, where antigen contact occurs.2) Includes lymph nodes, spleen, gut-associated tissue (Peyer's patches)3) Antigen-dependent lymphopoiesis depends on antigenic stimulation of

T and B lymphocytes.

C. Introduction to Leukocytes

1. Classified as phagocytes (granulocytes, monocytes) or immunocytes(lymphocytes, plasma cells, and monocytes)

2. WBC reference range (SI units) is 4.0-11.0 X 109/L (conventional units4.0-11.0Xl03/|xL).

3. Granulocytes include neutrophils, eosinophils, and basophils.4. Neutrophils are the first to reach the tissues and phagocytize (destroy)

bacteria. In the process, they die.5. Monocytes differentiate into macrophages, and as such they work in the

tissues to phagocytize foreign bodies. They arrive at the site of inflammationafter neutrophils and do not die in the process.

6. T lymphocytes provide cellular immunity. They represent 80% oflymphocytes in the blood. When activated, they proliferate and producecytokines/interleukins.

Page 9: Hematology

HEMATOPOIESIS • 237

7. B lymphocytes develop into plasma cells in the tissue and produceantibodies needed for humoral immunity. B lymphocytes represent 20% ofthe lymphocytes in the blood.

8. NK (natural killer) lymphocytes destroy tumor cells and cells infected withviruses. They are also known as large granular lymphocytes (LGLs).

9. Eosinophils modulate the allergic response caused by basophil degranulation.10. Basophils mediate immediate hypersensitivity reactions (type I,

anaphylactic).11. CD markers are surface proteins expressed by specific cell lines at different

maturation stages. As a cell matures, some markers vanish and new onesappear. More than 200 CD markers have been identified. Commonly usedmarkers include the following:

CD2,CD3

CD4

CDS

CD13

CD11c,CD14

CD19, CD20

CD33

CD34

GDI 6, CD56

Lymphoid, pan T cells

Helper/inducerTcells

Suppressor/cytotoxic T cells

Pan myeloid

Monocytes

Lymphoid, pan B cells

Pan myeloid cells

Stem cell marker (lymphoid and myeloid precursor)

NKcells

D. Medullary versus Extramedullary Hematopoiesis1. Medullary hematopoiesis: Blood cell production within the bone marrow

a. Begins in the fifth month of gestation and continues throughout life2. Extramedullary hematopoiesis: Blood cell production outside the bone

marrowa. Occurs when the bone marrow cannot meet body requirementsb. Occurs mainly in the liver and spleen; hepatomegaly and/or splenomegaly

often accompany thisE. Basic Cell Morphology

1 . Nucleusa. Contains chromatin composed of DNA and proteinsb. Contains nucleoli rich in RNA

Page 10: Hematology

CHAPTER 2: HEMATOLOGY

2. Cytoplasma. Golgi complex forms lysosomes.b. Lysosomes contain hydrolytic enzymes that participate in phagocytosis.c. Ribosomes assemble amino acids into protein.d. Mitochondria furnish the cell with energy (ATP).e. Endoplasmic reticulum is a system of interconnected tubes for protein

and lipid transport.

F. General Cell Maturation Characteristics for Leukocytes

mmature Cells Mature Cells

Cell is large Cell becomes smaller

Nucleoli present Nucleoli absent

Chromatin fine and delicate Chromatin coarse and clumped

Nucleus round Nucleus round, lobulated, or segmented

Cytoplasm dark blue (rich in RNA) Cytoplasm light blue (less RNA)

High N:C ratio Low N:C ratio

III. GRANULOCYTES

A. Basic Review1. The myeloid progenitor cell gives rise to a committed progenitor cell that is

acted on by growth factors to form granulocytes.

B. Maturation and Morphology of Immature Granulocytes1 . Myeloblast: Earliest recognizable granulocyte precursor

a. 14-20 |xmb. N:C ratio 7:1-4:1c. Round/oval nucleus with fine reddish-purple staining chromatind. 2-5 nucleolie. Dark blue cytoplasmf. No cytoplasmic granulesg. 1 % of the nucleated cells in the bone marrow

2. Promyelocytea. 15-21 (Jimb. N:C ratio 3:1c. Round/oval nucleus with slightly coarsening chromatin

Page 11: Hematology

GRANULOCYTES • 239

d. 1-3 nucleolie. Dark blue cytoplasmf. Cytoplasm has large, nonspecific/primary granules containing

myeloperoxidase.g. 2-5% of the nucleated cells in the bone marrow

3. Myelocyte: First stage where granulocyte types can be differentiated intoeosinophils, basophils, and neutrophilsa. 12-18 (Jimb. N:C ratio 2:1c. Round nucleus with coarse chromatind. Early myelocytes may have visible nucleoli.e. Light blue to light pink cytoplasmf. Prominent golgi apparatus—clear area located in the cytoplasm next to

the nucleusg. Cytoplasm has specific/secondary granules that contain hydrolytic

enzymes, including alkaline phosphatase and lysozyme.h. Nonspecific/primary granules are present and may still stain,i. Last stage capable of cell divisionj. Neutrophilic myelocyte makes up 13% of the nucleated cells in the bone

marrow.4. Metamyelocyte

a. 10-18 (Jtmb. N:C ratio 1.5:1c. Nucleus is indented in a kidney bean shape and has coarse, clumped

chromatin.d. Nuclear indent is less than half the width of a hypothetical round nucleus.e. Cytoplasm is pink and filled with pale blue to pink specific/secondary

granules.f. Nonspecific/primary granules are present but usually do not stain.g. Neutrophilic metamyelocyte makes up 16% of the nucleated cells in the

bone marrow.5. Band neutrophil

a. 10-15 |xmb. N:C ratio 1:2c. Nucleus is "C" or "S"-shaped with coarse, clumped chromatin lacking

segmentation.d. Nuclear indent is greater than half the width of a hypothetical round

nucleus.e. Cytoplasm is pink and filled with pale blue to pink specific/secondary

granules.f. Nonspecific/primary granules are present but usually don't stain.g. Band neutrophil makes up 12% of the nucleated cells in the bone

marrow, and 0-5% of peripheral white blood cells (WBCs).

Page 12: Hematology

240 • CHAPTER 2: HEMATOLOGY

h. Stored in the bone marrow and released when there is an increaseddemand for neutrophils

C. Morphology of Mature Granulocytes

1 . Segmented neutrophil (Referred to as: seg, polymorphonuclear cell (PMN),and poly)a. 10-15 (Jimb. N:C ratio 1:3c. Nucleus has coarse, clumped chromatin with 3-5 lobes connected by thin

filaments.d. Cytoplasm is pink and filled with small, pale blue to pink specific/

secondary granules.e. Nonspecific/primary granules are present but usually do not stain unless

in response to infection or growth factor.f. Segmented neutrophil makes up 12% of the nucleated cells in the bone

marrow, and 50-80% of peripheral WBCs.2. Eosinophil

a. Recognizable maturation stages include the eosinophilic myelocyte,eosinophilic metamyelocyte, eosinophilic band, and eosinophil(segmented form).

b. Eosinophils are 12-16 |xm.c. Nucleus is usually bilobed.d. Cytoplasm contains large, bright red-orange, secondary granules that

contain enzymes and proteins.e. Eosinophils make up less than 1% of the nucleated cells in the bone

marrow and 5% of peripheral WBCs3. Basophil

a. Recognizable maturation stages include the basophilic myelocyte,basophilic metamyelocyte, basophilic band, and basophil (segmentedform).

b. Basophils are 10-15 (Jim.c. Cytoplasm contains large, purple-black, secondary granules that contain

heparin and histamine.d. Granules may be numerous and obscure the nucleus, or they may "wash

out" in staining (because the granules are water soluble) and leave emptyareas in the cytoplasm.

e. Basophils make up less than 0.1 % of the nucleated cells in both the bonemarrow and peripheral blood

D. Granulocyte Function

1. Neutrophilsa. Blasts, promyelocytes, and myelocytes are in the bone marrow mitotic

pool 3-6 days, and that is where they divide.

Page 13: Hematology

GRANULOCYTES • 241

b. Metamyelocytes, bands, and segmented neutrophils are in the bonemaiTow post-mitotic pool about 6 days, and that is where they mature.

c. Released into circulation when mature or when neededd. Total blood granulocyte pool

1) Contains 50% circulating granulocyte pool (mainly neutrophils) thatis measured when a WBC count is performed

2) Contains 50% marginating granulocyte pool (mainly neutrophils)that adheres to vessel walls

3) There is a rapid and free exchange of neutrophils between thecirculating granulocyte pool and marginating granulocyte pool.

4) Neutrophils diapedese into the tissues from the marginating pool inresponse to antigenic stimulation.

5) Chemotactic factors attract the neutrophil to the site of inflammation;include complement, bacterial products, injured tissue, hemostaticcomponents.

6) Opsonins such as IgG and complement component C3b helpneutrophils recognize a substance as foreign.

7) Phagocytosis involves neutrophil attachment to the foreign object,formation of a vacuole around it, and neutrophilic degranulation torelease lytic enzymes (respiratory burst) in an effort to kill theorganism.

8) Neutrophils are sensitive to the oxidants they secrete and are destroyedin the process.

e. Blood and tissue cells in the body undergo cell death through necrosis orapoptosis.1) Necrosis is induced by extracellular forces such as lethal chemical,

biological, or physical events. The blood cell is "killed."2) Apoptosis is "programmed cell death" due to extracellular or

intracellular processes that depend on a signal.2. Visible response to infection by neutrophils (toxic changes)

a. Toxic changes are associated with bacterial infection or growth factortherapy. Any combination of these changes may be seen in some but notnecessarily all of the neutrophils.

b. Toxic granulation is prominent granulation due to persistent staining ofprimary granules. Neutrophilic cytoplasm normally contains onlyvisible, small, secondary granules.

c. Toxic vacuolation: Colorless areas in the cytoplasm that indicatephagocytosis and degranulation have occurred

d. Dohle bodies: Small oval inclusions (RNA) located in the cytoplasm stainlight blue

e. Shift to the left refers to an increased number of myelocytes,metamyelocytes, and/or bands in the peripheral blood. It is associatedwith either increased or decreased WBC counts.

Page 14: Hematology

242 • CHAPTER 2: HEMATOLOGY

1) Regenerative shift to the left is an appropriate bone marrow responseto increased demand for neutrophils. It is seen in infection or in otherphysiological or pathological conditions requiring neutrophils.a) WBC count above the reference rangeb) Most common type of left shift

2) Degenerative shift to the left is seen after an overwhelming infectionin which bone marrow production cannot keep up with increased needfor neutrophils.a) Associated with a poor prognosisb) WBC count below the reference range

3. Eosinophilsa. In the blood only a few hours before seeking a tissue site such as nasal

passages, skin, or urinary tractb. They can degranulate like neutrophils. They express Fc receptors for IgE,

which is a response to parasitic infections.c. They release substances that can neutralize products released by basophils

and mast cells; eosinophils modulate the allergic response.4. Basophils

a. In the blood only a few hours before migrating to the site of inflammationin the tissues

b. They express membrane receptors for IgE. Once activated, degranulationreleases histamine. This initiates the classic signs of immediatehypersensitivity reactions (Type I).

c. Basophils release a chemotactic factor that attracts eosinophils to the site.

E. Nonmalignant Granulocytic Disorders

1 . Shift/physiologic/pseudoneutrophiliaa. Redistribution of the blood pools causes a short-term increase in the total

WBC count and in the absolute number of neutrophils in the circulatinggranulocyte pool.

b. Caused by exercise, stress, pain, pregnancyc. It is not a response to tissue damage. The total blood granulocyte pool in

the body has not changed. The bone marrow has not released immatureneutrophils. There are no toxic changes, and there is no shift to the left.

2. Pathologic neutrophiliaa. Neutrophils leave the circulating pool, enter the marginating pool, and

then move to the tissues in response to tissue damage.b. Bone marrow reserves are released into the blood to replenish the

circulating pool. The WBC count can increase up to 50.0 X 109/L, andthere is a shift to the left with toxic changes to the neutrophils.

c. Bone marrow increases production of neutrophils to replenish reserves.d. Occurs in response to bacterial and other infections, tissue destruction,

drugs or toxins, growth factor, etc.

Page 15: Hematology

GRANULOCYTES • 243

3. Neutrophilic leukemoid reaction (NLR)a. Blood picture mimics that seen in chronic myelogenous leukemia.b. Benign, extreme response to a specific agent or stimulusc. The WBC count can increase to between 50.0 and 100.0 X 109/L, and

there is a shift to the left with toxic changes to the neutrophils.4. Leukoerythroblastic reaction

a. Presence of immature leukocytes and immature (nucleated)erythrocytes in the blood

b. Occurs in marrow replacement disorders, such as myelofibrosis5. Neutropenia

a. Decrease in absolute number of neutrophils; risk of infection increasesas neutropenia worsens

b. Due to bone marrow production defects:1) Chronic or severe infection depletes available neutrophil reserves. Use

exceeds bone marrow production.2) Hypersplenism causes neutrophils to be removed from circulation.3) Bone marrow injury (aplastic anemia), bone marrow infiltration

(leukemia, myelodysplastic syndromes, or metastatic cancer), bonemaiTow suppression by chemicals or drugs (chemotherapy)

4) DNA synthesis defects due to vitamin B12 or folate deficiency5) Many viral infections are associated with neutropenia.

6. Eosinophiliaa. Increase in the absolute number of eosinophilsb. Associated with:

1) Parasitic infections, allergic reactions, chronic inflammation2) Chronic myelogenous leukemia, including early maturation stages,

Hodgkin disease, tumors7. Eosinopenia

a. Decrease in the absolute number of eosinophilsb. Seen in acute inflammation and inflammatory reactions that cause release

of glucocorticosteroids and epinephrine8. Basophilia

a. Increase in the absolute number of basophilsb. Associated with:

1) Type I hypersensitivity reactions2) Chronic myelogenous leukemia, including early maturation stages,

polycythemia vera3) Relative transient basophilia can be seen in patients on hematopoietic

growth factors.9. Basopenia

a. Decrease in the absolute number of basophils associated withinflammatory states and following immunologic reactions

b. Difficult to diagnose because of their normally low reference range

Page 16: Hematology

244 • CHAPTER 2: HEMATOLOGY

10. Functional disorders of neutrophilsa. Chronic granulomatous disease (CGD)

1) Both sex-linked and autosomal recessive inheritance with the ratio ofaffected males to females being 6:1

2) Morphologically normal, but functionally abnormal because of enzymedeficiency that results in an inability to degranulate, which causesinhibited bactericidal function

3) Fatal early in lifeb. Chediak-Higashi syndrome

1) Autosomal recessive disorder causes large, gray-green, peroxidasepositive granules in the cytoplasm of leukocytes; abnormal fusion ofprimary and secondary neutrophilic granules

2) Both morphologically and functionally abnormal leukocytes; WBCsunable to degranulate and kill invading bacteria

3) Patients will present with photophobia and skin hypopigmentation.4) Fatal early in life

11. Nuclear abnormalities of neutrophilsa. Hypersegmentation characterized by 5 or more lobes in the neutrophil;

associated with megaloblastic anemia due to vitamin B12 or folic aciddeficiencies

b. Hyposegmentation refers to a tendency in neutrophils to have 1 or 2lobes; may indicate an anomaly or a shift to the left.1) Pelger-Huet anomaly

a) Autosomal dominant inheritanceb) Nucleus is hyperclumped, and it does not mature past the two-lobed

stage.c) Nucleus dumbbell- or peanut shaped; referred to as "pince-nez"d) Morphologically abnormal, but functionally normale) Must differentiate from a shift to the left associated with an

infection (toxic changes); infection requires treatment but Pelger-Huet anomaly (no toxic changes) does not.

2) Pseudo Pelger-Hueta) Acquired abnormality associated with myeloproliferative

disorders and myelodysplastic syndromes; can also be druginduced

b) Nucleus is usually round instead of the dumbbell shape that is seenin the anomaly.

c) Frequently accompanied by hypogranulation12. Inherited cytoplasmic anomalies

a. May-Hegglin anomaly1) Autosomal dominant inheritance2) Large, crystalline, Dohle-like inclusions in the cytoplasm of

neutrophils on Wright's stain; gray-blue and spindle (cigar) shaped

Page 17: Hematology

MONOCYTESANDMACROPHAGES • 245

3) Morphologically abnormal, but functionally normal4) Giant platelets, thrombocytopenia, and clinical bleeding are also

associated with this anomaly.b. Alder-Reilly anomaly

1) Autosomal recessive inheritance2) Large azurophilic granules appear in cytoplasm of all or only one cell

line. Granules contain degraded mucopolysaccharides due to anenzyme defect.

3) Morphologically abnormal, but functionally normal4) Must differentiate from toxic granulation present in neutrophils only

in infectious conditions

IV. MONOCYTES AND MACROPHAGES

A. Basic Review

1 . The myeloid progenitor cell gives rise to a committed progenitor cell,CFU-GM (colony-forming-unit-granulocyte-macrophage), that is actedon by growth factors (GM-CSF) and interleukins (ILs) to form monocytes.Monocytes form in the bone marrow, pass through the peripheral blood, andthen migrate into the tissues (macro-phages), where they fight infection.Macrophages are named according to their location in the body.a. Monocytes —peripheral bloodb. Kupffer cells—liverc. Microglial cells—central nervous systemd. Osteoclasts—bonee. Langerhans' cells—skinf. Alveolar cells—lung

B. Maturation and Morphology of Monocytes

1. Monoblast: Earliest recognizable monocyte precursora. 12-18 (Jim; N:C ratio 4:1b. Round/oval eccentric nucleus with fine chromatin; 1-2 nucleolic. Dark blue cytoplasm; may have a gray tint; no cytoplasmic granules

2. Promonocytea. 12-20 |JLm;N:C ratio 3:1b. Irregularly shaped, indented nucleus with fine chromatin; 0-1 nucleolic. Blue to gray cytoplasm; fine azurophilic granules

3. Monocytea. 12-20 (Jimb. Horseshoe- or kidney-bean-shaped nucleus, often with "brainlike"

convolutionsc. Fine, lacy chromatind. Blue-gray cytoplasm; may have pseudopods and vacuoles

Page 18: Hematology

246 • CHAPTER 2: HEMATOLOGY

e. Many fine azurophilic granules give the appearance of "ground glass."f. Transitional cell because it migrates into the tissue and becomes a fixed or

free macrophage4. Macrophage: "Tissue monocyte"

a. 15-80 |xmb. Indented, elongated, or egg-shaped nucleus with fine chromatinc. Blue-gray cytoplasm with many vacuoles and coarse azurophilic granules;

may contain ingested material

C. Monocyte Characteristics1. Granules are lysosomes that contain hydrolytic enzymes, including peroxidase

and acid phosphatase.2. Highly motile cell that marginates against vessel walls and into the tissues3. Reference range is 2-10% in peripheral blood.

D. Monocyte/Macrophage Function1. Play a major role in initiating and regulating the immune response2. They process ingested material and also process antigenic information,

which is relayed to the T-helper (CD4) lymphocyte. The T-helper lymphocytecoordinates the immune response to foreign antigens.

3. They arrive at the site of inflammation after neutrophils. Unlike neutrophils,the phagocytic process does not kill the monocyte.

4. Very efficient phagocytic cells with receptors for IgG or complement-coatedorganisms

5. Known as "scavenger cells" because of their ability to ingest foreign materiala. Blood monocytes ingest antigen-antibody complexes and activated

clotting factors, limiting the coagulation response.b. Splenic macrophages remove old/damaged RBCs and conserve iron for

recycling.c. Liver macrophages remove fibrin degradation products.d. Bone marrow macrophages remove abnormal RBCs, ingest bare

megakaryocyte nuclei or extruded RBC nuclei, and store and supplyiron for hemoglobin synthesis.

6. Monocytes secrete cytokines/interleukins and tumor necrosis factor.

E. Nonmalignant Monocytic Disorders1. Monocytosis

a. Increase in the absolute number of monocytes associated with:1) Recovery stage from acute bacterial infections and recovery following

marrow suppression by drugs2) Tuberculosis, syphilis, subacute bacterial endocarditis3) Autoimmune disorders (systemic lupus erythematosus, rheumatoid

arthritis)

Page 19: Hematology

LYMPHOCYTES AND PLASMA CELLS • 247

2. Lipid storage disordersa. Gaucher disease is the most common lipid storage disorder and

has an autosomal recessive inheritance pattern. A deficiency inglucocerebrosidase causes glucocerebroside to accumulate inmacrophages of the bone marrow, spleen, and liver, with Gauchercells more commonly seen in the bone marrow.

b. Niemann-Pick disease has an autosomal recessive inheritance pattern.A deficiency in sphingomyelinase causes sphingomyelin to accumulate inmacrophages in multiple organs and bone marrow, where Niemann-Pickcells can be seen.

c. Sea-blue histiocytosis is caused by an unknown deficiency. Sea-bluemacrophages are found in the spleen and bone marrow.

d. Others include Tay-Sachs and Fabry diseases3. Monocytopenia

a. Decrease in the absolute number of monocytesb. Associated with stem cell disorders such as aplastic anemia

V. LYMPHOCYTES AND PLASMA CELLS

A. Basic Review

1. The pluripotential stem cell gives rise to the lymphoid progenitor cell that isacted on by colony stimulating factors/interleukins/cytokines to form B andT lymphocytes. Pre-B lymphocytes differentiate in the bone marrow, andpre-T lymphocytes differentiate in the thymus through antigen-independent lymphopoiesis.

2. Bone marrow and thymus are primary lymphoid tissues.3. B- and T cells enter the blood and populate the secondary lymphoid tissues

(lymph nodes, spleen, and Peyer's patches in the intestine), where antigencontact occurs.

B. Maturation and Morphology of Lymphocytes

1. Lymphoblast: Earliest recognizable lymphocyte precursora. 10-18 |xm;N:C ratio 4:1b. Round/oval eccentric nucleus with fine chromatin; 1 or more nucleolic. Dark blue cytoplasm; no cytoplasmic granules

2. Prolymphocytea. 9-18 jjLm;N:C ratio 3:1b. Round or indented nucleus with coarsening chromatin; 0-1 nucleolic. Basophilic cytoplasm; no cytoplasmic granules

3. Lymphocytea. 7-18 (Jimb. Round, oval, or slightly indented nucleus; condensed chromatinc. Scant to moderate amount of blue cytoplasm; few azurophilic granules

Page 20: Hematology

248 • CHAPTER 2: HEMATOLOGY

4. Reactive lymphocytes have become activated as part of the immune response.Associated with lymphocytosis and can show the following characteristics:a. Generally, larger cell with increased amount of dark blue cytoplasm (RNA)b. Fine chromatin pattern with nucleolic. Irregular shape to the nucleusd. Irregular shape to the cytoplasm (tags, sharp ridges); indented by red cells

C. T Lymphocytes (T cells)1. Become immunocompetent in the secondary lymphoid tissue; dependent on

antigenic stimulationa. Acquire specific receptors for antigensb. Make up 80% of the peripheral blood lymphocytes

2. They are identified by membrane markers CD2, CD3, and others. The markersappear, disappear, and then reappear throughout cell development.

3. T lymphocyte functiona. T cells provide cellular immunity. They are responsible for graft

rejections and lysis of neoplastic cells, and they attack/destroy viral andfungal organisms.

b. Obtain antigenic information from monocytes; this information is passedto other T cells and B cells

c. Regulate humoral response by helping antigens activate B cellsd. End products of activation are cytokines/lymphokines/interleukins

4. Three T cell subsets are involved in the immune response and aredifferentiated by cluster designation (CD) markers.a. T helper/inducer cell (T-h, T4)

1) Identified by CD4 membrane marker2) Promotes activation of B cells by antigens

b. T suppressor cell (T-s, T8)1) Identified by CDS membrane marker2) Suppresses activation of B cells by antigens

c. Cytotoxic T cell (T-c, T8)1) Identified by CDS membrane marker2) Functions in viral infections and organ rejections

d. The normal T4:T8 ratio in circulating blood is 2:1. This ratio must bemaintained for proper immune response. It is used to monitor HIVpatients. T helper (CD4) cells are destroyed by the HIV virus, whichdecreases the ratio as the infection spreads.

D. B Lymphocytes (B cells)1. Become immunocompetent in the secondary lymphoid tissue; dependent on

antigenic stimulation.a. Acquire specific receptors for antigensb. Make up 20% of the peripheral blood lymphocytes

2. Identified by membrane markers CD19, CD20, and others

Page 21: Hematology

LYMPHOCYTES AND PLASMA CELLS • 249

3. B lymphocyte functiona. Contact with foreign antigens stimulates B lymphocytes to become

reactive lymphocytes, with the characteristic morphology associatedwith reactivity.

b. Reactive lymphocytes transform into immunoblasts, and then plasmacells that produce antibodies to provide humoral immunity.

c. Plasma cells1) End stage of B lymphocyte; dominant in lymph nodes; not normally

seen in circulation2) 10-20 |xm3) Abundant blue cytoplasm with prominent perinuclear (golgi) zone4) Eccentric nucleus with a very coarse, clumped chromatin pattern5) Make up less than 4% of nucleated cells in the bone marrow

E. Natural Killer (NK)/l_arge Granular Lymphocytes (LGLs)

1. Large cells with low N:C ratio, large cytoplasmic granules, and pale bluecytoplasm

2. Lack B cell or T cell membrane markers; are CD16 and CD56 positive3. Responsible for surveillance of cells for surface alterations such as tumor cells

or cells infected with viruses4. Activated by IL-2 to express nonspecific cytotoxic functions5. Attack antigens with attached IgG; called antibody-dependent cytotoxic cells

F. Nonmalignant Lymphocytosis Associated with Viral Infections1. Infectious mononucleosis

a. Epstein-Barr virus (EBV) infects B lymphocytes.b. Common in the 14-24 age group with symptoms ranging from malaise

and fever to pharyngitis, lymphadenopathy, and splenomegalyc. Transmitted through nasopharyngeal secretionsd. Lymphocytes usually >50% of the WBCs, with 20% being reactive

T lymphocytes attacking affected B lymphocytese. Positive heterophile antibody test

2. Cytomegalovirus (CMV)a. Symptoms similar to infectious mononucleosisb. Transmission is by blood transfusions and saliva exchange.c. 90% of lymphocytes can be reactive.d. Negative heterophile antibody teste. Transfused blood products are often tested for CMV.

3. Infectious lymphocytosisa. Associated with adenovirus and coxsackie A virusb. Contagious disease mostly affecting young childrenc. After a 12- to 21-day incubation period, symptoms appear and include

vomiting, fever, rash, diarrhea, and possible CNS involvement.d. Lymphocytosis with no reactive lymphocytes

Page 22: Hematology

250 • CHAPTER 2: HEMATOLOGY

G. Other Conditions Associated with Lymphocytosis1 . Viral—hepatitis, influenza, mumps, measles, rubella, and varicella2. Nonviral—Bordetella pertussis (whooping cough), brucellosis, toxoplasmosis

VI. MALIGNANT LEUKOCYTE DISORDERS

A. Basic Review

1. A malignant clone of cells proliferate that do not respond to normal regulatorymechanisms.a. Leukemia originates in the bone marrow and is initially systemic.b. Lymphoma originates in lymphoid tissue and is initially localized.

2. Etiology remains unclear. Multiple theories exist about oncogeneactivation, which most likely includes multiple factors:a. Viral—Viruses can suppress immune function or activate oncogenes

(HTLV-I,II,V)andHIV-l.b. Bone marrow damage—Radiation, chemicals, and malignancies

secondary to cancer treatmentsc. Chromosome defects—Some chromosomal abnormalities are diagnostic

for leukemic subtypes; t(15;17) is diagnostic for acute promyelocyticleukemia.

d. Genetic factors—Increased incidence in Down syndrome, Fanconi, andothers

e. Immune dysfunction—Hereditary and acquired defects in the immunesystem

3. Can be classified by stem cell involved and length of clinical coursea. Lymphoproliferative disorders—acute or chronicb. Myeloproliferative disorders—acute or chronic

4. Bone marrow examination used to aid in diagnosisa. Indications include:

1) Investigation of peripheral blood abnormalities, such as unexplainedcytopenias

2) Staging and management of patients with certain lymphomas or solidtumors

3) Ongoing monitoring of response to therapy in patients withmalignancy

b. Optimal sample for examination includes both the aspirate and corebiopsy specimen

c. Posterior superior iliac crest most commonly used; less commonly usedanterior iliac crest or sternum

d. Routinely assessed for cellularity, M:E ratio, megakaryocyteevaluation, iron stores, differential

e. Assessment may also include flow cytometry, cytogenetics, molecular, andmicrobiology testing

Page 23: Hematology

MALIGNANT LEUKOCYTE DISORDERS • 251

B. Comparison of Acute and Chronic Leukemias

1 . Durationa. Acute—Survival is weeks to months without treatment; death is due to

infection and bleeding.b. Chronic—Survival is years without treatment.

2. Predominant cell typea. Acute—Immature/blast cells predominate.

1) AML has myeloblasts.2) ALL has lymphoblasts.

b. Chronic—Maturing or mature cells predominate1) CML has granulocytes.2) CLL has lymphocytes.

3. Clinical manifestations and laboratory findingsa. Acute—sudden onset; affects all ages

1) Weakness and fatigue due to anemia2) Petechiae and bruising due to thrombocytopenia3) Fever and infection due to neutropenia4) Variable leukocyte count5) Marrow blasts ^20% based on World Health Organization

classification or >30% based on French-American-Britishclassification with cellularity >70%

b. Chronic—frequently asymptomatic initially; affects adults1) Anemia mild or absent2) Normal to slightly increased platelet count3) WBC count usually high4) Marrow cellularity is >70%.

c. Both acute and chronic1) Unexplained weight loss or night sweats2) Splenomegaly, hepatomegaly, lymphadenopathy

4. Treatmenta. Chemotherapy used is dependent on type of leukemia. Proper diagnosis

is crucial.b. Radiationc. Bone marrow/stem cell transplantd. Supportive with transfusions of red blood cells and platelets, antibiotics,

growth factors

C. French-American-British (FAB) and World Health Organization (WHO)1. Hematopoietic malignancy classifications2. FAB classification is based on cellular morphology and cytochemical stain

results. FAB defines acute leukemia as >30% bone marrow blasts.3. WHO classification is based on cellular morphology and cytochemical stains,

but also utilizes information obtained from immunologic probes of cell

Page 24: Hematology

252 • CHAPTER 2: HEMATOLOGY

markers, cytogenetics, molecular abnormalities, and clinical syndrome.WHO defines acute leukemia as ^20% bone marrow blasts.

4. WHO classification is now the standard for diagnosis.5. FAB classification is easier to use and is still widely taught.

D. Cytochemical Stains—Used in Diagnosis of Hematologic Disorders

1. Myeloperoxidase (MPO)a. Cells of the granulocytic series and to a lesser degree the monocytic series

contain the enzyme peroxidase in their granules that is detected by this stain.Auer rods also stain positive; lymphocytic cells are negative for this stain.

b. Used to differentiate blasts of acute myelogenous leukemias (AMLs)from acute lymphoblastic leukemias (ALLs)

2. Sudan black Ba. Stains phospholipids and lipoproteinsb. Granulocytic cells and Auer rods stain positive (blue-black granulation);

lymphocytic cells are negative for Sudan black B (reaction parallelsMPO).

c. Used to differentiate blasts of AML from ALL3. Esterases

a. Specific esterase stain (naphthol AS-D chloroacetate esterase stain)1) Detects esterase enzyme present in primary granules of granulocytic

cells; monocytic cells negative for this stainb. Nonspecific esterase stains (alpha-naphthyl acetate and alpha-naphthyl

butyrate)1) Detects esterase enzyme present in monocytic cells; granulocytic

cells negative for these stainsc. The esterase stains may be useful in distinguishing acute leukemias that

are of myeloid origin (FAB Ml, M2, M3, M4) from those leukemias thatare primarily cells of monocytic origin (FAB M5).

4. Periodic acid-Schiff (PAS)a. PAS stains intracellular glycogen bright pink.b. Immature lymphoid cells, malignant erythroblasts, and megakaryocytic

cells stain positive with this stain; myeloblasts and normal erythrocyticcells are negative with this stain.

c. Useful in diagnosis of erythroleukemia (FAB M6) and acutelymphoblastic leukemia

5. Leukocyte alkaline phosphatase (LAP)a. Detects alkaline phosphatase enzyme activity in primary granules of

neutrophilsb. A positive stain will show dark precipitate when alkaline phosphatase

activity is present; color is dependent on dye used.c. Used to differentiate chronic myelogenous leukemia (CML) from a

neutrophilic leukemoid reaction (NLR)

Page 25: Hematology

MALIGNANT LEUKOCYTE DISORDERS • 253

d. LAP score1) 100 neutrophils are graded on a scale from 0 to 4+ based on stain

intensity and size of granules. Results are added together.2) Reference range is 13-130.

e. Clinical significance1) Decreased LAP score: CML, paryoxysmal nocturnal hemoglobinuria2) Normal LAP score: CML in remission or with infection, Hodgkin

lymphoma in remission, secondary polycythemia3) Increased LAP score: Neutrophilic leukemoid reaction, polycythemia

vera, CML in blast crisis, late trimester pregnancy6. Tartrate-resistant acid phosphatase stain (TRAP)

a. Almost all blood cells contain the acid phosphatase enzyme and showpositivity with acid phosphatase stain. Once tartrate is added, staining isinhibited in most cells.

b. Only hairy cells from hairy cell leukemia are resistant to inhibition withtartrate and continue to stain positive; all other cells stain negative.

7. Perl's Prussian blue staina. Free iron precipitates into small blue/green granules in mature erythrocytes;

cells are called siderocytes. Iron inclusions are called siderotic granules orPappenheimer bodies when visible with Wright's stain.

b. Sideroblasts are nucleated RBCs in bone marrow that contain irongranules. These are normal. Ringed sideroblasts contain iron that encirclesthe nucleus. These are abnormal.

c. Increased percentage of siderocytes is seen in severe hemolytic anemias(e.g., beta-thalassemia major), iron overload, sideroblastic anemia, andpost-splenectomy; ringed sideroblasts are seen in bone marrow ofmyelodysplastic syndrome (refractory anemia with ringed sideroblasts[RARS]) and sideroblastic anemias.

E. Acute Lymphoproliferative Disorders

1. Unregulated proliferation of the lymphoid stem cell; classifiedmorphologically using FAB criteria, or immunologically using CD markersto determine cell lineage (T or B cell)

2. Clinical symptoms: Fever, bone/joint pain, bleeding, hepatosplenomegaly3. Laboratory: Neutropenia, anemia, and thrombocytopenia; variable WBC count,

hypercellular marrow with bone marrow blasts ^20% (WHO) or >30% (FAB)4. Lymphoblasts stain PAS positive; Sudan black B and myeloperoxidase

negative5. FAB classification of acute lymphoblastic leukemia (ALL)

a. FAB LI1) Most common childhood leukemia (2- to 10-year peak); also found in

young adults2) Small lymphoblasts, homogeneous appearance

Page 26: Hematology

254 • CHAPTER 2: HEMATOLOGY

3) Best prognosis4) Most T cell ALLs are FAB LI.

b. FABL21) Most common in adults2) Large lymphoblasts, heterogeneous appearance

c. FABL31) Leukemic phase of Burkitt lymphoma2) Seen in both adults and children3) Lymphoblasts are large and uniform with prominent nucleoli;

cytoplasm stains deeply basophilic and may show vacuoles.4) Poor prognosis5) ALL FAB L3s are of B cell lineage.

d. Burkitt lymphoma1) High-grade non-Hodgkin lymphoma phase of FAB L3 leukemia2) Endemic in East Africa with high association with Epstein-Barr virus;

children present with jaw/facial bone tumors3) U.S. variant seen in children and young adults; present with abdominal

mass6. Immunophenotyping of ALL

a. CD marker characteristics of B cell lineage1) Expressed by specific cell lines at different maturation stages; as cell

matures, loses some antigens and expresses new ones2) Progenitor B cells are CD19, CD34, and TdT (terminal

deoxynucleotidyl transferase) positive; CD10 (CALLA) negative.This is the least mature B cell.

3) Early-pre-B cells ALL are CD10 (CALLA), CD19, CD34, and TdTpositive. This is the most common subtype.

4) Pre-B cells ALL are CD10 (CALLA), CD19, CD20, and TdTpositive. This is the second most common subtype.

5) B cells ALL (early B) are CD19, CD20 positive; TdT negative. Thisis the most mature B cell and least common subtype.

6) CD19 is the only marker expressed through all stages of B cells.b. CD marker characteristics of T cell lineage

1) Differentiated from B cells using markers present on all T cells,including CD2, CD3, CDS, and CD7 (pan T cell markers). ImmatureT cells are TdT positive.

2) Immature T cells can have both or neither CD4 and CDS. Mature Tcells have one or the other, but not both.

3) T cell ALL occurs most often in males; mediastinal mass is acommon finding.

7. Genetic translocations are helpful in diagnosis. Common ones include:a. FAB L3/Burkitt lymphoma—1(8;14) with a rearrangement of the MYC

oncogene

Page 27: Hematology

MALIGNANT LEUKOCYTE DISORDERS • 255

b. Pre-B cell ALL associated with t(9;22); B cell ALL associated with

c. T cell ALL associated with t(7;ll)

F. Chronic Lymphoproliferative Disorders1. Chronic lymphocytic leukemia (CLL)

a. Found in adults over 60 years old; more common in males (2:1); survivalrate of 5-10 years

b. B cell malignancy (CD19, CD20 positive)c. Often asymptomatic and diagnosed secondary to other conditionsd. Laboratory: Bone marrow hypercellular; blood shows absolute

lymphocytosis of >5.0 X 109/L; homogeneous, small, hyperclumpedlymphocytes and smudge cells

e. Anemia is not usually present unless secondary to warm autoimmunehemolytic anemia (frequent complication).

f. Small lymphocyte lymphoma (SLL) is the lymphoma phase of CLL.2. Hairy cell leukemia (HCL)

a. Found in adults over 50 years old; more common in males (7:1)b. B cell malignancy (CD19, CD20 positive)c. Massive splenomegaly; extensive bone marrow involvement results in

dry tap on bone marrow aspirationd. Laboratory: Pancytopenia; cytoplasm of lymphocytes shows hair-like

projections; hairy cells are tartrate-resistant acid phosphatase (TRAP)stain positive

3. Prolymphocytic leukemia (PLL)a. Found in adults; more common in malesb. Can be either B cell (most common) or T cell malignancyc. Marked splenomegalyd. Laboratory: Characterized by lymphocytosis (>100 X 109/L) with many

prolymphocytes; anemia and thrombocytopeniae. Both B and T cell types are aggressive and respond poorly to treatment.

G. Other Lymphoid Malignancies1 . Plasma cell neoplasms

a. Multiple myeloma1) Monoclonal gammopathy causes B cell production of excessive IgG

(most common) or IgA, with decreased production of the otherimmunoglobulins.

2) Found in adults over 60 years old; incidence higher in males3) Multiple skeletal system tumors of plasma cells (myeloma cells)

cause lytic bone lesions and hypercalcemia.4) Identified on serum protein electrophoresis by an "M"-spike in

the gamma-globulin region; immunoglobulin class determined

Page 28: Hematology

256 m CHAPTER 2: HEMATOLOGY

using immunoelectrophoresis and quantified using an immunoassaymethod

5) Excessive IgG or IgA production by myeloma cells causes increasedblood viscosity.

6) Abnormal immunoglobulin binds to platelets, blocking receptor sitesfor coagulation factor binding; this results in prolonged bleeding.

7) Laboratory: Bone marrow plasma cells >30%, marked rouleaux,increased erythrocyte sedimentation rate (ESR), blue background toblood smear, plasma cells and lymphocytes on blood smear

8) Bence Jones proteins (free light chains—kappa or lambda) found inthe urine; toxic to tubular epithelial cells; cause kidney damage

b. Waldenstrom macroglobulinemia1) Monoclonal gammopathy causes B cell production of excessive IgM

(macroglobulin) and decreased production of the otherimmunoglobulins.

2) Found in adults over 60 years old3) Lymphadenopathy and hepatosplenomegaly; no bone tumors4) Identified on serum protein electrophoresis by an "M"-spike in the

gamma-globulin region; immunoglobulin class determined usingimmunoelectrophoresis and quantified using an immunoassay method

5) Excessive IgM production causes increased blood viscosity.6) Abnormal immunoglobulin may interfere with platelet function, fibrin

polymerization, and the function of other coagulation proteins.7) Laboratory: Marked rouleaux, increased ESR, blue background to

blood smear; plasmacytoid lymphocytes, plasma cells, andlymphocytes on blood smear

2. Lymphomaa. Proliferation of malignant cells in solid lymphatic tissueb. Initially localized; may spread to bone marrow and bloodc. Clinical symptom: Lymphadenopathyd. Diagnosis: Tissue biopsy, CD surface markers, cytogenetics, DNA

analysis/PCRe. World Health Organization (WHO) groups the lymphomas into Hodgkin,

B cell, and T/NK cell (non-Hodgkin) neoplasms.f. Hodgkin lymphoma (classical)

1) 40% of lymphomas; seen in patients between 15 and 35 years of ageand over 55 years of age; seen more frequently in males; certainsubtypes have an Epstein-Barr virus (EBV) association

2) Reed-Sternberg (RS) cells found in lymph node biopsy are large, multi-nucleated cells each with prominent, large nucleoli; B cell lineage

3) Hodgkin lymphoma subtypes using WHO classification:a) Nodular sclerosis—70% are this subtype; lowest EBV associationb) Mixed cellularity—20% are this subtype; highest EBV association

Page 29: Hematology

MALIGNANT LEUKOCYTE DISORDERS • 257

c) Lymphocyte richd) Lymphocyte depleted—uncommone) All subtypes are associated with RS cells

4) Laboratory: Mild anemia, eosinophilia, and monocytosis; increasedLAP score and ESR during active disease

g. Non-Hodgkin lymphoma1) WHO separates B cell and T/NK cell neoplasms into conditions with

precursor cells or mature cells.2) 60% of lymphomas; seen in patients over 50 years of age; seen more

frequently in males3) Enlarged lymph nodes or gastrointestinal (GI) tumors4) B cell neoplasms are more common; include Burkitt (lymphoma phase

of Burkitt leukemia), mantle cell, follicular, and other lymphomas5) Cells can be small and mature (e.g., small lymphocytic lymphoma) or

large and primitive (e.g., Precursor B cell lymphoblastic lymphoma).6) Can be slow growing or very aggressive

h. Mycosis fungoides (cutaneous T cell lymphoma)1) Classified by WHO as a T/NK cell neoplasm (non-Hodgkin

lymphoma)2) Seen in patients over 50 years of age3) Cutaneous lymphoma causes skin itching, leading to ulcerative

tumors.4) Sezary syndrome, a variant of mycosis fungoides, presents as a

disseminated disease with widespread skin involvement and circulatinglymphoma cells.

5) CD2, CDS, and CD4 positive

H. Acute Myeloproliferative Disorders

1. Unregulated proliferation of the myeloid stem cell; classified usingmorphology, cytochemical stains, CD markers, cytogenetics; WHOclassification standard for diagnosis; FAB classification still widelytaught

2. Platelets, erythrocytes, granulocytes, and/or monocytes can be affected.3. Found mainly in middle-aged adults; also children < 1 year old4. Clinical symptoms: Fever, malaise, weight loss, petechiae, bruises, mild

hepatosplenomegaly5. Laboratory: Neutropenia, anemia, and thrombocytopenia; variable WBC

count; hypercellular marrow with bone marrow blasts ^20% (WHO) or>30% (FAB)

6. Acute myelogenous leukemia (AML)a. FAB MO—Blasts exhibit myeloid markers CD13, CD33, and CD34 but

stain negatively with the usual cytochemical stains, myeloperoxidase(MPO), and Sudan black B (SBB). Constitutes <5% of AMLs.

Page 30: Hematology

258 • CHAPTER 2: HEMATOLOGY

b. FAB Ml (AML without maturation) shows 90% or more marrowmyeloblasts; may have Auer rods (fused primary granules)

c. FAB M2 (AML with maturation) shows <90% marrow myeloblasts;may have Auer rods; chromosome abnormality t(8;21)1) Both FAB Ml and FAB M2 are SBB, MPO, and specific esterase

positive.2) FAB Ml and FAB M2 account for 50% of the AMLs.3) CD13 and CD33 positive (pan myeloid markers)

d. Acute promyelocytic leukemia (APL; FAB M3)1) Characterized by >30% marrow promyelocytes with bundles of

Auer rods (faggot cells); heavy azurophilic granulation2) Clinical symptoms: Severe bleeding, hepatomegaly, and disseminated

intravascular coagulation (promyelocytes have procoagulant activity)3) Accounts for 5% of the AMLs4) SBB, MPO, and specific esterase positive5) CD13 and CD33 positive; diagnostic chromosome abnormality

t(15;17); PML/RARA oncogene involvede. Acute myelomonocytic leukemia (AMML; FAB M4)

1) Characterized by s=20% (WHO) or >30% (FAB) marrowmyeloblasts with >20% cells of monocytic origin; may haveAuer rods

2) Proliferation of unipotential stem cell CFU-GM that gives rise to bothgranulocytes and monocytes

3) Accounts for 30% of the AMLs4) Increased urine/serum lysozyme5) SBB, MPO, and specific and nonspecific esterase positive6) CD13 and CD33 positive (myeloid) and CD14 positive (monocytes)7) M4Eo is a subclass of AMML that presents with eosinophilia.

f. Acute monocytic leukemia (AMoL; FAB MS)1) Characterized by £=20% (WHO) or >30% (FAB) marrow

monoblasts2) Accounts for 10% of the AMLs3) Nonspecific esterase positive; CD 14 positive4) Contains two variants:

a) M5a is seen in children with >80% monoblasts in the bone marrow.b) M5b is seen in middle-aged adults with <80% monoblasts in the

bone marrow.g. Acute erythroleukemia (AEL, Di Guglielmo syndrome; FAB M6)

1) Characterized by 3=20% (WHO) or >30% (FAB) marrowmyeloblasts and >50% dysplastic marrow normoblasts

2) Accounts for 5% of the AMLs3) Malignant normoblasts are PAS positive. The myeloblasts are SBB

and MPO positive.

Page 31: Hematology

MALIGNANT LEUKOCYTE DISORDERS • 259

4) Malignant normoblasts are CD45 and CD71 (glycophorin A) positive.The myeloblasts are CD13, CD15, and CD33 positive.

h. Acute megakaryocytic leukemia (AMegL; FAB M7)1) Characterized by a proliferation of megakaryoblasts and atypical

megakaryocytes in the bone marrow; blasts may have cytoplasmicblebs

2) Accounts for < 1 % of the AMLs3) Marrow aspiration results in dry tap; blood shows pancytopenia4) Difficult to diagnose with cytochemical stains5) CD41, CD42, and CD61 (platelet markers) positive

i. Bilineage leukemias contain two cell populations. One populationexpresses myeloid antigens; the other population expresses lymphoidantigens.

j. Biphenotypic leukemias occur when myeloid and lymphoid antigensare expressed on the same cell; poor prognosis

k. The WHO classification of acute myeloid leukemias has more than20 subtypes; all have ^20% marrow blasts.

I. Chronic Myeloproliferative Disorders1. Characterized by hypercellular marrow, erythrocytosis, granulocytosis,

and thrombocytosisa. Defect of the myeloid stem cellb. Named for the cell line most greatly affectedc. All may terminate in acute leukemia.

2. Molecular diagnostic studies are helpful in identifying oncogenes.a. JAK2 oncogene is implicated in polycythemia vera (80%),

chronic idiopathic myelofibrosis (50%), and essential thrombocythemia(40%).

b. The BCR/ABL oncogene is associated with chronic myelogenousleukemia.

3. Chronic myelogenous leukemia (CML) presents with proliferation ofgranulocytes.a. Found mainly in adults 45 years of age and older; often diagnosed

secondary to other conditionsb. Clinical symptoms: Weight loss, splenomegaly, fever, night sweats, and

malaisec. Bone marrow has an increased M:E ratio.d. Laboratory: Blood findings include mild anemia and WBC between

50 and 500 X 109/L, with all stages of granulocyte production (shift tothe left), including early forms of eosinophils and basophils. Myelocytespredominate; may have a few circulating blasts.

e. CML can mimic a neutrophilic leukemoid reaction (NLR). LAP score isused to differentiate; LAP is low in CML and high in NLR.

Page 32: Hematology

260 • CHAPTER 2: HEMATOLOGY

f. Philadelphia chromosome, t(9;22), is present in virtually all patients.All cell lines are affected except lymphocytes. The few who lack thechromosome have a worse prognosis.

g. Chronic phase can last up to 5 years; accelerated phase (blast crisis)ultimately leads to acute leukemia in most patients. Recent therapies areimproving the prognosis.

4. Essential thrombocythemia (ET)a. Characterized by proliferation of megakaryocytesb. Found mainly in adults 60 years of age and olderc. Laboratory: Platelets commonly greater than 1000 X 109/L, giant forms,

platelet function abnormalities, leukocytosisd. Must differentiate from reactive thrombocytosis and polycythemia vera

5. Polycythemia vera (PV)a. Malignant hyperplasia of the multipotential myeloid stem cell causes

increase in all cell lines (polycythemia); erythrocytes most greatlyincreased despite decreased erythropoietin (EPO); inappropriateerythropoiesis

b. High blood viscosity can cause high blood pressure, stroke, and heart attack.c. Found in adults 50 years of age and olderd. Laboratory: Increased RBC (7-10 X 1012/L), hemoglobin (>20 g/dL),

and hematocrit (>60%) along with increased leukocytes and plateletsindicate polycythemia. RBC mass is increased with a normal plasmavolume.

e. Treatment is therapeutic phlebotomy, splenectomy, and chemotherapy.PV is a chronic disease with a life expectancy after diagnosis of up to20 years.

f. Must differentiate from other forms of polycythemia1) Secondary polycythemia

a) Increase in RBC mass is an appropriate response to increasedEPO or tissue hypoxia. Plasma volume, leukocyte count, andplatelet count are normal.

b) Can be caused by smoking, emphysema, or high altitude2) Relative (pseudo-) polycythemia

a) Decreased plasma volume with a normal RBC mass caused bydehydration (diarrhea, diuretics, or burns)

b) Increased hemoglobin, normal leukocyte and platelet count,normal EPO

6. Chronic idiopathic myelofibrosisa. Myeloid stem cell disorder characterized by proliferation of erythroid,

granulocytic, and megakaryocytic precursors in marrow with dyspoiesisb. Progressive marrow fibrosisc. Found in adults 50 years of age and olderd. Clinical symptoms: Bleeding due to abnormal platelet function;

extramedullary hematopoiesis causes splenomegaly and hepatomegaly

Page 33: Hematology

MALIGNANT LEUKOCYTE DISORDERS • 261

e. Laboratory: Anisocytosis, poikilocytosis with teardrop cells,leukoerythroblastic anemia (immature neutrophils and nucleated RBCs incirculation); abnormal morphology associated with all cell lines

J. Myelodysplastic Syndromes (MDSs)

1 . Basic Reviewa. Group of acquired clonal disorders affecting the pluripotential stem

cell; characterized by progressive blood cytopenias despite bone marrowhyperplasia

b. Dyspoiesis affects erythroid, myeloid, and megakaryocytic cell lines. Highincidence of terminating in acute myelogenous leukemia occurs.

c. MDS development can be triggered by chemotherapy, radiation, andchemicals.

d. Found in older adults; rarely found in children and young adultse. Hematologic evidence of dyspoiesis:

1) Erythroid: Variable anemia; erythrocytes can be macrocytic (with ovalmacrocytes) or microcytic and hypochromic; dimorphic erythrocytes,poikilocytosis, Howell-Jolly bodies, basophilic stippling, Cabot rings,nucleated RBCs

2) Myeloid: Neutropenia, hypogranulation, hyposegmentation ofneutrophils, shift to the left

3) Thrombocytes: Variable platelet count, giant platelets,hypogranulation, micromegakaryocytes

f. Five subgroups of MDS using the FAB classification scheme; up to 30%blasts in the bone marrow

2. Refractory anemia (RA)a. Anemia that is refractory (not responsive) to therapyb. Laboratory: Oval macrocytes, reticulocytopenia, dyserythropoiesis; bone

marrow blasts <5% and peripheral blood blasts <1%3. Refractory anemia with ringed sideroblasts (RARS)

a. Ringed sideroblasts comprise more than 15% of bone marrow nucleatedcells. Signs of dyserythropoiesis, neutropenia

b. Laboratory: Similar to RA; dimorphic erythrocytesc. This is the primary/idiopathic sideroblastic anemia discussed with the

anemias.4. Chronic myelomonocytic leukemia (CMML)

a. The one MDS that usually presents with leukocytosisb. Laboratory: Bone marrow blasts 5-20% and peripheral blood blasts

<5%; absolute monocytosis greater than 1.0 X 109/L5. Refractory anemia with excess blasts (RAEB)

a. Trilineage cytopenias commonb. Laboratory: Bone marrow and peripheral blood blasts are the same as

with CMML, but there is no absolute monocytosis.c. The higher the blast percent, the worse the prognosis.

Page 34: Hematology

262 • CHAPTER 2: HEMATOLOGY

6. Refractory anemia with excess blasts in transformation (RAEB-t)a. Laboratory: bone marrow blasts >20% but less than 30%; peripheral

blood blasts >5%b. WHO classification reassigns RAEB-t as an acute leukemia instead of a

myelodysplastic syndrome because of the bone marrow blast percent.7. WHO classification of MDS has additional groups (e.g., refractory cytopenia

with multilineage dysplasia, 5q deletion syndrome).8. WHO created the new category of myelodysplastic/myeloproliferative

disease, which includes the FAB's CMML.

VII. ERYTHROCYTES

A. General Characteristics1 . Oxygen transport, removal of metabolic waste2. Loss of nucleus is required for function.3. Normal life span is 120 days.

B. Erythropoietin1. Produced mainly by the kidneys2. Growth factor that stimulates erythrocyte production from myeloid

progenitor cell; influences colony-forming unit-erythrocytes (CFU-Es) todifferentiate into erythroblasts

C. Erythrocyte Maturation1. Pronormoblast (rubriblast)

a. Earliest RBC, size up to 20 |xm, with an N:C ratio of 8:1b. 1-3 nucleoli, nucleus has dark areas of DNAc. Chromatin is fine and uniform, and stains intenselyd. Deep blue cytoplasm with no granules

2. Basophilic normoblast (prorubricyte)a. Size up to 16 jjim with an N:C ratio of 6:1b. Centrally located nucleus with 0-1 nucleolic. Chromatin is coarsening.d. Cytoplasm is less blue but intensely basophilic (RNA).

3. Polychromatophilic normoblast (rubricyte)a. Size up to 12 (Jim with an N:C ratio of 4:1b. Eccentric nucleus with no nucleolic. Chromatin shows significant clumping.d. Begins to produce hemoglobin, resulting in gray-blue cytoplasm

4. Orthochromic normoblast (metarubricyte)a. Size up to 10 fxm with an N:C ratio of 0.5:1b. Eccentric nucleus with small, fully condensed (pyknotic) nucleus; no nucleolic. Pale blue to salmon cytoplasmd. Hemoglobin synthesis decreases

Page 35: Hematology

ERYTHROCYTES • 263

5. Reticulocytea. Size up to 10 |jimb. A reticulocyte contains no nucleus but has mitochondria and ribosomes.c. Last stage to synthesize hemoglobind. Last stage in bone marrow before release to the bloode. Reference ranges are 0.5-1.5% for adults and 2.5-6.5% for newborns,

with slightly increased ranges at higher altitudes.f. A supravital stain is used to enumerate reticulocytes.g. Reticulocyte count is one of the best indicators of bone marrow function,h. Stress reticulocytes are young cells released from bone marrow after older

reticulocytes have been released. This is a response to increased need,i. Hemoglobin continues to be produced by reticulocytes for approximately

24 hours after exiting the bone marrow.6. Mature erythrocyte

a. Size range is 6-8 (Jim.b. Round, biconcave discocytec. Salmon with central pallor (clearing in the center) when a blood smear is

Wright's stained1) Normal cells have a central pallor that is one-third the diameter of the

cell.2) Decreased central pallor is seen with spherocytic disorders,

including thermal injury and liver disease.3) Central pallor greater than one-third the diameter of the cell is seen in

microcytic anemias.d. RBC reference ranges in SI units:

1) Females 4.0-5.4 X 1012/L (conventional units 4.0-5.4 X 106/|xL)2) Males 4.6-6.0 X 1012/L (conventional units 4.6-6.0 X 106/(xL)

e. Erythropoiesis is regulated by erythropoietin produced in the kidney.Additional regulation includes:1) Hypoxia due to high altitudes, heart or lung dysfunction, anemia2) Androgens (male hormones that appear to enhance the activity of

erythropoietin) and hemolytic anemias (increased erythrocytedestruction)

D. Erythrocyte Physiology

1. Early RBCs get energy from oxidative phosphorylation. During maturation,the mitochondria are lost, and energy is derived from glycolysis.

2. Erythrocytes need proper volume ratio for exchange of blood gasesand flexibility to travel through capillaries. This is accomplished by thecation pump, a mechanism that keeps sodium outside and potassium insidethe cell.

3. Erythrocyte membrane is 50-60% lipid (phospholipids, cholesterol, andglycolipids) and 40-50% protein.

Page 36: Hematology

264 • CHAPTER 2: HEMATOLOGY

E. Substances Needed for Erythropoiesis

1. Iron: Must be in the ferrous state (Fe2+) to transport oxygen2. Amino acids: Globin-chain synthesis3. Folic acid/vitamin B12: DNA replication/cell division4. Others: Erythropoietin, vitamin 65 (pyridoxine), trace minerals

F. Erythrocytic Morphology and Associated Disease (Size and Shape)

1. Normocytes (discocytes) are normal erythrocytes that are approximately thesame size as the nucleus of a small lymphocyte.

2. Macrocytesa. RBCs greater than 8 |xm in diameter; MCV greater than 100 fLb. Seen in megaloblastic anemias, such as B12/folate deficiencyc. Seen in non-megaloblastic anemia of liver disease or accelerated

erythropoiesis; also seen in normal newborns3. Microcytes

a. RBCs less than 6 ixm in diameter; MCV less than 80 fLb. Seen in iron-deficiency anemia, thalassemias, sideroblastic anemia, and

anemia of chronic disease4. Anisocytosis

a. Variation in RBC size, indicating a heterogeneous RBC population(dimorphism)

b. Correlates with RDW (red blood cell distribution width), especially whenthe RDW exceeds 15.0%

c. Seen post-transfusion, post-treatment for a deficiency (e.g., iron), presenceof two concurrent deficiencies (e.g., iron and vitamin B12), and idiopathicsideroblastic anemia

5. Poikilocytosisa. General term to describe variation in shapeb. Associated with a variety of pathologic conditions

6. Echinocytes include crenated and burr cellsa. Have evenly spaced round projections; central pallor area presentb. Seen in liver disease, uremia, heparin therapy, pyruvate kinase deficiency,

or as artifactc. Caused by changes in osmotic pressure

7. Acanthocytes (spur cells)a. Have unevenly spaced pointed projections; lack a central pallor areab. Associated with alcoholic liver disease, post-splenectomy, and

abetalipoproteinemiac. Caused by excessive cholesterol in the membrane

8. Target cells (codocytes or Mexican hat cells)a. Show a central area of hemoglobin surrounded by a colorless ring and a peri-

pheral ring of hemoglobin; cells have an increased surface-to-volume ratio

Page 37: Hematology

ERYTHROCYTES • 265

b. Seen in liver disease, hemoglobinopathies, thalassemia, iron-deficiencyanemia

c. Caused by excessive cholesterol in the membrane or a hemoglobindistribution imbalance

9. Spherocytesa. Disk-shaped cell with a smaller volume than a normal erythrocyte; cells

have a decreased surface-to-volume ratiob. Lack a central pallor areac. Associated with defects of the red cell membrane proteinsd. MCHC may be >37%; increased osmotic fragilitye. Damaged RBC; seen in hereditary spherocytosis, G6PD deficiency, and

immune hemolytic anemiasf. Microspherocytes (<4 |jim) are frequently seen in severe thermal injury

(burns).10. Teardrops (dacryocytes)

a. Pear-shaped cell with one blunt projectionb. Seen in megaloblastic anemias, thalassemia, and extramedullary

hematopoiesis (myelofibrosis, myelophthisic anemia)1 1 . Sickle cells (drepanocytes)

a. Shapes vary but show thin, elongated, pointed ends and will appearcrescent shaped; usually lack a central pallor area

b. Contain polymers of abnormal hemoglobin Sc. Seen in hemoglobinopathies SS, SC, SD, and S/(3-thalassemiad. Cell shape is caused by cell membrane alterations due to an amino acid

substitution12. Helmet cells (horn cells or keratocytes)

a. Interior portion of cell is hollow, resembling a horn or helmetb. Seen in microangiopathic hemolytic anemias

13. Schistocytes (RBC fragments)a. Damaged RBC; fragments of various sizes and shapes are present, often

with pointed projectionsb. Seen in microangiopathic hemolytic anemias (e.g., DIC, HUS, TTP),

thermal injury, renal transplant rejection, and G6PD deficiency14. Stomatocytes (mouth cells)

a. Characterized by an elongated or slit-like area of central pallorb. Seen in liver disease, hereditary stomatocytosis, or as artifactc. Caused by osmotic changes due to cation imbalance (Na+/K+)

15. Elliptocytes (ovalocytes)a. Cigar- to egg-shaped erythrocytesb. Associated with defects of the red cell membrane proteinsc. Seen in hereditary elliptocytosis, iron-deficiency anemia (pencil forms),

megaloblastic anemia (macro-ovalocytes), thalassemia major

Page 38: Hematology

CHAPTER 2: HEMATOLOGY

G. Erythrocyte Inclusions and Associated Diseases

1. Nucleated RBCs (nRBCs, nucRBCs)a. Usually orthochromic normoblasts (metarubricyte) but can appear in any

erythrocytic stage of maturationb. Indicate bone marrow stimulation or increased erythropoiesisc. Associated with thalassemia major, sickle cell anemia, and other

hemolytic anemias, erythroleukemia, and myeloproliferative disordersd. Normal newborns can have a few.e. Healthy individuals should have none on a peripheral blood smear.

2. Howell-Jolly bodiesa. Small, round DNA fragments (0.5-1.0 jxm in diameter) usually one per

cell, but can be multipleb. Stain dark purple to black with Wright's stainc. Not seen in normal erythrocytes; normally pitted by splenic

macrophagesd. Seen in sickle cell anemia, beta-thalassemia major, and other severe

hemolytic anemias, megaloblastic anemia, alcoholism, post-splenectomy3. Basophilic stippling

a. Multiple, tiny, fine, or coarse inclusions (ribosomal RNA remnants)evenly dispersed throughout the cell; "blueberry bagel" appearance

b. Stain dark blue with Wright's stainc. Seen in thalassemias, megaloblastic anemias, sideroblastic anemia, lead

poisoning, and alcoholism4. Pappenheimer bodies

a. Small, irregular, dark-staining iron granules usually clumped togetherat periphery of the cell

b. Stain with Perl's Prussian blue stain; appear dark violet with Wright'sstain

c. Caused by an accumulation of ribosomes, mitochondria, and ironfragments

d. Seen in sideroblastic anemia, hemoglobinopathies, thalassemia,megaloblastic anemia, myelodysplastic syndrome (RARS)

5. Cabot ringsa. Thin, red-violet, single to multiple ringlike structures that may appear in

loop or figure-eight shapesb. Seen in megaloblastic anemia, myelodysplastic syndromes, lead poisoningc. Composed of fragments of nuclear material

6. Hemoglobin C crystalsa. Condensed, intracellular, rod-shaped crystalb. Seen in hemoglobin C or SC disease, but not in trait

7. Hemoglobin SC crystals (Washington monument)a. 1-2 blunt, fingerlike projections extending from the cell membraneb. Seen in hemoglobin SC disease

Page 39: Hematology

ERYTHROCTTES • 267

8. Heinz bodiesa. Multiple inclusions ranging in size from 0.3 to 2.0 (Jimb. Invisible with Wright's stain; must use a supravital stain to visualizec. Seen in G6PD deficiency, beta-thalassemia major, Hgb H disease, unstable

hemoglobinopathies, drug-induced anemiasd. Represent denatured hemoglobin

9. Malarial parasites—include P. vivax, P. falciparum, P. malariae and P. ovale

H. Erythrocyte Hemoglobin Content and Associated Diseases

1. Normochromasia: Cells have the normal one-third clear, central pallor area2. Hypochromasia

a. Central pallor area is greater than one-third the diameter of the cellb. MCH and MCHC usually decreasedc. Often associated with microcytosisd. Seen in iron-deficiency anemia, thalassemias, anemia of chronic disease,

sideroblastic anemia, myelodysplastic syndromes3. Polychromasia

a. Variation in hemoglobin content showing a slight blue tinge when stainedwith Wright's stain; residual RNA

b. Indicates reticulocytosis; supravital reticulocyte stain to enumeratec. Usually slightly macrocytic

4. Hyperchromasia (term no longer used)a. Current terminology is spherocyte; lacks a central pallor area

I. Abnormal Erythrocyte Distributions and Associated Diseases1. Rouleaux

a. Stacking or "coining" pattern of erythrocytes due to abnormal orincreased plasma proteins

b. May see excessively blue color to smear macroscopically andmicroscopically

c. Seen in hyperproteinemia, multiple myeloma, Waldenstrommacroglobulinemia, and conditions that produce increased fibrinogen(chronic inflammation)

d. May be artifact; considered normal in thicker area of the peripheral smeare. True rouleaux formation is determined in the thin area of the peripheral

smear.2. Agglutination

a. Characterized by clumping of erythrocytes with no patternb. Occurs when erythrocytes are coated with IgM antibodies and

complementc. Seen in cold autoimmune hemolytic anemia (cold agglutinin disease)d. Warm blood to 37°C to correct a false low RBC and hematocrit, and false

high MCHC (>37 g/dL) when using an automated cell counting instrument.

Page 40: Hematology

268 • CHAPTER 2: HEMATOLOGY

VIII. HEMOGLOBIN

A. Introduction

1. Hemoglobin is an oxygen-transporting protein contained within erythrocytes.2. The heme portion of hemoglobin gives erythrocytes their characteristic red

color.

B. Hemoglobin Structure1. Four identical heme groups, each consisting of a protoporphyrin ring and

ferrous (Fe24) iron2. Four globin (polypeptide) chains

a. Alpha chains have 141 amino acids.b. Beta, gamma, and delta chains have 146 amino acids.

3. The amino acid sequence of the globin chain determines the type ofhemoglobin; normal adult hemoglobin consists of two alpha and twonon-alpha chains in pairs.

C. Hemoglobin Synthesis1. 65% hemoglobin synthesis occurs in immature nRBCs.2. 35% hemoglobin synthesis occurs in reticulocytes.3. Heme synthesis occurs in the mitochondria of normoblasts and is

dependent on glycine, succinyl coenzyme A, aminolevulinic acid synthetase,and vitamin B6 (pyridoxine).

4. Globin synthesis occurs in the ribosomes, and it is controlled onchromosome 16 for alpha chains and chromosome 11 for all other chains.

5. Each globin chain binds to a heme molecule in the cytoplasm of theimmature RBC.

D. Hemoglobin/Erythrocyte Breakdown1. Intravascular hemolysis (10%)

a. Occurs when hemoglobin breaks down in the blood and freehemoglobin is released into plasma

b. Free hemoglobin binds to haptoglobin (major free hemoglobin transportprotein), hemopexin, and albumin, and it is phagocytized by livermacrophages.

c. Laboratory: Increased plasma hemoglobin, serum bilirubin, serum LD,and urine urobilinogen; hemoglobinuria and hemosiderinuria present;decreased serum haptoglobin

2. Extravascular hemolysis (90%)a. Occurs when senescent/old RBCs are phagocytized by macrophages in

the liver or spleenb. Protoporphyrin ring metabolized to bilirubin and urobilinogen;

excreted in urine and fecesc. Globin chains are recycled into the amino acid pool for protein synthesis.

Page 41: Hematology

HEMOGLOBIN • 269

d. Iron binds to transferrin and is transported to bone marrow for new RBCproduction, or it is stored for future use in the form of ferritin or hemosiderin.

E. Hemoglobin and Iron1. Most iron in the body is in hemoglobin and must be in the ferrous state (Fe24)

to be used. Fe2+ binds to oxygen for transport to lungs and body tissues.Ferric iron (Fe3+) is not able to bind to hemoglobin, but does bind totransferrin. Iron is an essential mineral and is not produced by the body.a. Serum iron measures the amount of Fe3+ bound to transferrin.b. Total iron-binding capacity (TIBC) measures the total amount of iron

that transferrin can bind when fully saturated.c. Serum ferritin is an indirect measurement of storage iron in tissues and

bone marrow.

F. Types of Hemoglobin

1. Hgb F contains two alpha- and two gamma-globin chains. Hgb F functions ina reduced oxygen environment. Hgb F predominates at birth (80%). Gammachain production switches over to beta chain production and is complete by6 months of age.a. Laboratory: Alkali denaturation test and Kleihauer-Betke acid elution

stain (Hgb F is resistant to denaturation/elution), column chromatography,radial immunodiffusion

b. Hgb F is a compensatory hemoglobin and can be increased in homozygoushemoglobinopathies and beta-thalassemia major.

2. Adulta. Hgb A contains two alpha- and two beta-globin chains.

1) Hgb A is subdivided into glycosylated fractions. Alc fraction reflectsglucose levels in the blood and is used to monitor individuals withdiabetes mellitus.

b. Hgb A2 contains two alpha- and two delta-globin chains.c. Reference range for a normal adult is 97% Hb A, 2% Hb A2, and 1% Hb F.

G. Different Forms of Normal Hemoglobin1 . Oxyhemoglobin: Hemoglobin with Fe2+ + O2; seen in arterial circulation2. Deoxyhemoglobin: Hemoglobin with Fe2+ but no O2; seen in venous circulation3. Carboxyhemoglobin: Hemoglobin with Fe2+ and carbon monoxide (CO);

hemoglobin has 200 X more affinity for CO than O2 so CO is earned insteadof O2; can result in death, but is reversible if given pure O2

4. Sulfhemoglobin: Hemoglobin with S; cannot transport O2; seldom reachesfatal levels; caused by drugs and chemicals; irreversible, not measured by thecyanmethemoglobin method

5. Methemoglobin: Hemoglobin with Fe3+; cannot transport O2; increasedlevels cause cyanosis and anemia

Page 42: Hematology

270 • CHAPTER 2: HEMATOLOGY

H. Oxygen Dissociation Curve

1. Oxygen affinity is the ability of hemoglobin to bind or release oxygen.Expressed in terms of the oxygen tension at which hemoglobin is 50%saturated with oxygen.

2. The relationship between oxygen tension and hemoglobin saturation withoxygen is described by the oxygen dissociation curve.a. Right shift decreases oxygen affinity, more O2 release to the tissues—

high 2,3-bisphosphoglycerate (formerly 2,3-diphosphoglycerate/2,3-DPG)level or increased body temperature; decreased body pH

b. Left shift increases oxygen affinity, less O2 release to the tissues—low2,3-bisphosphoglycerate (2,3-BPG) level or decreased body temperature;increased body pH

IX. ANEMIAS

A. Introduction: Anemia is defined as a decrease in erythrocytes and hemoglobin,resulting in decreased oxygen delivery to the tissues. The anemias can beclassified morphologically using RBC indices (MCV, MCH, and MCHC). Theycan also be classified based on etiology/cause. Anemia is suspected when thehemoglobin is <12 g/dL in men or <11 g/dL in women.

1. Relative (pseudo) anemiaa. RBC mass is normal, but plasma volume is increased.b. Secondary to an unrelated condition and can be transient in naturec. Causes include conditions that result in hemodilution, such as pregnancy

and volume overload.d. Reticulocyte count normal; normocytic/normochromic anemia

2. Absolute anemiaa. RBC mass is decreased, but plasma volume is normal. This is indicative

of a true decrease in erythrocytes and hemoglobin.b. Mechanisms involved include:

1) Decreased delivery of red cells into circulationa) Caused by impaired or defective productionb) Bone marrow fails to respond; reticulocytopenia

2) Increased loss of red cells from the circulationa) Caused by acute bleeding or accelerated destruction (hemolytic)b) Bone marrow can respond; reticulocytosis

B. Impaired or Defective Production Anemias

1. Iron-deficiency anemiaa. Most common form of anemia in the United Statesb. Prevalent in infants and children, pregnancy, excessive menstrual flow,

elderly with poor diets, malabsorption syndromes, chronic blood loss(GI blood loss, hookworm infection)

Page 43: Hematology

ANEMIAS • 271

c. Laboratory: Microcytic/hypochromic anemia; serum iron, ferritin,hemoglobin/hematocrit, RBC indices, and reticulocyte count low;RDW and total iron-binding capacity (TIBC) high; smear showsovalocytes/pencil forms

d. Clinical symptoms: Fatigue, dizziness, pica, stomatitis (cracks in the cornersof the mouth), glossitis (sore tongue), and koilonychia (spooning of the nails)

2. Anemia of chronic disease (ACD)a. Due to an inability to use available iron for hemoglobin productionb. Impaired release of storage iron associated with increased hepcidin levels

1) Hepcidin is a liver hormone and a positive acute-phase reactant. It playsa major role in body iron regulation by influencing intestinal ironabsorption and release of storage iron from macrophages.

2) Inflammation and infection cause hepcidin levels to increase; thisdecreases release of iron from stores.

c. Laboratory: Normocytic/normochromic anemia, or slightlymicrocytic/hypochromic anemia; increased ESR; normal to increasedferritin; low serum iron and TIBC

d. Associated with persistent infections, chronic inflammatory disorders(SLE, rheumatoid arthritis, Hodgkin lymphorna, cancer)

e. Anemia of chronic disease is second only to iron deficiency as a commoncause of anemia

3. Sideroblastic anemiaa. Caused by blocks in the protoporphyrin pathway resulting in defective

hemoglobin synthesis and iron overloadb. Excess iron accumulates in the mitochondrial region of the immature

erythrocyte in the bone marrow and encircles the nucleus; cells are calledringed sideroblasts.

c. Excess iron accumulates in the mitochondrial region of the matureerythrocyte in circulation; cells are called siderocytes; inclusions aresiderotic granules (Pappenheimer bodies on Wright's stained smears).

d. Siderocytes are best demonstrated using Perl's Prussian blue stain.e. Two types of sideroblastic anemia:

1) Primary—irreversible; cause of the blocks unknowna) Two RBC populations (dimorphic) are seen.b) This is one of the myelodysplastic syndromes—refractory anemia

with ringed sideroblasts (RARS).2) Secondary—reversible; causes include alcohol, anti-tuberculosis

drugs, chloramphenicolf. Laboratory: Microcytic/hypochromic anemia with increased ferritin

and serum iron; TIBC is decreased4. Lead poisoning

a. Multiple blocks in the protoporphyrin pathway affect heme synthesis.b. Seen mostly in children exposed to lead-based paint

Page 44: Hematology

272 • CHAPTER 2: HEMATOLOGY

c. Clinical symptoms: Abdominal pain, muscle weakness, and a gum leadline that forms from blue/black deposits of lead sulfate

d. Laboratory: Normocytic/normochromic anemia with characteristiccoarse basophilic stippling

5. Porphyriasa. These are a group of inherited disorders characterized by a block in the

protoporphyrin pathway of heme synthesis. Heme precursors before theblock accumulate in the tissues, and large amounts are excreted in urineand/or feces.

b. Clinical symptoms: Photosensitivity, abdominal pain, CNS disordersc. Hematologic findings are insignificant.

6. Megaloblastic anemiasa. Defective DNA synthesis causes abnormal nuclear maturation; RNA synthesis

is normal, so the cytoplasm is not affected. The nucleus matures slowerthan the cytoplasm (asynchronism). Megaloblastic maturation is seen.

b. Caused by either a vitamin B12 or folic acid deficiencyc. Laboratory: Fancytopenia, macrocytic/normochromic anemia with oval

macrocytes and teardrops, hypersegmented neutrophils; inclusions includeHowell-Jolly bodies, nucleated RBCs, basophilic stippling, Pappenheimerbodies, and Cabot rings; elevated LD, bilirubin, and iron levels due todestruction of fragile, megaloblastic cells in the blood and bone marrow

d. Vitamin Bj2 deficiency (cobalamin)1) Intrinsic factor is secreted by parietal cells and is needed to bind

vitamin Bj2 for absorption into the intestine.a) Pernicious anemia: Caused by deficiency of intrinsic factor,

antibodies to intrinsic factor, or antibodies to parietal cellsb) Prevalent in older adults of English, Irish, and Scandinavian descentc) Characterized by achlorhydria and atrophy of gastric parietal cells

2) Other causes of vitamin B12 deficiency include malabsorptionsyndromes, Diphyllobothrium latum tapeworm, total gastrectomy,intestinal blind loops, and a total vegetarian diet.

3) Clinical symptoms: Jaundice, weakness, sore tongue (glossitis), andgastrointestinal (GI) disorder, numbness and other CNS problems

4) Vitamin B12 deficiency takes 3-6 years to develop because of highbody stores.

e. Folic acid deficiency causes a megaloblastic anemia with a blood pictureand clinical symptoms similar to vitamin B12 deficiency, except there is noCNS involvement. It is associated with poor diet, pregnancy, orchemotherapeutic anti-folic acid drugs such as methotrexate. Folic acidhas low body stores.

7. Non-megaloblastic macrocytic anemias include alcoholism, liver disease,and conditions that cause accelerated erythropoiesis. The erythrocytes areround, not oval as is seen in the megaloblastic anemias.

Page 45: Hematology

ANEMIAS • 273

8. Aplastic anemiaa. Bone marrow failure causes pancytopenia.b. Laboratory: Decrease in hemoglobin/hematocrit and reticulocytes;

normocytic/normochromic anemia; no response to erythropoietinc. Most commonly affects people around the age of 50 and above. It can

occur in children.d. Patients have a poor prognosis with complications that include bleeding,

infection, and iron overload due to frequent transfusion needs.e. Treatment includes bone marrow or stem cell transplant and

immunosuppression.f. Can be genetic, acquired, or idiopathic

1) Genetic aplastic anemia (Fanconi anemia)a) Autosomal recessive traitb) Dwarfism, renal disease, mental retardationc) Strong association with malignancy development, especially acute

lymphoblastic leukemia2) Acquired aplastic anemia (secondary) caused by:

a) Antibiotics: Chloramphenicol and sulfonamidesb) Chemicals: Benzene and herbicidesc) About 30% of acquired aplastic anemias are due to drug exposure.d) Viruses: B19 parvovirus secondary to hepatitis, measles, CMV, and

Epstein-Barr viruse) Radiation or chemotherapyf) Myelodysplastic syndromes, leukemia, solid tumors, paroxysmal

nocturnal hemoglobinuria3) Idiopathic (primary): 50-70% of aplastic anemias have no known

cause.g. Diamond-Blackfan anemia

1) True red cell aplasia (leukocytes and platelets normal in number)2) Autosomal inheritance

9. Myelophthisic (marrow replacement) anemiaa. Hypoproliferative anemia caused by replacement of bone marrow

hematopoietic cells by malignant cells or fibrotic tissueb. Associated with cancers (breast, prostate, lung, melanoma) with bone

metastasisc. Laboratory: Normocytic/normochromic anemia; leukoerythroblastic

blood picture

C. Blood Loss Anemia

1 . Acute blood loss anemiaa. Characterized by a sudden loss of blood resulting from trauma or other

severe forms of injuryb. Clinical symptoms: Hypovolemia, rapid pulse, low blood pressure, pallor

Page 46: Hematology

274 • CHAPTER 2: HEMATOLOGY

c. Laboratory: Normocytic/normochromic anemia; initially normalreticulocyte count, hemoglobin/hematocrit; in a few hours, increase inplatelet count and leukocytosis with a left shift, drop inhemoglobin/hematocrit and RBC; reticulocytosis in 3-5 days

2. Chronic blood loss anemiaa. Characterized by a gradual, long-term loss of blood; often caused by

gastrointestinal bleedingb. Laboratory: Initially normocytic/normochromic anemia that over time

causes a decrease in hemoglobin/hematocrit; gradual loss of iron causesmicrocytic/hypochromic anemia

D. Hemolytic Anemias Due to Intrinsic Defects1. All cause a normocytic/normochromic anemia; usually hereditary with

reticulocytosis due to accelerated destruction2. Hereditary spherocytosis

a. Most common membrane defect; autosomal dominant; characterized bysplenomegaly, variable degree of anemia, spherocytes on the peripheralblood smear

b. Increased permeability of the membrane to sodiumc. Results in loss of membrane fragments; erythrocytes have decreased

surface area-to-volume ratio; rigid spherocytes culled/removed bysplenic macrophages

d. Laboratory: Spherocytes, MCHC may be >37 g/dL, increased osmoticfragility, and increased serum bilirubin

3. Hereditary elliptocytosis (ovalocytosis)a. Autosomal dominant; most persons asymptomatic due to normal

erythrocyte life span; >25% ovalocytes on the peripheral blood smearb. Membrane defect is caused by polarization of cholesterol at the ends of the

cell rather than around pallor area.4. Hereditary stomatocytosis

a. Autosomal dominant; variable degree of anemia; up to 50%stomatocytes on the blood smear

b. Membrane defect due to abnormal permeability to both sodium andpotassium; causes erythrocyte swelling

5. Hereditary acanthocytosis (abetalipoproteinemia)a. Autosomal recessive; mild anemia associated with steatorrhea, neurological

and retinal abnormalities; 50-100% of erythrocytes are acanthocytesb. Increased cholesterol:lecithin ratio in the membrane due to abnormal

plasma lipid concentrations; absence of serum p-lipoprotein needed forlipid transport

6. G6PD (glucose-6-phosphate dehydrogenase) deficiencya. Sex-linked enzyme defect; most common enzyme deficiency in the

hexose monophosphate shunt

Page 47: Hematology

ANEMIAS • 275

b. Reduced glutathione levels are not maintained because of decreasedNADPH generation.

c. Results in oxidation of hemoglobin to methemoglobin (Fe3+); denaturesto form Heinz bodies

d. Usually, not anemic until oxidatively challenged (primaquine, sulfadrugs); then severe hemolytic anemia with reticulocytosis

7. Pyruvate kinase (PK) deficiencya. Autosomal recessive; most common enzyme deficiency in Embden-

Meyerhof pathwayb. Lack of ATP causes impairment of the cation pump that controls

intracellular sodium and potassium levels.c. Decreased erythrocyte deformability reduces their life span.d. Severe hemolytic anemia with reticulocytosis and echinocytes

8. Paroxysmal nocturnal hemoglobinuria (PNH)a. An acquired membrane defect in which the red cell membrane has an

increased sensitivity for complement binding as compared to normalerythrocytes

b. Etiology unknownc. All cells are abnormally sensitive to lysis by complement.d. Characterized by: Pancytopenia; chronic intravascular hemolysis causes

hemoglobinuria and hemosiderinuria at an acid pH at night; PNH noted forlow leukocyte alkaline phosphatase (LAP) score; Ham's and sugar watertests used in diagnosis; increased incidence of acute leukemia

e. Although Ham's and sugar water tests have been traditionally used indiagnosis of PNH, the standard now used is flow cytometry to detectdeficiencies for surface expression of glycosyl phosphatidylinositol(GPI)-linked proteins such as CD55 and CD59.

E. Hemolytic Anemias Due to Extrinsic/Immune Defects1. All cause a normocytic/normochromic anemia due to defects extrinsic to

the RBC. All are acquired disorders that cause accelerated destruction withreticulocytosis.

2. Warm autoimmune hemolytic anemia (WAIHA)a. RBCs are coated with IgG and/or complement. Macrophages may

phagocytize these RBCs, or they may remove the antibody or complementfrom the RBC's surface, causing membrane loss and spherocytes.

b. 60% of cases are idiopathic; other cases are secondary to diseases that alterthe immune response (e.g., chronic lymphocytic leukemia, lymphoma);can also be drug induced.

c. Laboratory: Spherocytes, MCHC may be >37 g/dL, increased osmoticfragility, bilirubin, reticulocyte count; occasional nRBCs present; positivedirect antiglobulin test (DAT) helpful in differentiating from hereditaryspherocytosis.

Page 48: Hematology

276 • CHAPTER 2: HEMATOLOGY

3. Cold autoimmune hemolytic anemia (CAIHA or cold hemagglutinin disease)a. RBCs are coated with IgM and complement at temperatures below 37°C.

RBCs are lysed by complement or phagocytized by macrophages.Antibody is usually anti-I but can be anti-i.

b. Can be idiopathic, or secondary to Mycoplasma pneumoniae, lymphoma,or infectious mononucleosis

c. Laboratory: Seasonal symptoms; RBC clumping can be seen bothmacroscopically and microscopically; MCHC >37 g/dL; increased bilirubin,reticulocyte count; positive DAT detects complement-coated RBCs

d. If antibody titer is high enough, sample must be warmed to 37°C to obtainaccurate RBC and indices results.

4. Paroxysmal cold hemoglobinuria (PCH)a. An IgG biphasic Donath-Landsteiner antibody with P specificity fixes

complement to RBCs in the cold (less than 20°C); the complement-coatedRBCs lyse when warmed to 37°C.

b. Can be idiopathic, or secondary to viral infections (e.g., measles, mumps)and non-Hodgkin lymphoma

c. Laboratory: Variable anemia following hemolytic process; increasedbilirubin and plasma hemoglobin, decreased haptoglobin; DAT may bepositive; Donath-Landsteiner test positive

5. Hemolytic transfusion reactiona. Recipient has antibodies to antigens on donor RBCs; donor cells are

destroyed.b. ABO incompatibility causes an immediate reaction with massive

intravascular hemolysis that is complement induced.1) Usually IgM antibodies2) Can trigger DIG due to release of tissue factor from the lysed RBCs

c. Laboratory: Positive DAT, increased plasma hemoglobin6. Hemolytic disease of the newborn (HDN)

a. May be due to Rh incompatibility (erythroblastosis fetalis)1) Rh negative woman is exposed to Rh antigen from fetus and forms IgG

antibody; this antibody will cross the placenta and destroy RBCs of thenext fetus that is Rh positive.

2) Laboratory: Severe anemia, nRBCs, positive DAT; very high bilirubinlevels cause kernicterus leading to brain damage

3) Exchange transfusions in utero or shortly after birth4) No longer a common problem with use of Rh immunoglobulin (RhoGam)

b. May be due to ABO incompatibility1) Group O woman develops IgG antibody that crosses the placenta and

coats fetal RBCs when fetus is group A or B. The coated RBCs arephagocytized.

2) Laboratory: Mild or no anemia, few spherocytes, weakly positiveDAT, slightly increased bilirubin

Page 49: Hematology

HEMOGLOBINOPATHIES • 277

F. Hemolytic Anemias Due to Extrinsic/Non-lmmune Defects1. All cause a normocytic/normochromic anemia caused by trauma to the

RBC. All are acquired disorders that cause intravascular hemolysis withschistocytes and thrombocytopenia.

2. Microangiopathic hemolytic anemias (MAHAs)a. Disseminated intravascular coagulation (DIC)

1) Systemic clotting is initiated by activation of the coagulation cascadedue to toxins or conditions that trigger release of procoagulants (tissuefactor). Multiple organ failure can occur due to clotting.

2) Fibrin is deposited in small vessels, causing RBC fragmentation.b. Hemolytic uremic syndrome (HUS)

1) Occurs most often in children following a gastrointestinal infection(e.g., E. coli)

2) Clots form, causing renal damage.c. Thrombotic thrombocytopenic purpura (TTP)

1) TTP occurs most often in adults.2) It is likely due to a deficiency of the enzyme ADAMTS 13 that is

responsible for breaking down large von Willebrand factor multimers.When multimers are not broken down, clots form, causing RBCfragmentation and central nervous system impairment.

3. March hemoglobinuria: Transient hemolytic anemia that occurs afterforceful contact of the body with hard surfaces (e.g., marathon runners,tennis players)

4. Other causesa. Infectious agents (e.g., P. falcipamm, Clostridium perfringens) damage

the RBC membrane. Schistocytes and spherocytes are seen on the bloodsmear.

b. Mechanical trauma, caused by prosthetic heart valves (Waring blendersyndrome), chemicals, drugs, and snake venom, damage the RBCsthrough various mechanisms.

c. Thermal burns (third degree) cause direct damage to the RBC membrane,producing acute hemolysis, which is characterized by severe anemia withmany schistocytes and micro-spherocytes.

X. HEMOGLOBINOPATHIES

A. Introduction: These are a group of inherited disorders causing structurallyabnormal globin chain synthesis due to amino acid substitutions (qualitativedefect); changes in RBC deformability and electrophoretic mobility can occur.Homozygous/disease conditions (both globin chains affected) are more seriousthan heterozygous/trait conditions (only one globin chain affected). Target cells areassociated with the hemoglobinopathies. Hemoglobin electrophoresis, isoelectricfocusing and/or DNA (PCR) analysis may be used to confirm the diagnosis. The

Page 50: Hematology

278 • CHAPTER 2: HEMATOLOGY

amino acid substitution causing formation of Hgb S is the most common, Hgb Cis the second most common, and Hgb E is the third most common.

B. Sickle Cell Disease (Hgb SS)1. Sickle cell disease is caused when valine replaces glutamic acid at position 6

on both beta chains. It results in a decrease in hemoglobin solubility andfunction. Defect is inherited from both parents.

2. Occurs most commonly in African-American, African, Mediterranean, andMiddle East populations

3. No Hgb A is produced, and approximately 80% Hgb S and 20% Hgb F(the compensatory hemoglobin) are seen. Hgb A2 is variable.

4. Hemoglobin insolubility results when deoxyhemoglobin is formed.Hemoglobin crystallizes in erythrocytes. It is characterized by the classicsickled shape of erythrocytes.

5. Clinical findingsa. Erythrocytes become rigid and trapped in capillaries; blood flow restriction

causes lack of oxygen to the tissues, resulting in tissue necrosis.b. All organs are affected, with kidney failure being a common outcome;

hyposplenism and joint swelling also occur.c. Vaso-occlusive crisis occurs with increased bone marrow response to the

hemolytic anemia. Crisis can be initiated by many physiological factors,including surgery, trauma, pregnancy, high altitudes, etc.

d. Apparent immunity to Plasmodiumfalciparum6. Diagnosis is made after 6 months of age (time of beta-gamma globin chain

switch), with life expectancy of 50 years with proper treatment. Death usuallyresults from infection or congestive heart failure.

7. Laboratorya. Severe normochromic/normocytic hemolytic anemia with

polychromasia resulting from premature release of reticulocytes; bonemarrow erythroid hyperplasia (M:E ratio decreases)

b. Sickle cells, target cells, nucleated RBCs, Pappenheimer bodies, andHowell-Jolly bodies are seen. Increased bilirubin and decreasedhaptoglobin are characteristic due to hemolysis.

c. Positive hemoglobin solubility screening testd. Hgb S migrates with hemoglobins D and G on alkaline hemoglobin

electrophoresis; can differentiate using acid electrophoresis.

C. Sickle Cell Trait (Hgb AS)1. Sickle cell trait is caused when valine replaces glutamic acid at position 6 on

one beta chain. Defect is inherited from one parent. One normal beta chaincan produce some Hgb A.

2. Approximately 60% Hgb A and 40% Hgb S are produced, with normalamounts of Hgbs A2 and F.

Page 51: Hematology

HEMOGLOB1NOPATHIES • 279

3. This heterozygous trait is the most common hemoglobinopathy in theUnited States.

4. Sickle cell trait generally produces no clinical symptoms. Anemia is rare but,if present, will be normochromic/normocytic, and sickling can occur duringrare crisis states (same as in Hgb SS).

5. Positive hemoglobin solubility screening test6. Apparent immunity to Plasmodium falciparum

D. Hgb C Disease/Hgb CC1. Hgb C disease is caused when lysine replaces glutamic acid at position 6 on

both beta chains. Defect is inherited from both parents.2. Occurs in the African-American and African populations3. No Hgb A is produced; approximately 90% Hgb C, 2% Hgb A2, and 7%

Hgb F are produced. Mild anemia may be present.4. Laboratory: Normochromic/normocytic anemia with target cells;

characterized by intracellular rodlike C crystals5. Hgb C migrates with hemoglobins A2, E, and O on alkaline hemoglobin

electrophresis; can differentiate hemoglobins using acid electrophoresis.6. The heterozygous Hgb C trait patient is asymptomatic, with no anemia; the

one normal beta chain is able to produce approximately 60% Hgb A and 40%Hgb C, with normal amounts of Hgb A2 and Hgb F.

E. Hgb SC Disease1 . Hgb SC disease is a double heterozygous condition where an abnormal

sickle gene from one parent and an abnormal C gene from the other parent isinherited.

2. Seen in African, Mediterranean, and Middle Eastern populations; symptomsless severe than sickle cell anemia but more severe than Hgb C disease

3. No Hgb A is produced; approximately 50% Hgb S and 50% Hgb C areproduced. Compensatory Hgb F may be elevated up to 7%.

4. Laboratory: Moderate to severe normocytic/normochromic anemia withtarget cells; characterized by SC crystals; may see rare sickle cells or Ccrystals; positive hemoglobin solubility screening test

F. Other Hemoglobinopathies1. Hemoglobin E

a. Caused when lysine replaces glutamic acid at position 26 on the beta chainb. Found more commonly in Southeast Asian, African, and

African-American populationsc. Homozygous condition results in mild anemia with microcytes and target

cells; heterozygotes are asymptomatic.d. Hgb E migrates with hemoglobins A2, C, and O on alkaline hemoglobin

electrophoresis.

Page 52: Hematology

280 • CHAPTER 2: HEMATOLOGY

2. Hemoglobin Da. Caused when glycine replaces glutamic acid at position 121 on the beta

chainb. Found more commonly in Middle Eastern and Indian populationsc. Both homozygous and heterozygous conditions are asymptomatic.d. Hgb D migrates with Hgb S and Hgb G on alkaline hemoglobin

electrophoresis.

XI. THALASSEMIAS

A. Introduction: Group of inherited disorders causing decreased rate of synthesisof a structurally normal globin chain (quantitative defect); characterized bymicrocytic/hypochromic RBCs and target cells

1. Classified according to the globin chain affected2. Found in Mediterranean (beta), Asian (alpha), and African (alpha and beta)

populations3. Severity varies from no clinical abnormalities to transfusion-dependent to fatal4. Thalassemia major: Severe anemia; either no alpha or no beta chains produced5. Thalassemia minor/trait: Mild anemia; sufficient alpha and beta chains

produced to make normal hemoglobins A, A2, and F, but may be in abnormalamounts

B. Beta-Thalassemia1. Major/homozygous (Cooley anemia)

a. Markedly decreased rate of synthesis or absence of both beta chains resultsin an excess of alpha chains; no Hgb A can be produced; compensate withup to 90% Hgb F.

b. Excess alpha chains precipitate on the RBC membrane, form Heinzbodies, and cause rigidity; destroyed in the bone marrow or removed bythe spleen

c. Symptomatic by 6 months of age; hepatosplenomegaly, stunted growth,jaundice; prominent facial bones, especially the cheek and jaw; iron overloadfrom RBC destruction and multiple transfusions cause organ failure

d. Laboratory: Severe microcytic/hypochromic anemia, target cells,teardrops, many nRBCs, basophilic stippling, Howell-Jolly bodies,Pappenheimer bodies, Heinz bodies; increased serum iron and increasedbilirubin reflect the hemolysis

2. Minor/heterozygousa. Decreased rate of synthesis of one of the beta chains; other beta chain

normalb. Laboratory: Mild microcytic/hypochromic anemia, with a normal or

slightly elevated RBC count; target cells, basophilic stipplingc. Hgb A is slightly decreased, but Hgb A2 is slightly increased to compensate

Page 53: Hematology

HEMATOLOGYTESTS • Z81

C. Alpha-Thalassemia1 . Major (hydrops fetalis)

a. All four alpha genes are deleted; no normal hemoglobins are produced.b. 80% hemoglobin Bart's (74) produced; cannot carry oxygen; incompatible

with life; die in utero or shortly after birth2. HgbH disease

a. Three alpha genes are deleted. Decrease in alpha chains leads to betachain excess.

b. Hemoglobin H ($4), an unstable hemoglobin, is produced. Heinz bodiesform and rigid RBCs are destroyed in the spleen. Distinguishing character-istics include: moderate microcytic/hypochromic anemia; up to 30%Hgb H; the rest is Hgb A.

3. Minor/traita. Two alpha genes are deleted. Patients are usually asymptomatic and

discovered accidentally. Up to 6% Hgb Bart's in newborns may be helpfulin diagnosis; absent by 3 months of age

b. Mild microcytic/hypochromic anemia often with a high RBC count andtarget cells

4. Silent carriera. One alpha gene is deleted. Patients are asymptomatic and are often not

diagnosed unless gene analysis is done.b. Borderline low MCV may be the only sign.

D. Thalassemia/Hemoglobinopathy Interactions

1. Caused by the inheritance of a thalassemia gene from one parent and ahemoglobin variant gene from the other parent

2. Severity and symptoms depend on the specific interactions.3. Common interactions include Hgb S/beta-thalassemia, Hgb C/beta-

thalassemia, and Hgb E/beta-thalassemia.

XII. HEMATOLOGYTESTS

A. Blood Cell Enumeration—Manual Methods

1. Manual WBC count using a hemacytometera. Dilute a well-mixed, EDTA, whole blood sample 1:20 with 3% glacial

acetic acid; allow 10 minutes for complete RBC lysis; fill both sides ofhemacytometer; allow 1-2 minutes for settling.

b. Use bright field or phase microscopy, count WBCs seen in the four 1-mm2

corner squares on both sides of the hemacytometer, use the 10X objective.Total area counted is 8 mm2.

c. Formula

3 _ Total WBCs counted X Dilution (20)WBC/mmJ -

Total area counted (mm ) X Depth (0.1)

Page 54: Hematology

282 • CHAPTER 2: HEMATOLOGY

d. Alternate dilution factor and area counted can be used; appropriateadjustments must be made to the formula. Other diluents (1% ammoniumoxalate) can also be used.

e. Correction for presence of nucleated RBCs

, 100 X Uncorrected WBCsCorrected WBC/mm = 10Q + # nRBCs per 100 WBCs

2. Platelet counta. Dilute a well-mixed, EDTA, whole blood sample 1 : 100 with 1%

ammonium oxalate; allow 10 minutes for complete RBC lysis; fill bothsides of hemacytometer; allow 10 minutes for complete settling in ahumidified chamber to prevent evaporation.

b. Use phase (preferred) or bright field microscopy, count platelets seenin the center 1-mm2 square on both sides of the hemacytometer, use the40X objective. Total area counted is 2 mm2.

c. Formula

PLT count/mm3 =3 _ Total PLTs counted X Dilution (100)

Total area counted ( m m ) X Depth (0.1)

3. Sources of error involving manual cell countsa. Specimen clottedb. Sample inadequately mixed before dilutingc. Equipment not thoroughly cleaned or driedd. Technical errors due to evaporation on the hemacytometer, diluting/plating,

following procedure, counting of cells, calculating results

B. Blood Cell Enumeration—Automated Methods

1 . Electrical impedancea. Cells pass through an aperture with an electrical current flowing through

simultaneously. Cells do not conduct current but rather they changeelectrical resistance, which is then counted as voltage pulses.

b. The number of pulses generated is proportional to the number of cellspresent; amplitude of the pulse generated is proportional to the size ofthe cell.

c. Sample is diluted in isotonic conductive solution that preserves cell shapeand characteristics.1) Dilutions used are dependent on instrument/methodology used.2) Platelets are counted simultaneously with RBCs.3) Sample for counting WBCs is mixed with reagent to lyse RBCs.

A commercially available reagent, which both lyses RBCs andconverts hemoglobin to cyanmethemoglobin, can be used to determinehemoglobin and WBCs in one dilution.

d. Thresholds are used to separate cell populations and subpopulations.

Page 55: Hematology

HEMATOLOGY TESTS • 283

e. Hydrodynamic focusing is utilized to reduce cell coincidence (chance ofone cell being counted more than once).

2. Light scattering optical methoda. Uses a flow cytometer with laser to measure light scattering properties of

cells1) Forward angle light scatter measures cell size.2) Side angle light scatter provides information on cell granularity and

lobularity.3) Number of pulses generated is proportional to the number of cells present.

b. Dilutions used are dependent on instrument/methodology used.3. Interpretative reports give relative percentages and absolute counts for the 5

leukocyte subpopulations (most instruments).4. Suspect "flags" indicate problems: Exceeding linearity, lack of agreement

among apertures, unacceptable distribution caused by unusual cell populations.5. Automated cell count errors

a. WBC counts exceeding instrument linearity limits result in increased cellturbidity and may falsely increase the hemoglobin, MCH, and MCHC.

b. Glucose over 600 mg/dL (hyperosmolarity) may increase the MCV andhematocrit and decrease the MCHC.

c. Cold agglutinins increase the MCV, MCH, and MCHC and decrease theRBC count and hematocrit.

d. Lipemia increases the hemoglobin, MCH, and MCHC.e. Repeat the analysis if:

1) Rule of three (shown below) failure on a normocytic sample(especially MCHC >37 g/dL)a) RBC X 3 = Hgbb) RBC X 9 = Hctc) Hgb X 3 = Hct

2) Any result outside linearity limits established by manufacturer (diluteinto linearity range)

3) Unexplained delta check failures (e.g., results do not correlate withrecent previous results, especially MCV)

C. Histograms and Scatterplots1. A histogram utilizes impedance technology, and it is a representation of cell

number versus one measured property, usually cell size. It is used for WBCs,RBCs, and platelets.a. WBC histogram

1) 35^-50 fL is the reference size range for WBCs.2) 1st peak: 35-90 fL is the range for lymphocytes.3) 2nd peak: 90-160 fL is the range for mononuclear cells (monocytes,

reactive lymphocytes, and immature WBCs).4) 3rd peak: 160^-50 fL is the range for granulocytes.

Page 56: Hematology

284 • CHAPTER 2: HEMATOLOGY

b. Abnormal WBC histogram1) Population before 35 fL may indicate nucleated RBCs (nRBCs), giant

or clumped platelets.2) Peak overlap at 90 fL may indicate reactive lymphocytes or blast cells.3) Peak overlap at 160 fL may indicate an increase in bands, immature

neutrophils, eosinophils, or basophils.4) Population after 450 fL may indicate a high granulocyte count.

c. RBC histogram1) 36 fL and above is the reference size range for RBCs.2) A normal RBC histogram will show a single peak between 70 and

110 fL that will correlate with the MCV.d. Abnormal RBC histogram

1) Two peaks indicate a dimorphic erythrocyte population.2) Increased curve width will correlate with an increased RDW

(anisocytosis).3) Shift to the right indicates an increased MCV (macrocytic).4) Shift to the left indicates a decreased MCV (microcytic).

e. Platelet histogram1) 2-20 fL is the reference size range for platelets.2) Lower region interference (<2 fL) indicates electrical interference;

upper region interference (>20 fL) indicates microcytic RBCs orschistocytes, giant or clumped platelets.

2. A scatterplot/scattergram is a two-dimensional representation of two ormore cell properties or characteristics plotted against each other (e.g., sizeversus granularity or lobularity). Scatterplots of WBCs are displayed on amonitor and are color coded for different subpopulations.a. Methodologies include radio frequency, fluorescence, and cytochemistry.b. Correlation between abnormal cell populations and suspect flags is

generally very good.

D. Hemoglobin Measurement

1. Blood oxygen capacity: Measures functional hemoglobin

Oxygen capacity in mL/dL blood— = grams of Hgb/dL blood

1.34 (Hgb oxygen capacity)

2. Cyanmethemoglobin method is the reference method; it will measure allhemoglobins except for sulfhemoglobin.a. Uses Drabkin reagent (potassium ferricyanide and KCN) to lyse RBCs

and convert heme iron to the feme state (Fe3+), forming methemoglobin.KCN in the reagent converts methemoglobin to Cyanmethemoglobin; readspectrophotometrically at 540 nm.

b. Automated cell counters use some modification of theCyanmethemoglobin method to determine hemoglobin concentration.

Page 57: Hematology

HEMATOLOGY TESTS • 285

E. Reticulocyte Counts

1 . Supravital new methylene blue stain is used to demonstrate reticulum inreticulocytes.

2. Reticulocyte (retic) formulas:a. Relative count

# of ReticsRetics (%) = X 100

1000 RBCs observed

b. Absolute count

Absolute retic (X 109/L) = Retic % X RBC count (X 1012/L)/100

3. Corrected reticulocyte counts are calculated to account for the degree ofanemia by using a standard normal hematocrit of 45% expressed in SI units.

Hct (L/L)Corrected retic count = Retic % X —

0.45 L/L

4. Immature reticulocyte fraction (IRF) is an instrument calculated parameterthat indicates the ratio of immature reticulocytes to total reticulocytes.

F. Erythrocyte Sedimentation Rate: ESR measures degree of settling of RBCsin plasma in an anticoagulated specimen during a specific time, usually 1 hour.High fibrinogen or protein levels increase the ESR.

1 . Reference range: Approximately 0-20 mm/hr; age and sex dependent2. ESR is increased in chronic inflammatory conditions, including rheumatoid

arthritis and pregnancy (increased fibrinogen), bacterial infection, malignancy,tissue damage, multiple myeloma, Waldenstrom macroglobulinemia, andsevere anemia.a. Sources of error causing falsely increased results: Tilted column,

hemolysis, increased room temperature3. ESR is normal to decreased in polycythemia, sickle cell anemia, spherocytosis,

and other conditions with poikilocytosis (prevents rouleaux formation).a. Sources of error causing falsely decreased results: Clotted sample,

excess anticoagulant, "old" blood (spherocytes form)

G. Hemoglobin F (Kleihauer-Betke method): Count dense-staining Hgb Fcells and the number of ghost cells containing Hgb A to obtain percentage.

1. It is used to detect the presence of fetal cells in the maternal circulation duringproblem pregnancies because Hgb F in fetal cells resists acid elution.

2. It differentiates hereditary persistence of fetal hemoglobin from otherconditions associated with high Hgb F levels.

3. Normal newborns have 70-90% Hgb F levels.

Page 58: Hematology

286 • CHAPTER 2: HEMATOLOGY

H. Solubility Test for Hemoglobin S (Sickle Cell Prep)1. Hemoglobin S is insoluble when combined with a reducing agent (sodium

dithionite).2. Hgb S will crystallize and give a turbid appearance to the solution.3. The test will not differentiate homozygous from heterozygous conditions

containing Hgb S.4. Follow up a positive solubility test with hemoglobin electrophoresis.

I. Hemoglobin Electrophoresis1. Procedure for the identification of normal and abnormal hemoglobins2. Methodology is based on net negative charges, which cause hemoglobins to

migrate from the negative (cathode) region toward the positive (anode)region. The distance a particular hemoglobin molecule migrates is due to itsnet electrical charge.

3. Two types of electrophoresis: Cellulose acetate at pH 8.6 and citrate agarat pH 6.2

4. Migration of hemoglobin is dependent on net negative charge and buffer pH.5. Cellulose Acetate (pH 8.6) Hemoglobin Electrophoresis

Cathode (-) Anode (+)

(x) origin A2 S F A

C D

E G

a. At pH 8.6, Hgb A migrates the fastest, and Hgb A2, C, E, and O migratethe slowest.

6. Citrate Agar (pH 6.2) Hemoglobin Electrophoresis

Anode (+) Cathode (-)

C S (x) origin A F

A2

D

G

E

O

Page 59: Hematology

HEMATOLOGYTESTS • 287

a. At pH 6.2, Hgb S is differentiated from Hgb D and G.b. At pH 6.2, Hgb C is differentiated from Hgb A2, E, and O.

J. Flow Cytometry

1. Principle: Cells in a suspension of buffered solution are labeled with one toseveral fluorescent compounds. This cell suspension is run under highpressure and in a single, narrow stream through a laser, causing excitation ofthe fluorescent compound(s) and resulting in the emission of light energy. Thisenergy is detected by a photomultiplier tube and is subsequently convertedinto computerized data, which upon analysis provides information regardingnumber, size, and cellular composition of the population assayed.

2. Major components of a flow cytometera. Fluidics—Flow chamber for single cell separation, sheath fluid, and

hydrodynamic focusingb. Optics—Excitation light sources include lasers (argon, krypton, helium-

neon, helium-cadmium, diode) or lamps (mercury, xenon-mercury). Lightis separated by dichroic mirrors and filters.

c. Electronics—Photomultiplier tube detects light energy, then coverts thisto voltage pulses; computers translate pulses into data files.

3. Hydrodynamic focusing uses laminar flow to line the cells up single file.4. Light is scattered at 90 degrees or forward.5. Fluorescent dyes used in flow cytometry include, but are not limited to,

allophycocyanin (APC), acridine orange (AO), chromomycin A3, cyanine dye(Cy), fluorescein isothiocyanate (FITC), peridinin chlorophyll protein(PerCP), phycoerythrin (PE), propidium iodine (PI), pyronin Y, rhodamineisothiocyanate, and sulforhodamine 101 acid chloride.

6. Specimens analyzed by flow cytometry: Leukocytes, erythrocytes, lymphnodes, peripheral whole blood, bone marrow, tumors, and other tissues

7. Clinical applications: Differentiation of T and B cells; cell cycle analysis;diagnosing and following patients with leukemia, lymphoma, andautoimmune or deficiency diseases; karyotyping; and monitoring a patient'sresponse to drug therapy

Page 60: Hematology

^reviewquestions

IN o -L JLV LJ \~s JL JLUJN1 O Each of the questions or incomplete statements that follow is

comprised of four suggested responses. Select the best answer or completion statement in each case.

Hematopoiesis1. What is the first type of cell produced by

the developing embryo?A. ErythrocyteB. GranulocyteC. LymphocyteD. Thrombocyte

2. What percentage of tissue located in thebone marrow cavities of adults is fat?A. 10%B. 25%C. 50%D. 75%

3. Which of the following is not characteris-tic of pluripotent hematopoietic stemcells?A. Possess self-renewal abilityB. Produce progenitor cells committed to

a single cell lineageC. Express the stem cell marker CD 13D. Are morphologically unrecognizable

4. In an adult, what are the two best areas forobtaining active bone marrow byaspiration?A. Vertebra, tibiaB. Sternum, vertebraC. Anterior iliac crest, tibiaD. Posterior iliac crest, sternum

5. What is the normal ratio of myeloid toerythroid precursors in bone marrow (M:Eratio)?A. 1:1B. 1:3C. 4:1D. 8:1

6. Which of the following does not accuratelydescribe hematopoietic growth factors?A. Bind to target cell receptors to express

activityB. Action of majority is lineage restrictedC. May promote or suppress cell deathD. Can stimulate or inhibit cell

proliferation

288

Page 61: Hematology

REVIEW QUESTIONS • 289

7. In the third month of gestation, what is theprimary site of hematopoiesis?A. LiverB. Marrow of long bonesC. SpleenD. Yolk sac

8. The mechanism that relays informationabout tissue oxygen levels to erythro-poietin-producing sites is located in theA. BrainB. KidneyC. LiverD. Spleen

9. Antigen-independent lymphopoiesisoccurs in primary lymphoid tissue locatedin theA. Liver and kidneyB. Spleen and lymph nodesC. Peyer's patches and spleenD. Thymus and bone marrow

10. Programmed cell death is calledA. NecrosisB. ApoptosisC. Cellular senescenceD. Terminal differentiation

11. In what area of the bone marrow doeshematopoiesis take place?A. CordsB. EndosteumC. EndotheliumD. Sinuses

12. Bone marrow cellularity refers to theratio ofA. Red cell precursors to white cell

precursorsB. Hematopoietic tissue to adipose tissueC. Granulocytic cells to erythrocytic cellsD. Extravascular tissue to intravascular

tissue

13. Interleukins and colony stimulatingfactors are cytokines produced byA. B lymphocytes and erythrocytesB. Erythrocytes and thrombocytesC. Monocytes and T lymphocytesD. Neutrophils and monocytes

14. What is the approximate total bloodvolume in an adult?A. 1LB. 2LC. 6LD. 12L

15. The myeloid progenitor cell can producecells committed toA. Granulocytic, erythrocytic, monocytic,

or megakaryocytic lineagesB. Granulocytic, monocytic, lympho-

cytic, or megakaryocytic lineagesC. Erythrocytic, granulocytic, monocytic,

or lymphocytic lineagesD. Erythrocytic, granulocytic, lympho-

cytic, or megakaryocytic lineages

16. The largest hematopoietic cells in normalbone marrow areA. OsteoblastsB. OsteoclastsC. MegakaryocytesD. Plasma cells

17. When evaluating a bone marrow aspiratesmear, which finding is consideredabnormal?A. A predominance of granulocyte

precursors as compared to nucleatedred cells

B. Detection of stainable iron inmacrophages and erythroid precursorswith Prussian blue

C. An average of three megakaryocytesseen per low power (10X) field

D. The presence of 10% myeloblasts onthe cell differential count

Page 62: Hematology

290 • CHAPTER 2: HEMATOLOGY

18. As most blood cell lines mature, which ofthe following is characteristic?A. Cell diameter increasesB. Nucleus to cytoplasm ratio (N:C)

decreasesC. Nuclear chromatin becomes less

condensedD. Basophilia of the cytoplasm increases

19. Which of the following describesthrombopoietin (TPO)?A. Renal hormone that regulates marrow

red cell productionB. Marrow hormone secreted by

developing megakaryoblastsC. Hormone produced by the liver that

stimulates megakaryopoiesisD. Pituitary hormone that controls

platelet sequestration by the spleen

20. When the hepatic phase of fetal life isreactivated in an adult, hematopoiesis canbe termedA. Myeloid or medullaryB. Myeloid metaplasia or extramedullaryC. Myelophthisis or myelodysplasiaD. Mesoblastic or mesenchymal

Erythrocytes21. What is the average life span of a normal

red blood cell?A. 1 dayB. 10 daysC. 60 daysD. 120 days

22. The Na+-K+ cation pump is an importantmechanism in keeping the red blood cellintact. Its function is to maintain a highlevel ofA. Intracellular Na+

B. Intracellular K+

C. Plasma Na+

D. Plasma K+

23. Which of the following depicts thestructure of the hemoglobin molecule?A. Two heme groups, two globin chainsB. Four heme groups, two globin chainsC. Two heme groups, four globin chainsD. Four heme groups, four globin chains

24. Which of the following describes theprocess known as culling?A. Release of red cells from the bone

marrowB. Binding of free hemoglobin by

transport proteinsC. Incorporation of iron into proto-

porphyrin IXD. Removal of abnormal red cells by the

spleen

25. Hemoglobin forms that are incapable ofoxygen transport includeA. Deoxyhemoglobin and oxyhemoglobinB. Oxyhemoglobin and

carboxyhemoglobinC. Carboxyhemoglobin and

methemoglobinD. Methemoglobin and deoxyhemoglobin

26. The majority of iron found in an adult is aconstituent ofA. FerritinB. MyoglobinC. HemoglobinD. Peroxidase

27. A senescent red blood cell is one that hasA. Been hemolyzedB. Lived its life spanC. Become deformedD. Lost its mitochondria

28. What red cell morphologic abnormality isdescribed by the term "poikilocytosis"?A. Variations in sizeB. Deviations from normal shapeC. Presence of inclusionsD. Alterations in hemoglobin concentration

Page 63: Hematology

REVIEW QUESTIONS • 291

29. Howell-Jolly bodies are composed ofA. DNAB. IronC. ReticuluraD. RNA

30. When spherocytes are reported, what isobserved on the peripheral blood smear?A. Red cells without a central pallorB. Red cells with blunt projectionsC. Red cells with sharp projectionsD. Red cells with intracellular rod-shaped

crystals

31. The red cells found in lead poisoningcharacteristically exhibit coarse granulescomposed of that arereported as .A. Precipitated hemoglobin; Pappen-

heimer bodiesB. Aggregated ribosomes; basophilic

stipplingC. Nuclear fragments; Pappenheimer

bodiesD. Excess iron deposits; basophilic

stippling

32. Rouleaux of red blood cells when seenin the monolayer of a blood smear ischaracteristic ofA. HypersplenismB. HypogammaglobulinemiaC. Cold hemagglutinin diseaseD. Multiple myeloma

33. Which of the following is most frequentlyassociated with the inclusion bodies seenin Color Plate !•?A. Iron overload stateB. Post-transfusionC. Post-splenectomyD. Iron-deficient state

34. Which of the following statements aboutiron absorption is true?A. Absorption occurs in the ileum.B. The mucosal cell always absorbs the

correct amount of iron to meet needs.C. Absorption increases when erythro-

poietic activity increases.D. Alkaline pH favors absoiption.

35. What term describes a mature red bloodcell that contains iron granules or deposits?A. SiderosomeB. SideroblastC. Ringed sideroblastD. Siderocyte

36. Which of the following is associated witha "shift to the left" in the oxygen dissocia-tion curve of hemoglobin?A. Decreased pH and elevated temperatureB. Decreased oxygen affinityC. Decreased oxygen releaseD. Presence of 2,3-bisphosphoglycerate

(2,3-BPG)

37. Which of the following statements doesnot characterize erythropoietin (EPO)?A. Transforms the CFU-E into the earliest

recognizable RBC precursorB. Increases the rate of red blood cell

production by the bone marrowC. Shortens the maturation time of

developing erythroid precursorsD. Decreases stimulation of erythro-

poiesis when cellular hypoxiaincreases

38. Which of the following factors will resultin an immediate increase in oxygendelivery to the tissues?A. Increased pHB. High altitudesC. Increased hemoglobin binding of

2,3-BPGD. Increased renal release of erythropoietin

Page 64: Hematology

292 • CHAPTER 2: HEMATOLOGY

39. Periods of intense erythropoietin activitycause premature release of marrowreticulocytes into the blood. Which of thefollowing is not true of these earlyreticulocytes?A. Loss of residual RNA occurs immedi-

ately upon marrow releaseB. Circulate longer than usual before

reaching maturityC. May be termed shift or stress

reticulocytesD. Show diffuse basophilia with Wright's

stain

40. Which of the following inclusions is onlyvisible with supravital staining?A. Basophilic stipplingB. Cabot ringsC. Heinz bodiesD. Pappenheimer bodies

41. The presence of schistocytes on theperipheral blood smear is commonlyassociated withA. Increased iron mobilizationB. Increased red cell destructionC. Decreased erythropoietin activityD. Decreased red cell proliferation

42. Which of the following may be a sign ofaccelerated bone marrow erythropoiesis?A. Hypercellular marrow with a

decreased number of RBC precursorsB. Bone marrow M:E ratio of 6:1C. Nucleated red cells in the peripheral

circulationD. Low erythrocyte, hemoglobin, and

hematocrit levels

43. Microcytic, hypochromic red cells aremost often associated with impairedA. DNA synthesisB. RNA metabolismC. Hemoglobin synthesisD. Enzyme metabolism

44. When in bone marrow, the nucleated redcells present in Color Plate 21 would bestaged asA. Basophilic normoblastsB. Polychromatophilic normoblastsC. Orthochromic normoblastsD. Pronormoblasts

45. When acanthocytes are found on the bloodsmear, it is usually the result ofA. Abnormal membrane permeabilityB. Altered membrane lipidsC. Mechanical traumaD. Polymerization of hemoglobin

molecules

46. Which erythrocyte metabolic pathwaygenerates adenosine triphosphate (ATP)via glycolysis?A. Embden-MeyerhofB. Hexose monophosphateC. Rapoport-LueberingD. Methemoglobin reductase

47. Which of the following red blood cellprecursors is the last stage to undergomitosis?A. PronormoblastB. Basophilic normoblastC. Polychromatophilic normoblastD. Orthochromic normoblast

48. The major adult hemoglobin requires thesynthesis of alpha-globin chains andA. Beta-globin chainsB. Delta-globin chainsC. Epsilon-globin chainsD. Gamma-globin chains

49. Defective nuclear maturation commonlyresults in the production of red cellsthat areA. NormocyticB. HypochromicC. MacrocyticD. Microcytic

Page 65: Hematology

REVIEW QUESTIONS • 293

50. The major storage form of iron isA. FerritinB. TransferrinC. HemosiderinD. Hemachromatin

51. The red cells observed on a peripheralblood smear show extreme anisocytosiswith an equal number of macrocytes andmicrocytes. Which of the following valuescorrelate with this finding?A. MCV 108.0 fL, RDW 14.0%B. MCV 90.0 fL, RDW 25.0%C. MCV 75.0 fL, RDW 16.0%D. MCV 88.0 fL, RDW 12.0%

52. Excessive extravascular red cell destruc-tion is associated withA. HemoglobinemiaB. BilirubinemiaC. HemoglobinuriaD. Hemosiderinuria

53. Which protein is primarily responsible fortransport of hemoglobin dimers resultingfrom intravascular hemolysis?A. HemopexinB. AlbuminC. HemosiderinD. Haptoglobin

54. The morphologic abnormality characteris-tically found in hemoglobinopathies isA. ElliptocytesB. DacryocytesC. CodocytesD. Discocytes

55. Where do the early and late stages ofheme synthesis occur?A. On ribosomesB. In mitochondriaC. In cytoplasmD. In nucleoli

56. Spectrin is a protein that occupies a majorrole inA. Red cell membrane structureB. Reducing ferric ironC. Red cell transport and removal of CC>2D. Iron recovery during hemoglobin

degradation

57. What is the function of reduced glu-tathione (GSH) in the red blood cell?A. Promotes Kreb's cycle activityB. Maintains anion balance during the

"chloride shift"C. Neutralizes intracellular oxidants that

accumulateD. Prevents oxygen uptake by hemoglobin

58. What does measuring the total iron-binding capacity (TIBC) represent?A. Amount of free iron in serumB. Circulating protein-bound ironC. Amount of iron that transferrin can bindD. Indirect measurement of iron stores

59. Serum ferritin is a good indicator of theamount ofA. Cytochrome ironB. Storage ironC. Hemoglobin ironD. Transferrin saturation

60. Fetal hemoglobin differs from adulthemoglobin in that hemoglobin FA. Has a lower oxygen affinityB. Resists elution from red cells with acid

solutionsC. Is no longer synthesized after birth in a

normal individualD. Has four gamma-globin chains

Erythrocyte Disorders

61. Impaired DNA metabolism is characteristicofA. Hemoglobin C diseaseB. Iron-deficiency anemiaC. Sideroblastic anemiaD. Megaloblastic anemia

Page 66: Hematology

294 • CHAPTER 2: HEMATOLOGY

62. Which of the following is associated withglucose-6-phosphate dehydrogenase(G6PD) deficiency?A. G6PD gene is located on the

X chromosome.B. Ongoing intravascular hemolysis

occurs.C. All circulating red cells, including

reticulocytes, lack enzyme activity.D. Splenectomy can relieve the rate of red

cell destruction.

63. In regard to variant hemoglobin E, «2

|32 26 Glu -* L^s, which of the followingstatements is false?A. There are two normal alpha chains.B. Glutamic acid replaces lysine on

position 26 of the beta chains.C. Hemoglobin E is the second most

common hemoglobin variant known.D. Glutamic acid is normally found at

position 26 of the beta chain.

64. Color Plate 3B shows the peripheral bloodof a 16-year-old female with a sporadichistory of dizzy spells, fainting, andjaundice. This patient also had a historyof periodic abdominal pain related togallstones. Upon physical examination,she exhibited mild splenomegaly. Herhemoglobin was 107 g/L (10.7 g/dL),hematocrit was 0.32 L/L (32%), red cellindices were normal, and the directantiglobulin test was negative. Based onhistory and peripheral blood morphology,which of the following statements is mostlikely true ?A. Hemoglobin S will be revealed by

electrophoresis.B. Tests to confirm iron deficiency should

be ordered.C. An intrinsic hereditary defect of red

cells should be suspected.D. The anemia is secondary to spleen and

gallbladder disorders.

65. A 9-month-old male was seen in theEmergency Department with a femurfracture that had occurred from a fall downthe stairs. Upon physical examination, thephysician noted hepatosplenomegaly,extreme pallor, and a slight arrhythmia.A complete blood count revealed thefollowing:

WBC

RBC

Hemoglobin

Hematocrit

MCV

MCH

MCHC

ROW

1 2.2 X 1 09/L (1 2.2 X 1 03/(JiL)

3.05 X 1 01 2/L (3.05 X 1 06/(jiL)

61 g/L (6.1 g/dL)

0.20 L/L (20%)

65.5 fL

20 pg

305 g/L (30.5 g/dL)

25%

The Wright's stained blood smear showedthe findings seen in Color Plate 4B.Hemoglobin electrophoresis wasordered with results as follows:

HgbA

HgbA2

HgbF

0%

3%

97%

Which condition is most likely causing thehematologic abnormalities?A. Alpha-thalassemia majorB. Cooley beta-thalas semia maj orC. Hemoglobin H diseaseD. Hereditary persistence of

hemoglobin F

Page 67: Hematology

REVIEW QUESTIONS • 295

66. A 14-year-old African-American malewas seen in the clinic for abdominal pain.A complete blood count revealed thefollowing:

WBC

RBC

Hemoglobin

Hematocrit

MCV

MCH

MCHC

ROW

7.0 X 1 09/L (7.0 X 1 03/|JLL)

2.90 X 1 01 2/L (2.90 X 1 06/(JiL)

85 g/L (8.5 g/dL)

0.25 L/L (25%)

86.2 f L

29.3 pg

340 g/L (34.0 g/dL)

21%

The peripheral smear showed the redblood cell morphology seen in Color Plate5B. What condition is suggested by thesefindings?A. Hemoglobin E diseaseB. Hemoglobin S diseaseC. Hemoglobin SC diseaseD. Hemoglobin C disease

67. Pica is most commonly associated withwhich of the following conditions?A. Pyridoxine deficiencyB. Lack of erythrocyte folateC. Iron deficiencyD. Porphyrias

68. Of the following, the leading cause offolate deficiency isA. Increased requirementsB. Dietary insufficiencyC. Drug inhibitionD. Malabsorption

69. Which of the following statements aboutsickle cell syndromes is false?A. Asplenism may result from repeated

sickling crises in the homozygousstate.

B. Heterozygous persons may be partlyprotected from infection by falciparummalaria.

C. Hemoglobin S is more soluble indithionite than is normal hemoglobin.

D. Trait conditions are generallyasymptomatic with no sickle cellformation.

70. The findings seen in Color Plate 6B can befound in patients with microangiopathichemolytic anemia (MAHA). Which of thefollowing conditions could not beresponsible for this type of red celldestruction?A. Disseminated intravascular

coagulation (DIG)B. Hemolytic uremic syndrome (HUS)C. Thrombotic thrombocytopenic

purpura (TTP)D. Idiopathic thrombocytopenic purpura

(ITP)

71. Which of the following blood findingsdoes not correlate with the presence ofringed sideroblasts in the bone marrow?A. Pappenheimer bodiesB. Basophilic stipplingC. Increased total iron-binding capacityD. Increased percent transferrin

saturation

72. Which of the following conditions is notusually associated with marked reticulo-cytosis?A. Four days after a major hemorrhageB. Drug-induced autoimmune hemolytic

anemiaC. Sickle cell anemiaD. Pernicious anemia

Page 68: Hematology

296 • CHAPTER 2: HEMATOLOGY

73. Hereditary stomatocytosis is manifestedphysiologically by changes inA. Hemoglobin oxygen affinityB. Membrane cation permeabilityC. Efficiency of hemoglobin reductionD. Glycolytic ATP production

74. In addition to an increase in red bloodcells, which of the following is character-istic of polycythemia vera?A. Decreased platelets, decreased

granulocytes, decreased erythropoietinlevel

B. Decreased platelets, decreasedgranulocytes, increased erythropoietinlevel

C. Increased platelets, increasedgranulocytes, increased erythropoietinlevel

D. Increased platelets, increasedgranulocytes, decreased erythropoietinlevel

75. Which of the following is not characteris-tic of aplastic anemia?A. Extramedullary hematopoiesisB. Bone marrow hypoplasiaC. Absolute reticulocytopeniaD. Blood findings of pancytopenia

76. What values would you expect to obtainon hemoglobin and hematocrit determina-tions done immediately after a majorhemorrhage, if hemoglobin andhematocrit values were normal priorto the hemorrhage?A. Both normalB. Both decreasedC. Hemoglobin decreased, hematocrit

normalD. Hemoglobin normal, hematocrit

decreased

77. Results from a 1-day-old infant include ahemoglobin of 201 g/L (20.1 g/dL),hematocrit of 0.60 L/L (60.0%), MCV of110.2 fL, and 4 nucleated red cells/100WBCs. How should these results beinterpreted?A. The elevated hemoglobin and

hematocrit values indicate possibledehydration.

B. The nucleated red cells suggestaccelerated erythropoiesis due to ahemolytic process.

C. Testing should be done to identify thecause of the macrocytosis.

D. No further testing is indicated.

78. When viewing Color Plate 1m, the redblood cells with a single elongatedprojection are known asand may be seen inA. Acanthocytes; liver diseaseB. Echinocytes; liver diseaseC. Drepanocytes; myelofibrosisD. Dacryocytes; myelofibrosis

79. A patient with normocytic, normochromicanemia secondary to small cell carcinomamay be exhibiting an anemia designated asA. HemolyticB. MegaloblasticC. MyelophthisicD. Sideroblastic

80. Idiopathic aplastic anemia is best definedas a form of anemia thatA. Has no identifiable causeB. Is caused by a physician's treatmentC. Follows exposure to ionizing radiationD. Develops after a viral infection

81. Which of the following is a true red bloodcell aplasia?A. Marrow replacement anemiaB. Fanconi anemiaC. Diamond-Blackfan anemiaD. Donath-Landsteiner anemia

Page 69: Hematology

REVIEW QUESTIONS • 297

82. Which of the following is not a cause ofabsolute secondary erythrocytosis?A. Defective cardiac or pulmonary

functionB. High-altitude adjustmentC. Dehydration secondary to diuretic useD. Hemoglobins with increased oxygen

affinity

83. A cellulose acetate hemoglobin elec-trophoresis (alkaline pH), performed onthe blood of a stillborn infant, revealed asingle band that migrated farther towardthe anode than did the Hb A control. Whatis the most likely composition of thestillborn infant's hemoglobin?A. Four beta chainsB. Four gamma chainsC. Two alpha and two beta chainsD. Two alpha and two gamma chains

84. The most likely cause of the stillborninfant's condition in question 83 isA. Erythroblastosis fetalisB. Rh hemolytic disease of the fetusC. Hydrops fetalisD. ABO hemolytic disease of the

newborn

85. Which of the following conditions showsimilar CBC and blood smear findings?A. Beta-thalassemia major and minorB. Folic acid and vitamin B12 deficienciesC. Acute and chronic blood lossD. Sickle cell disease and trait

86. Which of the following would be useful inidentifying the cause of the blood profileseen in Color Plate 8«?A. Osmotic fragility testB. Reticulocyte countC. Direct antiglobulin testD. Urine urobilinogen level

87. Which of the following conditions is notassociated with the presence of schisto-cytes and spherocytes?A. Clostridial septicemiaB. Prosthetic heart valvesC. Severe thermal burnsD. Aplastic anemia

88. A 30-year-old woman who has beenvomiting for 3 days has a hemoglobinvalue of 180 g/L (18.0 g/dL) and ahematocrit of 0.54 L/L (54.0%). Herresults suggest the presence ofA. Absolute erythrocytosisB. Primary polycythemiaC. Secondary polycythemiaD. Relative polycythemia

89. An excessive accumulation of iron in bodytissues is calledA. HemochromatosisB. ErythroblastosisC. MegaloblastosisD. Acrocyanosis

90. Abetalipoproteinemia is characterizedby mild anemia and numerous

on the peripheralblood smear.A. AcanthocytesB. ElliptocytesC. EchinocytesD. Stomatocytes

91. What is the most common cause of irondeficiency?A. BleedingB. GastrectomyC. Inadequate dietD. Intestinal malabsorption

Page 70: Hematology

298 • CHAPTER 2: HEMATOLOGY

92. Which of the following does not charac-terize beta-thalassemia major?A. Transfusion-dependent anemiaB. Decreased alpha chains result in

excess beta chains.C. Iron chelation therapy is necessary.D. Common in persons of Mediterranean

ancestry

93. In the anemia of chronic disease, what arethe usual serum iron and transferrin levels?A. Serum iron decreased, transferrin

decreasedB. Serum iron decreased, transferrin

increasedC. Serum iron normal, transferrin normalD. Serum iron increased, transferrin

increased

94. In children, the most important effect oflead poisoning is on theA. LiverB. KidneyC. Neurologic systemD. Development of erythrocytes

95. Which of the following would not result inthe dual population of red cells representedin Color Plate 9B?A. Blood transfusionB. Oral iron therapyC. Spleen removalD. Coexisting deficiencies

96. What is the most likely genetic defect in thehemoglobin of cells seen in Color Plate 10B?A. Substitution of valine for glutamic acid

in position 6 of the alpha-globin chainB. Substitution of valine for glutamic acid

in position 6 of the beta-globin chainC. Substitution of lysine for glutamic acid

in position 6 of the alpha-globin chainD. Substitution of lysine for glutamic acid

in position 6 of the beta-globin chain

97. On what is the classification of sickle celltrait versus sickle cell disease based?A. Severity of the clinical symptomsB. Number of irreversibly sickled cells

(ISCs)C. Level of compensatory hemoglobin FD. Percentage of hemoglobin S on

electrophoresis

98. Which of the following is the mostappropriate treatment for sickle cellanemia?A. HydroxyureaB. Supportive therapyC. Hyperbaric oxygenD. Iron

99. Which of the following values can be usedto indicate the presence of a hemolyticanemia?A. Hemoglobin levelB. Hematocrit levelC. Erythrocyte countD. Reticulocyte count

100. A pre-operative, 20-year-old female has amild microcytic anemia, with target cellsand stippled red cells observed on theblood smear. Her hemoglobin A2 level isquantified at 5%. What do these findingssuggest?A. Iron-deficiency anemiaB. Heterozygous alpha-thalassemiaC. Heterozygous beta-thalassemiaD. Hemoglobin S/beta-thalassemia

101. What causes the hemolytic process inglucose-6-phosphate dehydrogenasedeficiency following oxidant exposure?A. Coating of red cells by antibodyB. Osmotic pressure changesC. Complement attachmentD. Precipitation of denatured hemoglobin

Page 71: Hematology

REVIEW QUESTIONS • 299

102. In clinically severe hereditary spherocyto-sis, which of the following findings wouldnot be found post-splenectomy?A. Rise in the red cell count and

hemoglobin levelB. Higher number of circulating

reticulocytesC. Increased number of Howell-Jolly

bodiesD. Transient elevation in the platelet count

103. Which of the following laboratory resultsis not consistent with accelerated red celldestruction?A. Increased serum bilirubinB. Increased plasma hemoglobinC. Increased serum lactate dehydroge-

nase (LD)D. Increased serum haptoglobin

104. Acquired hemolytic anemias are usuallydue toA. Extracorpuscular factorsB. Defects within the bone marrowC. Intracellular factorsD. Changes in hemoglobin stability

105. The antibody associated with paroxysmalcold hemoglobinuria shows specificity forA. ABO antigensB. I antigensC. P antigensD. Rh antigens

106. A 69-year-old male is admitted withpallor, mild tachycardia, and difficultywalking because of numbness in theextremities. His CBC reveals a hemoglo-bin of 78 g/L (7.8 g/dL), a hematocrit of0.25 L/L (25.0%), and MCV of 118.5 fL.This patient's symptoms and the bloodfindings seen in Color Plate 11 • are mostsuggestive of anemia due to a lack ofA. Folic acidB. Vitamin Bj2C. Vitamin B6

D. Ascorbic acid

107. A clinical laboratory scientist examined aWright's stained peripheral smear and sawwhat appeared to be small, dark-staininggranules in the mature erythrocytes. Asecond smear was stained with Prussianblue and a positive result was obtained.Based on this information, which of thefollowing would you expect to beabnormal?A. Plasma hemoglobin levelB. Serum ferritin levelC. Hemoglobin electrophoresisD. Test for parietal cell antibodies

108. Hemoglobinopathies are characterized byA. Absent or reduced rate of globin-chain

synthesisB. Inability to transport and release

oxygen to the tissuesC. Inhibition of iron chelation needed for

heme biosynthesisD. Production of structurally abnormal

hemoglobin variants

109. Which of the following statements abouthereditary spherocytosis is true?A. Abnormally shaped cells are produced

in the bone marrow.B. Cells have a decreased mean cell

hemoglobin concentration (MCHC).C. Membrane loss and red cell trapping

occur in the splenic microcirculation.D. Red cell osmotic fragility is decreased.

110. Which of the following statements abouthereditary elliptocytosis (HE) is true?A. Characteristic oval shape occurs in

mature erythrocytes.B. Heterogeneous group of disorders

linked to Rh-null individuals.C. Cellular defect involves the lipid

composition of the membrane.D. HE cells are abnormally permeable to

calcium.

Page 72: Hematology

300 • CHAPTER 2: HEMATOLOGY

111. Which of the following disorders is notcommonly linked to the development ofanemia of chronic disease?A. Persistent infectionsB. Noninfectious inflammatory disordersC. Chronic gastrointestinal blood lossD. Malignancy

112. Which of the following statements abouthemoglobin C disease is false?A. Electrophoresis shows approximately

60% hemoglobin A and 40% hemo-globin C.

B. Target cells are frequently seen onperipheral smears.

C. Red cells may contain bar-shapedintracellular crystals.

D. The disorder is less severe than sicklecell disease.

113. Which of the following is associated withsickle cells?A. Increased oxygen tension promotes

sickling.B. There is decreased mechanical

fragility.C. There is increased deformability.D. Increased sickling occludes vessels.

114. A bone marrow M:E ratio of 4:1 would bean expected finding forA. Sickle cell anemiaB. Aplastic anemiaC. Beta-thalassemia majorD. Megaloblastic anemia

115. An elderly man with a 10-year history ofchronic lymphocytic leukemia presentedwith jaundice and fatigue that wasattributed to a recent 3-gram drop in hishemoglobin. Many spherocytes andpolychromatophilic red cells were foundon his Wright's stained blood smear.

Which type of immune hemolytic anemiais most likely?A. Idiopathic warm autoimmune

hemolytic anemiaB. Secondary warm autoimmune

hemolytic anemiaC. Primary cold hemagglutinin diseaseD. Paroxysmal cold hemoglobinuria

116. A moderately anemic patient withsuspected pernicious anemia (PA) showsintrinsic factor antibodies and a lowcobalamin level. Which of the followingwould not support the diagnosis of PA?A. Gastric atrophy and achlorhydriaB. Oval macrocytes and Howell-Jolly

bodiesC. Bone marrow erythroid precursors

exhibit normoblastic maturation.D. Elevated serum lactate dehydrogenase

(LD) and bilirubin levels

117. A cellulose acetate electrophoresisrevealed a large band of hemoglobin inthe hemoglobin S position. This bandquantified at 95%. The peripheral smearrevealed 70% target cells, and thesolubility test was negative. Based on thisinformation, what is the hemoglobin?A. Hemoglobin CB. Hemoglobin DC. Hemoglobin ED. Hemoglobin S

118. A previously healthy man experiencesweakness and hemoglobinuria after takingthe antimalarial agent primaquine. Thishemolytic attack most likely occurredbecause of a deficiency ofA. Pyruvate kinaseB. Glucose-6-phosphate dehydrogenaseC. 2,3-BisphosphoglycerateD. Methemoglobin reductase

Page 73: Hematology

REVIEW QUESTIONS • 301

119. Which of the following is an acquired redcell membrane defect that results inincreased sensitivity to complementbinding?A. March hemoglobinuriaB. Paroxysmal nocturnal hemoglobinuriaC. Paroxysmal cold hemoglobinuriaD. Methemoglobinemia

120. Which of the following is not associatedwith acquired reversible sideroblasticanemias?A. Methotrexate therapyB. Lead intoxicationC. Isoniazid treatment for tuberculosisD. Acute alcohol ingestion

121. Which of the following statements aboutthe relative anemia of pregnancy is false?A. It is due to a reduction in the number

of erythrocytes.B. It is normocytic and normochromic.C. It does not produce an oxygen deficit

for the fetus.D. It is associated with an increase in

plasma volume.

122. The anemia found in chronic renal failureis most likely caused byA. Loss of erythropoietin synthesisB. Lack of cellular oxygen demandC. Defective iron absorptionD. Destruction of red cells by uremic

metabolites

123. Which of the following phrases aboutaplastic anemia is false?A. Stem cell disorderB. Risk of life-threatening infectionC. Frequent bleeding complicationsD. Reduced red cell survival

124. The fish tapeworm Diphyllobothrium latumis associated with the development ofA. Microcytic anemiaB. Macrocytic anemiaC. Hemolytic anemiaD. Hypoproliferative anemia

125. An increase in erythropoietin is nota normal compensating mechanism inwhich of the following conditions?A. Renal tumorsB. Heavy smokingC. Cardiovascular diseaseD. Pulmonary disease

126. Thalassemias are the result of aA. Structural defect in the heme portion

of hemoglobinB. Quantitative defect in globin-chain

synthesisC. Qualitative defect in globin-chain

structureD. Change in hemoglobin solubility

properties

127. Which of the following characterizes iron-deficiency anemia?A. Decreased serum iron, decreased

transferrin saturation, normal ferritinB. Decreased serum transferrin,

decreased transferrin saturation,decreased ferritin

C. Increased serum transferrin, decreasedtransferrin saturation, decreased ferritin

D. Increased serum transferrin, increasedtransferrin saturation, decreased serumiron

128. Clinical manifestations of a homozygousmutation involving the beta-globin genewill most likely appearA. During embryonic developmentB. In the neonate at birthC. No later than 3 weeks after birthD. By 6 months of age

Page 74: Hematology

302 • CHAPTER 2: HEMATOLOGY

129. The hemolysis associated with infectionby malaria organisms is due to theA. Release of merozoites from erythro-

cytesB. Invasion of erythrocytes by merozoitesC. Host's immunologic response to

infected erythrocytesD. Toxins produced by the malarial

organism

130. A clinical laboratory scientist received a5 mL EDTA tube that contained 0.5 mLof anticoagulated blood. A smear wasprepared and stained with Wright's stain.When examined microscopically, themajority of cells appeared to have manyevenly distributed, blunt spicules on thesurface. How should this cellularappearance be interpreted?A. An anemic condition requiring further

testingB. Spur cells caused by using incorrect

technique during slide preparationC. Artifact caused by a dirty spreader

slideD. Crenated cells caused by incorrect

blood to anticoagulant ratio

131. A failure to generate sufficient ATP ischaracteristic of red blood cells withA. Pyruvate kinase deficiencyB. Glucose-6-phosphate dehydrogenase

deficiencyC. Lipoprotein deficiencyD. Hexokinase deficiency

132. When iron use exceeds absorption, whichof the following occurs first?A. Hemoglobin level decreases.B. Iron stores are depleted.C. Transferrin synthesis increases.D. Excretion of iron decreases.

133. The major mechanism responsible for theanemia of chronic disease isA. Impaired release of storage iron

because of increased hepcidin levelsB. Damaged bone marrow stem cellsC. Immune destruction caused by red cell

autoantibodiesD. Increased erythropoietin response by

committed red cell progenitor cells

134. Which of the following is not a character-istic of the idiopathic type of sideroblasticanemia?A. Refractory to treatmentB. Blocks in heme synthesis are unknownC. Reversible with intramuscular vitamin

Bj2 injectionsD. Subtype of myelodysplastic syndromes

135. Thinning of bones and deformation offacial bone structure seen in homozygousbeta-thalassemia is aA. Consequence of disturbances in

calcium metabolismB. Result of hyperplastic marrow activityC. Secondary disorder due to immuno-

logic responseD. Result of increased fibroclast activity

136. Which of the following does not accu-rately describe cold autoimmunehemolytic anemia?A. Red cell agglutination in extremities

induces Raynaud's phenomenon.B. It may occur secondary to

Mycoplasma pneumonia.C. Hemolysis is complement-mediated or

via removal of coated cells.D. The autoantibody is usually an IgG

type directed against Rh antigens.

Page 75: Hematology

REVIEW QUESTIONS

137. Which of the following represents ananemia that would have a high red celldistribution width (RDW)?A. Sickle cell disease during crisisB. Thalassemia minorC. Aplastic anemiaD. Anemia of chronic disorders

138. In which of the following disorders wouldsplenomegaly not be a common finding?A. Homozygous beta-thalassemiaB. Hereditary spherocytosisC. Hemoglobin SC diseaseD. Folic acid deficiency

Leukocytes139. Functionally, white blood cells are divided

intoA. Granulocytes, nongranulocytesB. Polymorphonuclears, mononuclearsC. Phagocytes, immunocytesD. Granulocytes, lymphocytes

140. What is the largest white blood cellnormally found in the peripheral blood?A. EosinophilB. NeutrophilC. LymphocyteD. Monocyte

141. What is the approximate amount of timea granulocyte spends in the circulationbefore migrating into the tissues?A. Less than 1 dayB. About 3 daysC. Up to 5 daysD. More than 10 days

142. What percentage of neutrophils in theperipheral blood constitutes the circulat-ing pool?A. 100%B. 80%C. 50%D. 30%

143. What is the major phagocytic cell involvedin the initial defense against bacterialpathogens?A. NeutrophilB. EosinophilC. BasophilD. Monocyte

144. What is the growth factor that is primarilyresponsible for regulating granulocyte andmonocyte production?A. ErythropoietinB. Colony stimulating factorC. InterleukinD. Thrombopoietin

145. What does the granulocyte mitotic pool inthe bone marrow contain?A. Myeloblasts and promyelocytesB. Band and segmented formsC. The majority of marrow granulocytesD. Myelocytes and metamyelocytes

146. A "shift to the left," when used to describea cell population, refers toA. Increased cells in the blood due to a

redistribution of blood poolsB. An increase in immature blood cells

following release of bone marrowpools

C. A cell production "hiatus" or gapD. A higher percentage of lymphocytes

than neutrophils

147. Which of the following is characteristic ofagranulocytosis?A. Neutrophils without granulesB. Decreased numbers of granulocytes,

red cells, and plateletsC. Immature granulocytes in the

peripheral bloodD. Decreased numbers of granulocytes

Page 76: Hematology

304 CHAPTER 2: HEMATOLOGY

148. Which of the following is not a character-istic of T lymphocytes?A. Secrete cytokinesB. Synthesize antibodyC. Comprise majority of cells in the

blood lymphocyte poolD. Regulate the immune response

149. An adult has a total white blood cell countof 4.0 X 109/L (4.0 X 103/jJiL). The dif-ferential count is as follows: polymor-phonuclear neutrophils (PMNs) 25%,bands 5%, lymphocytes 65%, and mono-cytes 5%. The absolute value referencerange for lymphocytes is 1.0—4.0 X 109/L.Which of the following is true?A. The percentage of lymphocytes is

normal.B. There is an absolute lymphocytosis.C. There is a relative lymphocytosis.D. There is both an absolute and a relative

lymphocytosis.

150. Which of the following statements iscorrect?A. Hypersegmented neutrophils have four

nuclear lobes.B. Auer rods are composed of fused

primary granules.C. Toxic granules are prominent

secondary granules.D. Dohle bodies are agranular patches of

DNA.

151. Which of the following factors is notassociated with variations in the totalwhite blood cell count?A. AgeB. ExerciseC. Emotional stressD. Sex

152. Of the following, an absolute neutrophilcount of 1.0 X 109/L would be associatedwithA. Shortness of breathB. Bleeding tendenciesC. Risk of infectionD. No clinical symptoms

153. Which of the following statements aboutbasophils is false?A. Morphologically, basophils resemble

tissue mast cells.B. Membrane receptors bind IgG,

initiating anaphylactic reactions.C. Basophilic granules contain heparin

and histamine.D. Granules are water soluble.

154. The most mature granulocyte precursorthat can undergo mitosis is theA. MyeloblastB. PromyelocyteC. MyelocyteD. Metamyelocyte

155. Production of primary granules ceasesand production of secondary granulescommences with what cell stage?A. MyeloblastB. PromyelocyteC. MyelocyteD. Metamyelocyte

156. Which of the following statements abouteosinophils is false?A. They contain a type of peroxidase that

is distinct from that of neutrophils.B. Eosinophilic granules contain

lysozyme.C. Eosinophils are an important line of

defense against parasites.D. Major basic protein is a component of

eosinophil granules.

Page 77: Hematology

REVIEW QUESTIONS • 305

157. Which of the following is characteristic ofprimary granules?A. Coated with a phospholipid membraneB. Called azurophilic or specific granulesC. Contain myeloperoxidase and

lactoferrinD. Present in the promyelocyte stage only

158. Which of the following are indicators of aneutrophilic response to tissue damage orinflammatory stimuli?A. Toxic granules and Dohle bodies in

the neutrophilsB. Vacuoles and Barr bodies in the

neutrophilsC. Hypersegmented neutrophils and

Auer rodsD. Pyknotic neutrophils and Russell

bodies

159. What is the term for cell movementthrough blood vessels to a tissue site?A. DiapedesisB. OpsonizationC. MarginationD. Chemotaxis

160. Vasodilation and bronchoconstriction arethe result of degranulation by which of thefollowing blood cells?A. EosinophilsB. MonocytesC. NeutrophilsD. Basophils

161. On what basis can B and T lymphocytesbe distinguished?A. Differences in nuclear shapeB. Monoclonal antibody reactions to

surface and cytoplasmic antigensC. Cytoplasmic granularity and overall

cell sizeD. Chromatin pattern in the nucleus

162. Cells that produce immunoglobulins inresponse to antigenic stimulation aredesignatedA. Natural killer cellsB. Plasma cellsC. VirocytesD. Thymocytes

163. Which of the following statements aboutneutrophils is false?A. Suppress allergic reactions caused by

basophilsB. Have surface receptors for IgG and

complement componentsC. Contain alkaline phosphatase and

muramidaseD. Act in nonspecific phagocytosis and

are destined to die

164. Which of the following characteristicswould be least likely to distinguishreactive lymphocytes from monocytes?A. Sharp indentation of the cytoplasmic

margin by adjacent red blood cellsB. Presence of large azurophilic granulesC. Irregular, indented nuclear shapeD. Abundant, deeply basophilic

cytoplasm

165. Which of the following can differentiatemetamyelocytes from other stages ofgranulocyte maturation?A. Presence of specific granulesB. Indentation of nucleusC. Absence of nucleoliD. Color of cytoplasm

166. Lymphocyte concentrations in theperipheral blood are greatest duringwhat age interval?A. 1 to 4 yearsB. 4 to 15 yearsC. 16 to 40 yearsD. 40 to 70 years

Page 78: Hematology

306 CHAPTER 2: HEMATOLOGY

167. Which of the following is the least likelyto be expressed by early B cell precursors?A. SIgM, a surface membrane

immunoglobulinB. CD34, a hematopoietic stem cell

markerC. TdT (terminal deoxynucleotidyl

transferase), a nuclear enzymeD. CDlO(CALLA), a surface antigen

168. Which of the following statements aboutmacrophages is incorrect?A. They are mature tissue forms of blood

monocytes.B. They serve as antigen-presenting cells

to the immune system.C. Their quantity of lysosomes and

acid hydrolases decreases duringmaturation.

D. They remove damaged or dying cellsand cellular debris.

169. Antigen-dependent lymphopoiesis occursin secondary lymphoid tissue located intheA. Liver and kidneyB. Spleen and lymph nodesC. Lungs and Peyer's patchesD. Thymus and bone marrow

170. Which of the following is not produced byneutrophils during the respiratory burst?A. Hydroxyl radicals (OH~)B. Hydrogen peroxide (H2O2)C. Superoxide anion (O^~)D. Myeloperoxidase

Leukocyte Disorders171. In patients with infectious mononucleosis,

which blood cells are infected by thecausative agent?A. MonocytesB. T lymphocytesC. B lymphocytesD. Histiocytes

172. Which of the following statements abouthairy cell leukemia is true?A. It is an acute disease, primarily

affecting young adults.B. Splenomegaly is an unusual finding.C. Hairy cells contain tartrate-resistant

acid phosphatase.D. Hairy cells are abnormal T lympho-

cytes.

173. Based on the WHO classificationsystem, B cell ALL (FAB type L3) and

represent differentclinical presentations of the samedisease entity.A. Burkitt lymphomaB. Hodgkin lymphomaC. Mycosis fungoidesD. Small lymphocytic lymphoma

174. The presence of both immature neutrophilsand nucleated erythrocytes in the periph-eral blood is most accurately called aA. Neutrophilic left shiftB. Regenerative left shiftC. Neutrophilic leukemoid reactionD. Leukoerythroblastic reaction

175. In which anomaly is a failure of granulo-cytes to divide beyond the band or two-lobed stage observed?A. Pelger-HuetB. May-HegglinC. Alder-ReillyD. Chediak-Higashi

176. In which of the following are eosinophilsnot increased?A. Gushing syndromeB. Allergic disordersC. Skin disordersD. Parasitic infection

Page 79: Hematology

REVIEW QUESTIONS • 307

177. Which of the following represents theprincipal defect in chronic granulomatousdisease (CGD)?A. Chemotactic migrationB. PhagocytosisC. Lysosomal formation and functionD. Oxidative respiratory burst

178. The blood shown in Color Plate 11 • is froma leukemia patient following treatment.These findings are most suggestive oftherapy withA. Corticosteroids (e.g., prednisone)B. A folate antagonist (e.g., methotrexate)C. Recombinant erythropoietinD. Chloramphenicol

179. A patient with normal hemoglobin andWBC count values, a persistently elevatedplatelet count (over 1000 X 109/L),increased marrow megakaryocytes, and ahistory of frequent bleeding and clottingepisodes most likely hasA. Polycythemia veraB. Chronic myelofibrosisC. Essential thrombocythemiaD. Chronic myelogenous leukemia

180. An adult patient with massivesplenomegaly has mild anemia, a slightlyelevated WBC count, and an LAP score of170. The blood smear shows teardroperythrocytes and leukoerythroblastosis.These findings are most consistent withA. Chronic myelogenous leukemiaB. Idiopathic myelofibrosisC. Primary polycythemiaD. Primary thrombocythemia

181. Which of the following infections does notreveal a blood picture as seen in ColorPlate 12B?A. Epstein-Barr virus (EBV)B. Bordetellapertussis (whooping cough)C. Cytomegalovirus (CMV)D. Toxoplasma gondii (toxoplasmosis)

182. The most common type of chroniclymphocytic leukemia (CLL) in theUnited States involves theA. BcellB. NKcellC. TcellD. Plasma cell

183. Which of the following are characteristicfindings in Waldenstrom disease?A. Increased IgA and

hepatosplenomegalyB. Increased IgE and renal failureC. Increased IgG and hypercalcemiaD. Increased IgM and blood hyperviscosity

184. Which of the following would not cause atotal WBC count of 62.2 X 109/L (62.2 X103/jjLL) and the blood findings seen inColor Plate 13B?A. Treatment with myeloid growth factorsB. Gram-negative septicemiaC. Human immunodeficiency virus (HIV)D. Systemic fungal infection

185. The peripheral blood shown in Color Plate14B is from a 69-year-old female. HerWBC count was 83.0 X 109 cells/L (83.0 X103/(xL) and her platelet count wasnormal. Based on the cell morphology andthis information, what is the most likelydiagnosis?A. Acute lymphoblastic leukemiaB. Chronic lymphocytic leukemiaC. Waldenstrom macroglobulinemiaD. Viral infection

186. In which of the following is progression toacute leukemia least likely?A. Chronic myelogenous leukemia (CML)B. Refractory anemia with excess blasts

(RAEB)C. Refractory anemia with ringed

sideroblasts (RARS)D. Chronic lymphocytic leukemia (CLL)

Page 80: Hematology

CHAPTER 2: HEMATOLOGY

187. A Gaucher cell is best described as amacrophage withA. "Wrinkled" cytoplasm due to an

accumulation of glucocerebrosideB. "Foamy" cytoplasm filled with

unmetabolized sphingomyelinC. Pronounced vacuolization and

deposits of cholesterolD. Abundant cytoplasm containing

storage iron and cellular remnants

188. Which of the following suggests adiagnosis of Hodgkin disease rather thanother lymphoproliferative disorders?A. Presence of a monoclonal population

of large lymphoid cellsB. Predominance of immature B cells

with irregular nuclear cleftsC. Circulating T cells with a convoluted,

cerebriform nucleusD. Presence of giant binucleated Reed-

Sternberg cells with prominentnucleoli

189. In a patient with fever of unknown origin,which of the following findings is notconsistent with an inflammatory process?A. Increased C-reactive proteinB. Increased albumin levelC. Increased fibrinogen levelD. Increased erythrocyte sedimentation

rate

190. The presence of the chromosomalabnormality t(15;17) and a high incidenceof disseminated intravascular coagulation(DIG) is diagnostic ofA. Acute myeloblastic leukemia without

maturation (FAB type Ml)B. Acute myeloblastic leukemia with

maturation (FAB type M2)C. Acute promyelocytic leukemia (FAB

type M3)D. Acute myelomonocytic leukemia

(FAB type M4)

191. Which of the following is not commonlyfound in acute myelogenous leukemias?A. NeutropeniaB. ThrombocytopeniaC. HepatosplenomegalyD. Lymphadenopathy

192. The child whose blast cells are shown inColor Plate 15> has acute lymphoblasticleukemia that is precursor B cell typeand CALLA positive. Analysis by flowcytometry would likely show cells thatimmunophenotype forA. CD2,CD7B. CD10,CD19C. CD13,CD33D. CD14,CD34

193. The patient whose bone marrow is shownin Color Plate 16 • most likely has a(n)A. Acute leukemiaB. Chronic leukemiaC. Myelodysplastic syndromeD. Aplastic anemia

194. Multiple myeloma is characterized by thepresence in urine of large amounts ofA. CryoglobulinsB. IgG heavy chainsC. IgG light chainsD. Beta microglobulins

195. Which of the following is not classified asa myeloproliferative disorder?A. Polycythemia veraB. Essential thrombocythemiaC. Multiple myelomaD. Chronic myelogenous leukemia

196. Which of the following gene mutationscorrelates with the t(9;22) that is presentin Philadelphia chromosome positivechronic myelogenous leukemia?A. MYC/IGHB. BCR/ABLC. PML/RARAD. JAK2

Page 81: Hematology

REVIEW QUESTIONS • 309

197. Which of the following statements doesnot correctly describe the WHO (WorldHealth Organization) classification ofhematopoietic neoplasms?A. Acute leukemia is defined as the

presence of at least 20% bone marrowblasts.

B. Diagnosis is based on cellularmorphology and cytochemistry.

C. It groups lymphoid disorders intoB cell, T/NK cell, and Hodgkinlymphoma.

D. Diagnostic criteria include morpho-logic, cytochemical, immunologic,cytogenetic, and molecular features.

198. Which of the following would be leasthelpful in distinguishing chronic myeloge-nous leukemia (CML) from a neutrophilicleukemoid reaction?A. An extreme leukocytosis with

increased neutrophilic bands,metamyelocytes, and myelocytes

B. Leukocyte alkaline phosphatase scoreC. Presence of marked splenomegalyD. Neutrophils with Dohle bodies and

toxic granulation

199. The cytoplasmic inclusion present in thecell shown in Color Plate !?•A. Excludes a diagnosis of acute

myelogenous leukemiaB. Stains positive with leukocyte alkaline

phosphatase (LAP)C. Stains positive with myeloperoxidase

(MPO)D. Identifies the cell as a malignant

lymphoblast

200. Which of the following is a typical findingin chronic leukemias at onset?A. Symptoms of infection and bleedingB. Significant thrombocytopeniaC. Severe anemiaD. Elevated leukocyte count

201. In what condition would an LAP score of10 most likely be found?A. Bacterial septicemiaB. Late pregnancyC. Polycythemia veraD. Chronic myelogenous leukemia

202. Which of the following is not associatedwith neutrophilia?A. Staphylococcal pneumoniaB. Cm shing inj uryC. Infectious hepatitisD. Neoplasms (tumors)

203. In which of the following would anabsolute monocytosis not be seen?A. TuberculosisB. Recovery stage of acute bacterial

infectionC. Collagen disordersD. Infectious mononucleosis

204. Coarse PAS positivity may be found in theleukemic cells ofA. Acute myeloblastic leukemia (FAB

type Ml)B. Acute lymphoblastic leukemia (FAB

type LI)C. Acute myelomonocytic leukemia

(FAB type M4)D. Acute monocytic leukemia (FAB

type M5)

205. Which of the following is not among thediagnostic criteria used for classifying themyelodysplastic syndromes?A. Unexplained anemia refractory to

treatmentB. Hypogranular and hyposegmented

neutrophilsC. Abnormal platelet size and

granulationD. Hypocellular bone marrow with 25%

blasts

Page 82: Hematology

310 CHAPTER 2: HEMATOLOGY

206. Naphthol AS-D chloroacetate esterase(specific) is usually positive in

cells, and alpha-naphthyl acetate esterase (nonspecific)is useful for identifying blast cells of

lineage.A. Granulocytic; monocyticB. Monocytic; granulocyticC. Granulocytic; lymphocyticD. Monocytic; lymphocytic

207. The familial disorder featuring pseudo-Dohle bodies, thrombocytopenia, andlarge platelets is calledA. May-Hegglin anomalyB. Chediak-Higashi syndromeC. Pelger-Huet anomalyD. Alder-Reilly anomaly

208. Alder-Reilly anomaly is an abnormality ofA. Lysosomal fusionB. Nuclear maturationC. Oxidative metabolismD. Mucopolysaccharide metabolism

209. What is the initial laboratory techniquefor the diagnosis of monoclonalgammopathies?A. Immunologic markers of marrow

biopsy cellsB. Cytochemical staining of marrow and

peripheral blood cellsC. Serum and urine protein

electrophoresisD. Cytogenetic analysis of marrow cells

210. Which of the following statements aboutHodgkin disease is false?A. Peak incidence occurs in young adults.B. Staging determines extent of disease

and treatment course.C. Stage IV has the best prognosis.D. Almost a 2:1 male predominance over

females is characteristic.

211. The blast cells shown in Color Plate 18 •are CD14 and CD33 positive, Sudan blackB positive, specific esterase positive, andnonspecific esterase positive. Which typeof acute leukemia is most consistent withthe immunophenotyping and cytochemicalstaining results?A. Acute lymphoblastic leukemia, T cell

typeB. Acute erythroleukemiaC. Acute myelomonocytic leukemiaD. Acute monocytic leukemia

212. Which type of leukemia is associated withthe best prognosis for a cure?A. Chronic lymphocytic leukemia in the

elderlyB. Acute lymphoblastic leukemia in

childrenC. Acute myelogenous leukemia in

childrenD. Chronic myelogenous leukemia in

young adults

213. What is the key diagnostic test forHodgkin lymphoma?A. Bone marrow biopsyB. Lymph node biopsyC. Spinal tapD. Skin biopsy

214. A bone marrow with 90% cellularity andmyeloid:erythroid (M:E) ratio of 10:1 ismost characteristic ofA. Chronic myelogenous leukemiaB. Primary polycythemiaC. Beta-thalassemia majorD. Aplastic anemia

Page 83: Hematology

REVIEW QUESTIONS • 311

215. A 60-year-old patient presents withextreme fatigue. Her blood and bonemarrow findings are as follows: severeanemia with a dual RBC population, 3%marrow blasts, and numerous ringedsideroblasts. This information is mostconsistent withA. Refractory anemia (RA)B. Refractory anemia with ringed

sideroblasts (RARS)C. Refractory anemia with excess blasts

(RAEB)D. Chronic myelomonocytic leukemia

(CMML)

216. Which of the following is not amechanism by which neutropeniamay be produced?A. HypersplenismB. Marrow injury or replacementC. Recent strenuous exerciseD. Drug-induced antibodies

217. Which of the following is not a character-istic finding in polycythemia vera?A. Blood pancytosisB. Increased red cell massC. Increased erythropoietin levelD. Increased blood viscosity

218. In what disorder is significant basophiliamost commonly seen?A. Hairy cell leukemiaB. Plasma cell leukemiaC. Acute lymphoblastic leukemiaD. Chronic myelogenous leukemia

219. Acute erythroleukemia (FAB type M6) ischaracterized by increasedA. Promyelocytes and lysozyme activityB. Marrow megakaryocytes and

thrombocytosisC. MaiTow erythroblasts and multinucle-

ated red cellsD. Marrow monoblasts and immature

monocytes

220. The blood findings present in Color Plate20 • are from a patient with complaints offatigue and severe lower back pain. Whichof the following would not be typical ofthis disease?A. Bone tumors of plasma cellsB. HypercalcemiaC. Progressive renal impairmentD. Normal sedimentation rate

221. Myeloid metaplasia refers toA. Displacement of normal marrow cells

by fibrous tissueB. Hematopoietic failureC. Extramedullary hematopoiesisD. Tumors (neoplasms) of the bone marrow

222. Which of the following statements aboutnon-Hodgkin types of lymphoma is true?A. Lymphadenopathy is the most

common presenting symptom.B. Initially, they present as a systemic

disease rather than a localized tumor.C. They are often associated with

multiple bone lesions.D. They are characterized by proliferation

of malignant cells primarily involvingthe bone marrow.

Methodology223. What combination of reagents is used to

measure hemoglobin?A. Hydrochloric acid and p-dimethyl-

aminobenzaldehydeB. Potassium ferricyanide and potassium

cyanideC. Sodium bisulfite and sodium

metabisulfiteD. Sodium citrate and hydrogen peroxide

224. The slowest-moving hemoglobin(s) on analkaline electrophoresis at pH 8.6 is(are)A. AB. A2, C,E, andOC. FD. S, D, andG

Page 84: Hematology

312 • CHAPTER 2: HEMATOLOGY

225. A patient with suspected sickle cell traithas negative solubility test results, buthemoglobin electrophoresis at pH 8.6shows an apparent A-S pattern. What isthe most likely explanation?A. Patient has hemoglobin AS, and the

solubility test is incorrect.B. Patient has hemoglobin AA, and the

electrophoresis is incorrect.C. Patient has hemoglobin AD or AG, and

both procedures are correct.D. Tests need to be repeated; impossible to

determine which procedure is correct.

226. Which of the following is an incorrectstatement about the solubility test forHemoglobin S?A. Hemoglobin S polymerizes when

deoxygenated.B. Testing performed on a 2-day-old infant

can result in a false negative result.C. Sickle cell trait can be differentiated

from sickle cell anemia with this test.D. The test is positive in hemoglobin

^Harlem-

227. Which of the following is not associatedwith causing a falsely low ESR?A. Column used is slanted.B. EDTA tube is clotted.C. EDTA tube is one-third full.D. EDTA specimen is 24 hours old.

228. A platelet count is performed on anautomated instrument from an EDTAblood sample. Smear evaluation revealsthe presence of platelet clumps. Thespecimen is redrawn using sodium citrateas the anticoagulant, and a count of300 X 109/L is obtained. What is thecorrect platelet count to report?A. 270X109/LB. 300X109/LC. 330X109/LD. 360X109/L

229. To best preserve cellular morphology,differential smears from an EDTAspecimen should be made no more than

hour(s) after collection.A. 1B. 5C. 12D. 24

230. The blood smear made on a patient withpolycythemia vera is too short. Whatshould be done to correct this problem?A. Decrease the angle of the spreader slide.B. Increase the angle of the spreader slide.C. Adjust the angle of the spreader slide

to 45 degrees.D. Use a smaller drop of blood.

231. The components of Wright's stain includeA. Crystal violet and safraninB. Brilliant green and neutral redC. New methylene blue and carbolfuchsinD. Methylene blue and eosin

232. What is the reason for red blood cells to bebright red and the WBC nuclei to bepoorly stained when using Wright's stain?A. The staining time is too long.B. The stain or buffer is too alkaline.C. The stain or buffer is too acidic.D. The smear was not washed long enough.

233. If 60 reticulocytes are counted in 1000 redblood cells, what is the reticulocyte count?A. 0.06%B. 0.6%C. 6.0%D. 60.0%

234. Using the percent reticulocyte from question233 and an RBC count of 3.00 X 1012/L(3.00 X 106/|xL), the calculated absolutereticulocyte count reported in SI units isA. 1.8X109/LB. 18X109/LC. 180X109/LD. 180X103/(JLL

Page 85: Hematology

REVIEW QUESTIONS • 313

235. The Sudan black B stain shown in ColorPlate 19B is a stain forA. GlycogenB. LipidsC. MyeloperoxidaseD. Acid phosphatase

236. The following numbers were obtained inevaluating leukocyte alkaline phosphatase(LAP) activity in neutrophils. What isthe score?

240. A WBC count is performed on a hemacy-tometer using a 1:20 dilution. 308 cells areseen in a total area of 8 mm2. What is theWBC count?A. 3.8X109/LB. 7.7X109/LC. 15.4X109/LD. 38.5X109/L

241. Which set of results indicates that an errorin measurement has occurred?

0

15

1

20

2

30

3

20

4

15

A. 100B. 115C. 200D. 215

237. Perl's Prussian blue is a stain used todetectA. DNAB. RNAC. IronD. Glycogen

238. Which of the following red cell inclusionsstain with both Perl's Prussian blue andWright's stain?A. Howell-Jolly bodiesB. Basophilic stipplingC. Pappenheimer bodiesD. Heinz bodies

239. What is the depth between the countingplatform and the coverslip on a hemacy-tometer?A. 0.01mmB. 0.10mmC. 1.00mmD. 0.1 cm

3B.

C.

D.

RBCX1012/L

2.50

2.75

3.40

3.75

Hgb (g/dL)

7.6

9.5

10.0

11.1

Hct (%)

22.9

24.8

31.0

34.0

242. Which of the following would not be thecause of a falsely high MCHC of 38.3 g/dLon an automated instrument?A. Hereditary spherocytosisB. LipemiaC. Presence of a cold agglutininD. Instrument sampling or mixing error

243. What is the principle of automatedimpedance cell counters?A. Angle of laser beam scatter by cellsB. Amplification of an electrical current

by cellsC. Interruption of an electrical current by

cellsD. Change in optical density of the

solution containing cells

244. A clinically significant difference betweentwo electronic cell counts is indicatedwhen the standard deviation is greater thanA. ±1.0B. ±1.5C. ±2.0D. ±3.0

Page 86: Hematology

314 CHAPTER 2: HEMATOLOGY

245. Side angle scatter in a laser-based cellcounting system is used to measureA. Cell sizeB. Cytoplasmic granularityC. Cell numberD. Immunologic (antigenic) identification

246. A white blood cell count is done on anautomated impedance cell counter from apatient with the blood picture seen in ColorPlate 4 •. The WBC count is most likelyA. Falsely increased because of nRBCsB. Falsely increased because of red cell

fragmentsC. Falsely decreased because of nRBCsD. Accurate; no error with this metho-

dology

247. The hemoglobin A2 quantification usinganion exchange chromatography will bevalid inA. Hemoglobin C diseaseB. Hemoglobin E traitC. Hemoglobin O traitD. Beta-thalassemia minor

248. Which of the following is not associatedwith an increased osmotic fragility and adecreased surface area-to-volume ratio?A. Beta-thalassemia majorB. Hereditary spherocytosisC. Warm autoimmune hemolytic anemiaD. Burn victims

249. A clotted EDTA tube can be used toperform a(n)A. Erythrocyte sedimentation rateB. Solubility test for hemoglobin SC. HematocritD. Platelet count

250. The test value range that includes 95% ofthe normal population is theA. Reference intervalB. Linearity limitC. Reportable rangeD. Critical range

251. To establish a standard curve for readinghemoglobin concentration,A. A commercial control material is

used.B. A wavelength of 640 nm is employed.C. Certified standards are used.D. A patient blood sample of known

hemoglobin concentration is used.

252. Which of the following is not a source oferror when measuring hemoglobin by thecyanmethemoglobin method?A. Excessive anticoagulantB. White blood cell count that exceeds

linearity limitsC. Lipemic plasmaD. Scratched or dirty hemoglobin

measuring cell

253. Which of the following statements aboutmicrohematocrits is false?A. Excessive centrifugation causes

falsely low results.B. A tube less than half full causes falsely

low results.C. Hemolysis causes falsely low results.D. Trapped plasma causes falsely high

results.

254. The erythrocyte sedimentation rate(ESR) is influenced by the red cellphenomenon seen in Color Plate 20 •.Which of the following factors willneither contribute to this phenomenonnor affect the ESR?A. Size of the red blood cellsB. Shape of the red blood cellsC. Hemoglobin content of the red blood

cellsD. Composition of the plasma

Page 87: Hematology

REVIEW QUESTIONS • 315

255. An EDTA blood sample run on anautomated impedance cell counter hasgenerated a warning flag at the upperregion of the platelet histogram illustratedbelow. Which of the following would notbe a cause of this warning flag?A. Nucleated RBCsB. Microcytic RBCsC. EDTA-dependent platelet agglutininsD. Giant platelets

Platelet Histogram

Femtoliters

256. To evaluate normal platelet numbers in anappropriate area of a blood smear,approximately how many platelets shouldbe observed per oil immersion field?A. 1-4B. 4-10C. 8-20D. 20-50

257. Which of the following statements aboutmanual reticulocyte counts is false?A. The blood/stain mixture is incubated

for 5-10 minutes.B. New methylene blue, a supravital

stain, is used.C. RBC inclusions can result in falsely

elevated counts.D. An erythrocyte must have at least 4

blue particles to be counted as areticulocyte.

258. When are automated cell countersrequired to have a calibration checkperformed?A. At least every 3 monthsB. After replacement of any major partC. After performing monthly mainte-

nanceD. When the control values are greater

than 2 standard deviations from themean

259. A blood sample was run through anautomated cell counter and the followingresults were obtained: WBC 6.9 X 109/L(6.9 X 103/(JiL), RBC 3.52 X 1012/L (3.52X 106/|xL), Hgb 120 g/L (12.0 g/dL), Hct0.32 L/L (32.0%), MCH 34.1 pg, MCHC37.5 g/dL. Which of the troubleshootingsteps that follows should be performed toobtain reportable results?A. Perform a saline replacement

procedure.B. Warm the specimen to 37°C and rerun.C. Perform a microhematocrit.D. None; the results are reportable.

260. Which of the following tests could beperformed on a hemolyzed blood sample?A. Hemoglobin onlyB. Hemoglobin and platelet countC. RBC count and hematocritD. No results would be reportable.

261. For which of the following procedureswould heparin be a recommendedanticoagulant?A. Platelet countB. Coagulation testsC. Smear-based red cell morphologyD. Osmotic fragility

Page 88: Hematology

316 • CHAPTER 2: HEMATOLOGY

262. In the platelet count procedure usingphase microscopy,A. Platelets appear dark against a light

background.B. The entire ruled counting surface of

the hemacytometer is used.C. Ammonium oxalate will lyse the WBCs.D. Platelets should be counted immedi-

ately after plating the hemacytometer.

263. What is the quality control term used todescribe the reproducibility of a test?A. AccuracyB. PrecisionC. Standard deviationD. Specificity

Case Histories

Use the following information to answerquestions 264-268.

The peripheral blood shown in Color Plate 41 isfrom a 10-month-old Greek boy with the followingresults on an automated impedance counter: WBC35.0 X 109/L (35.0 X 103/(xL); RBC 2.50 x 1012/L(2.50 x 106/|JL,L); hemoglobin 45 g/L (4.5 g/dL);hematocrit 0.16 L7L (16%); platelet count250 X 109/L (250,000/(jLl_); reticulocyte count 8.0%;110 nucleated red blood cells/100 WBCs andmany targets are seen. Other laboratory results areas follows: serum iron elevated; total iron-bindingcapacity (TIBC) decreased; serum ferritin elevated.

265. What would be the appearance of thechild's red blood cells on a peripheralsmear?A. Microcytic, hypochromicB. Normocytic, hypochromicC. Normocytic, normochromicD. Microcytic, normochromic

266. The CBC, serum iron, total iron-bindingcapacity, and serum ferritin levels are mostcharacteristic ofA. Beta-thalassemia minorB. Iron-deficiency anemiaC. Alpha-thalassemia minorD. Beta-thalassemia major

267. What type(s) of hemoglobin will bedetected on this child using hemoglobinelectrophoresis?A. A onlyB. AandFC. A, increased A2, FD. F only

268. Why is it difficult to diagnose this disorderin a newborn?A. The liver is immature.B. The beta chains are not fully

developed at birth.C. It is similar to hemolytic disease of the

newborn (HDN) because of ABOincompatibility.

D. There are normally many erythrocyteprecursors in the peripheral blood.

264. What is the corrected white blood cellcount expressed in SI units of X109/L?A. 4.6B. 12.5C. 16.7D. 18.4

Page 89: Hematology

REVIEW QUESTIONS • 317

Use the following information to answer

questions 269-271.

Use the following information to answerquestions 272-274.

A 75-year-old man with rheumatoid arthritiscomplains to his physician of pain and fatigue.

His CBC results are as follows: WBC 6.8 x 109/L(6.8 X 103/M,L); RBC 3.49 x 1012/L (3.49 x 106/n,L);

hemoglobin 97 g/L (9.7 g/dL); hematocrit 0.29

L/L (29%); MCV 83 fl_; MCHC 33.9 g/dL. Otherlaboratory results are as follows: serum iron

and total iron-binding capacity (TIBC) bothdecreased, serum ferritin slightly elevated.

269. If the serum iron is 22 jJig/dL and theTIBC is 150 |xg/dL, what is the percenttransferrin?A. 7%B. 10%C. 12%D. 15%

270. The results of the CBC and iron studies inthis case are most characteristic ofA. Beta-thalassemia minorB. Iron deficiencyC. Sideroblastic anemiaD. Anemia of chronic disease

271. Which of the following is not associatedwith the anemia described in question 270?A. Chronic gastrointestinal blood lossB. Hodgkin lymphomaC. TuberculosisD. Systemic lupus erythematosus

The peripheral blood shown in Color Plate 11 • isfrom a 19-year-old female college student who

has been living primarily on tea, beer, and cerealfor the past 9 months because she finds dining

hall food distasteful. She visits student healthcomplaining of fatigue. Her CBC results are asfollows: WBC 2.5 X 109/L (2.5 X 103/|JLL); RBC

2.10 X 1012/L (2.10 x 106/(xL); hemoglobin 85 g/L

(8.5 g/dL); hematocrit 0.24 L/L (24%); plateletcount 110x109/L (110,000/M.L); MCV 114 fL;

MCHC 35.0 g/dL; reticulocyte count 0.8%.

272. What test(s) should be done first todetermine a diagnosis in this patient?A. Vitamin B12 and Mate levelsB. Iron studiesC. Bone marrow examinationD. Osmotic fragility

273. In the absence of neurological symptoms,the anemia in this patient is most likelycaused by a lack ofA. An enzymeB. IronC. Folic acidD. Intrinsic factor

274. Which of the following is not a laboratoryfinding in this general classification ofanemia?A. Target cells and schistocytesB. Teardrop cells and macro-ovalocytesC. Howell-Jolly bodies and Cabot ringsD. Elevated serum LD and iron levels

Page 90: Hematology

318 CHAPTER 2: HEMATOLOGY

Use the following information to answer

questions 275-277.

Use the following information to answerquestions 278-280.

A 45-year-old Scandinavian woman with white

hair appears older than her age. She complainsto her physician of weakness, a tinglingsensation in her lower extremities, and

shortness of breath. Her CBC results are asfollows: WBC 3.4 x 109/L (3.4 x 103/|xL); RBC1.90 x 1012/L (1.90 x 106/|JLL); hemoglobin level

86 g/L (8.6 g/dL); hematocrit 0.25 L/L (25%);MCV 132 fl_; MCHC 34.4 g/dL; platelet count

100 x 109/L (100,000/|jiL). Cabot rings are noted

on the peripheral smear.

275. The clinical and laboratory findings aremost consistent withA. Liver diseaseB. Pernicious anemiaC. Folic acid deficiencyD. Aplastic anemia

276. Which of the following is not associatedwith this disorder?A. AlcoholismB. Antibodies to intrinsic factor or

parietal cellsC. Diphyllobothrium latum infectionD. Achlorhydria

277. Which of the following statements aboutmegaloblastic anemia is true?A. Oral folate therapy reverses the

neurologic symptoms of PA.B. Intramuscular injections of vitamin

B12 will reverse the neurologicsymptoms of PA.

C. Methotrexate (chemotherapeuticagent) is a vitamin B^ antagonist.

D. Folate deficiency takes years todevelop.

A 32-year-old African-American traveling to Africaon business had been healthy until he begantaking primaquine for prevention of malaria. Hewent to his physician because he felt faint and his

urine was black. His CBC results are as follows:WBC 6.5 X 109/L (6.5 X 103/^L); RBC 1.67 X1012/L (1.67 x 1 O%L|_); hemoglobin level 50 g/L

(5.0 g/dL); hematocrit 0.15 L7L (15%); MCV 89.8fL; MCHC 33.3 g/dL; platelet count 175 x 109/L

(175,000/|jiL); reticulocyte25.0%.

278. The most likely cause of this hemolyticepisode isA. G6PD deficiencyB. Hereditary spherocytosisC. Sickle cell diseaseD. Pyruvate kinase deficiency

279. The defect in this disorder is caused by anA. Amino acid substitutionB. Intrinsic red blood cell membrane defectC. Enzyme deficiency in the hexose

monophosphate shuntD. Enzyme deficiency in the

Embden-Meyerhof pathway

280. Inclusions that form when the patient isoxidatively challenged are composed ofA. RNAB. Denatured hemoglobinC. DNAD. Iron

Page 91: Hematology

REVIEW QUESTIONS • 319

Use the following information to answer

questions 281-283.

Use the following information to answer

questions 284-288.

A 15-month-old malnourished child is broughtto the clinic for a routine examination. Her CBC

results are as follows: WBC 9.5 x 109/L (9.5 x103/(xL); RBC 2.70 X 1012/L (2.70 X 10%iL);hemoglobin 67 g/L (6.7 g/dL); hematocrit

0.25 L/L (25%); MCV 73.5 fl_; MCHC 26.8 g/dL;

reticulocyte 0.2%; ROW 19%. Abnormal RBCmorphology present included pencil forms and

target cells.

281. What is this toddler's most probablediagnosis?A. Folic acid deficiencyB. Hereditary spherocytosisC. Iron deficiencyD. Erythroblastosis fetalis

282. The earliest indicator of this disease state isA. Decreased folic acidB. Decreased serum ironC. Decreased serum ferritinD. Increased bilirubin

283. What is the toddler's absolute reticulocytecount?A. 0.05X109/LB. 0.5X109/LC. 5X109/LD. 50X109/L

An 8-year-old girl is seen by the family physician.On physical examination, the physician notesfever, sore throat, bruising, petechiae, and pallor.

A CBC is drawn and the results are as follows:WBC 110x109/L (110X103/M-L); RBC 1.70 x

1012/L (1.70 x 106/|xL); hemoglobin 55 g/L (5.5

g/dL); hematocrit 0.16 L/L (16%); differentialcount shows 93% blasts and 7% lymphocytes.A bone marrow examination is performed and

reveals 85% blasts. All of the blasts are smallwith no variation in their appearance.

284. Which of the following would you expectto most accurately reflect the child'splatelet count?A. 10X109/LB. 100X109/LC. 200X109/LD. 400X109/L

285. What is this child's most probablediagnosis?A. Acute lymphoblastic leukemiaB. Acute myelogenous leukemiaC. Hairy cell leukemiaD. Myelodysplastic syndrome

286. Which of the following cytochemicalstains would most likely be positive in theblast cells of this patient?A. MyeloperoxidaseB. Leukocyte alkaline phosphataseC. Periodic acid-SchiffD. Nonspecific esterase

Page 92: Hematology

320 • CHAPTER 2: HEMATOLOGY

287. Terminal deoxyribonucleotidyl transferase(TdT) is present inA. Precursor B and precursor T lymphoid

cellsMature B and T lymphocytesPrecursor B cells and matureB lymphocytesPrecursor T cells and mature

B.C.

D.T lymphocytes

288. The presence of CD2, CD5, CD7 and theabsence of CD10 (CALLA) are associatedwithA. B lymphocytesB. T lymphocytesC. Myeloid cellsD. Monocytic cells

Use the following information to answer

questions 289-292.

The peripheral blood smear in Color Plate 1 ?•and the Sudan black B stain in Color Plate 19 •

are from a 90-year-old man complaining of fatigueand nosebleeds. The physician noted the patient

was febrile and had petechiae. CBC results wereas follows: WBC 20.0 x 109/L (20.0 x 103/(jLl_);RBC 2.58 x 1012/L (2.58 x 106/|xl_); hemoglobin

77 g/L (7.7 g/dL); hematocrit 0.24 L7L (24%);

platelet count 32 x 109/L (32,000/(jLl_); differential

count shows 75% blasts, 20% lymphocytes, and5% segmented neutrophils. A bone marrow

examination revealed 80% cellularity with 80%blasts. The blasts were myeloperoxidase andspecific esterase positive; nonspecific esterase

and PAS negative.

289. What is this patient's most likely diagnosis?A. Acute myelogenous leukemia without

maturation (FAB type Ml)B. Acute myelogenous leukemia with

maturation (FAB type M2)C. Acute monocytic leukemia

(FAB type M5)D. Myelodysplastic syndrome

290. Cytogenetic studies would most likelyshow which of the following chromosomeabnormalities?A. t(8;21)B. t(8;14)C. t(9;22)D. t(15;17)

291. Using World Health Organization (WHO)criteria for the diagnosis of acute leukemia,the percentage of bone marrow blasts mustbe at leastA. 5B. 20C. 30D. 50

Page 93: Hematology

REVIEW QUESTIONS • 321

292. Which of the following is not consideredan underlying condition that predisposes apatient to acute leukemia?A. Viral infectionsB. Bacterial infectionsC. Chronic bone marrow dysfunctionD. Congenital chromosome abnormalities

Use the following information to answerquestions 293-296.

An 83-year-old woman is seen in the emergency

department complaining of fatigue and recent

weight loss. Her CBC results are as follows:WBC 2.6 X 109/L (2.6 X 103/|xL); RBC 2.79 X

1012/L (2.79 x 106/|jiL); hemoglobin 92 g/L(9.2 g/dL); hematocrit 0.28 L/L (28%); MCV100.0 fl_; ROW 23.5%; platelet count 42 x 109/L(42,000/|xl_); differential count shows 42%segmented neutrophils, 45% band neutrophils,3% lymphocytes, 3% metamyelocytes, 4%myelocytes, 3% blasts, and 4 nRBC/100 WBC.Morphologic changes noted on the differential

smear include poor granulation and hyposeg-mentation of the neutrophils, giant platelets thatdisplay poor granulation, oval macrocytes,basophilic stippling, Cabot rings, Pappenheimer

bodies, and Howell-Jolly bodies. Threemicromegakaryocytes are seen per 100 WBCs.

Serum B12 and folate levels are normal.

294. The expected bone marrow findings in thisdisorder using WHO criteria areA. Hypocellular; blasts > 20%B. Hypocellular; blasts <20%C. Hypercellular; blasts > 20%D. Hypercellular; blasts <20%

295. If the bone marrow in this patient had 18%blasts, the most likely disorder would beA. Chronic myelomonocytic leukemia

(CMML)B. Chronic myelogenous leukemia (CML)C. Refractory anemia with ringed

sideroblasts (RARS)D. Refractory anemia with excess blasts

(RAEB)

296. Which of the following is a false statementabout myelodysplastic syndromes?A. MDS is "preleukemic" and frequently

terminates in acute leukemia.B. Treatment for MDS is only supportive

and not a cure.C. Median survival for all types of MDS

is 5 years.D. The lower the blast percent, the longer

is the survival rate.

293. The peripheral blood findings are mostconsistent withA. Myelodysplastic syndromeB. Degenerative left shiftC. Megaloblastic anemiaD. Chronic myelogenous leukemia

Page 94: Hematology

322 CHAPTER 2: HEMATOLOGY

Use the following information to answer

questions 297-300.

A 53-year-old man reported to the laboratory forroutine blood work as part of a yearly physical.

He had been feeling tired for the last few months.Physical examination revealed splenomegaly. HisCBC results are as follows: WBC 80.0 x 109/L(80.0 X 103/(juL); RBC 4.10x1012/L (4.10 X

10%,L); hemoglobin 123 g/L (12.3 g/dL);hematocrit 0.37 L7L (37.0%); platelet count 650 x

109/L (650,000/jjiL); differential count shows 40%polymorphonuclear neutrophils, 18% bands, 5%metamyelocytes, 7% myelocyte, 28% lympho-cytes, and 2% monocytes. No RBC or WBCmorphologic abnormalities are seen.

297. The peripheral blood findings are mostconsistent with a diagnosis ofA. Neutrophilic leukemoid reactionB. Chronic myelogenous leukemiaC. Acute myelogenous leukemiaD. Regenerative left shift

298. Which of the following would yield the mostdiagnostic information for this patient?A. Sudan black B (SBB)B. Periodic acid-Schiff (PAS)C. Tartrate-resistant acid phosphatase

(TRAP)D. Leukocyte alkaline phosphatase (LAP)

299. Which of the following myeloproliferativedisorders is characterized by the presenceof a t(9;22) chromosome abnormality andthe BCR/ABL oncogene?A. Polycythemia veraB. Essential thrombocythemiaC. Chronic myelogenous leukemiaD. Chronic idiopathic myelofibrosis

300. How does the presence of this chromo-some abnormality affect the prognosis?A. It is not a prognostic indicator.B. The prognosis is better when the

abnormality is present.C. The prognosis is better when the

abnormality is not present.D. Progression to acute lymphoblastic

leukemia occurs more often when theabnormality is present.

Page 95: Hematology

Ianswers

rationalesf̂e0^ ^fe^

Hematopoiesis

1.A. The need for oxygen delivery to developingtissues results in the production of erythrocytesbefore other blood cells. Erythropoiesis com-mences in the yolk sac as early as the fourteenthday of embryonic development. These primitivered cells produce embryonic hemoglobins thattemporarily serve oxygen needs of the fetus.Myelopoietic and lymphopoietic activities beginwhen the liver and spleen become sites of pro-duction at 6-9 weeks of gestation; however, ery-thropoiesis still predominates. At this time, thered cells produce hemoglobin F, which is thechief oxygen carrier during fetal life.

2.C. In the infant, there is an increased demand forblood formation because of the rate of growth. Atbirth, all bone marrow cavities are filled withhematopoietic tissue (active red marrow). As thegrowth rate slows, there is less need for activemarrow. Fatty infiltration of the marrow becomesnoticeable at about 4 years of age as cell produc-tion diminishes within the shafts of the long

bones and is filled with yellow inactive tissue. Fatcomprises 50% of the total marrow space in theadult. Except for lymphopoiesis, hematopoiesisis confined to the flat bones and pelvic area by theage of 25 years.

C. Hematopoietic stem cells can make copies ofthemselves to maintain the stem cell pool andpossess the ability to generate cells of all line-ages (pluripotential). These stem cells give rise tomultipotential myeloid and lymphoid progenitorcells, which ultimately produce progenitor cellsthat are restricted to a specific cell lineage. Withappropriate cytokine stimulus, the committed pro-genitor cells undergo proliferation to recognizableprecursors that produce an amplified number ofmature end-stage cells. Stem cells and progenitorcells cannot be morphologically distinguished(look similar to small lymphocytes) but canbe identified phenotypically by markers suchas the stem cell marker CD34. CD34 expressionis lost as antigens for a specific cell lineageare expressed. CD 13 is a marker expressed bymyeloid precursors.

323

Page 96: Hematology

324 CHAPTER 2: HEMATOLOGY

4.

D. Unlike the infant, in which all bone marrow iscapable of forming blood cells, the active marrowin an adult is confined to the flat bones of theskeleton such as the sternum and posterior iliaccrest. Although the spinous processes of the verte-brae contain active marrow, these sites are rarelyused for aspiration in adults because of the dangerof damage to the spinal cord. Sternal puncturealso presents a possibility of serious damage tounderlying structures, but this site may be usedbecause of easy accessibility or if the aspirate is a"dry tap" in the iliac crest. To obtain both a bonemaiTow aspirate and core biopsy, most marrowspecimens are taken from the posterior iliac crest.The anterior iliac crest may occasionally be usedin adults and sometimes the tibia in children lessthan 2 years of age.

5.C. The ratio between all granulocytes and theirprecursors and all nucleated red cell precursorsrepresents the myeloid-to-erythroid ratio. Myeloidprecursors outnumber erythroid precursors byabout 3 or 4 to 1 in the normal bone marrow.Although there are many more red blood cellsin the peripheral blood than granulocytes, redblood cells have a much longer life span in cir-culation (120 days) as compared to granulocytes(about 8 hours). Granulocytes, therefore, requirea more continual production than erythrocytesand are the most numerous marrow precursors.Alterations in the M:E ratio, such as 1:1 or 8:1,may indicate erythroid hyperplasia or granulo-cytic hyperplasia, respectively.

6.B. A diverse group of growth factors (cytokines)regulate and maintain hematopoiesis in a steadystate. Most hematopoietic growth factors are notlineage restricted but can act on more than one celltype and have multiple functions. For example,interleukins (IL-3) and colony stimulating factors

(GM-CSF) affect multiple cell lines; whereas ery-thropoietin action is limited to erythroid cells.Cytokines are glycoproteins that usually expressactivity by binding to specific receptors on targetcells. The action of growth factors on hematopoi-etic progenitor and precursor cells can stimulate orinhibit cell proliferation and differentiation as wellas promote or suppress cell death. Growth factorsmay act alone or together to exert a positive ornegative influence on hematopoiesis as well as onthe function of mature cells. A determining factorfor controlling the rate of cell production iscytokine stimulation in response to physiologicneed.

7.

A. The liver of the fetus assumes responsibilityfor hematopoiesis about the second month of ges-tation. From 3 to 6 months of fetal development,the spleen, thymus, and lymph nodes are alsoinvolved, but the principal site of hematopoiesisremains the liver. By the seventh gestationalmonth, the bone marrow becomes the primaryhematopoietic site. Around birth, the liver andspleen have ceased hematopoiesis (except forsplenic lymphopoiesis) but maintain the potentialfor reactivation of hematopoiesis.

8.B. Erythropoietin (EPO) is a hormone thatstimulates red cell production in the bone mar-row by its action on the committed RBC progen-itor cells. To maintain optimal erythrocyte massfor tissue oxygenation, the body's mechanismfor sensing tissue oxygen levels is located in thekidney. Erythropoietin production increaseswhen hypoxia is detected by renal oxygen sen-sors, with 90% being synthesized in the kidneyand 10% in the liver. EPO levels in the bloodvary according to the oxygen carrying capacityof the blood (e.g., EPO levels rise in anemia andfall when tissue oxygen levels return to normal).

Page 97: Hematology

ANSWERS & RATIONALES 325

9. 12.

D. The marrow-derived common lymphoid pro-genitor cell ultimately gives rise to lymphocytes ofT, B, or NK (presumably) cell lineages. Antigen-independent lymphopoiesis occurs in primarylymphoid tissue located in the thymus and bonemarrow. The formation of immunocompetent Tand B cells from precursor cells is influenced byenvironment (thymus, bone marrow) and severalinterleukins. Antigen-dependent lymphopoiesisoccurs in secondary lymphoid tissue (spleen,lymph nodes, Peyer's patches of the gastrointesti-nal tract) and begins with antigenic stimulation ofimmunocompetent cells.

10.B. Apoptosis is physiological cell death that canbe induced by deprivation of growth factors or pre-vented by growth-promoting cytokines. Apoptosisplays an important role in the regulation of cellnumber and is deregulated in certain malignancies.Necrosis is accidental cell death by phagocyticcells and is associated with lethal physical damage.Cellular senescence describes cells that have livedtheir life span and will die of old age. Terminal dif-ferentiation refers to mature end-stage cells that areno longer capable of replication.

11.A. Bone marrow consists of vessels, nerves,hematopoietic cells at various levels of matu-ration, and stromal cells encased in a mem-brane lining called the endosteum. The vascularsystem empties into a system of sinuses (venoussinusoids). A layer of endothelium lines these sinu-soids. Blood cell formation occurs in hematopoi-etic cords located outside of the sinusoids andbetween the trabeculae of spongy bone. The bonemarrow stroma (macrophages, adipocytes, fibrob-lasts, endothelial cells) forms an optimal microen-vironment for developing cells by providingsupport and secreting cytokines. Mature differen-tiated cells can deform to penetrate the vessel walland enter the sinuses and blood circulation.

B. Bone marrow cellularity in the normal adultis approximately 50% hematopoietic tissue and50% adipose tissue (fat), with a range of 30-70% cellularity being normocellular. Marrowcellularity is usually estimated from the corebiopsy. An intact bone marrow can respond todemand by increasing its activity several timesthe normal rate if sufficient supplies and growthfactors are available. The marrow becomeshypercellular when inactive fatty tissue isreplaced by active hematopoietic marrow. Incontrast, bone marrow failure may result inhypocellularity or aplasia with increased fat anda reduced number of hematopoietic cells.

13.C. Interleukins and colony stimulating factorsare cytokines produced by a variety of cells,including monocytes/macrophages, T lympho-cytes, fibroblasts, and endothelial cells. It isessential that cytokines are continuously sup-plied by cells present in the bone marrowmicroenvironment during hematopoietic celldevelopment, or cells will die. Erythropoietinfunctions as a true hormone because it is pro-duced by the kidney, released into the blood, andcarried to the bone marrow, where it stimulatesred cell production.

14.C. In a normal adult, the total blood volume isapproximately 12 pints or 6 liters. Cells accountfor about 45% (44% is red cell mass) and plasmaaccounts for 55%. Alterations in red cell mass orplasma volume are reflected in the RBC countand in measurements of hemoglobin and hemat-ocrit. True anemia or polycythemia is due to adecrease or increase in total RBC mass, respec-tively. A reduction in plasma volume with a nor-mal RBC mass may cause relative (pseudo)polycythemia. Conversely, an increase in plasmavolume with normal RBC mass may cause rela-tive (pseudo) anemia.

Page 98: Hematology

326 CHAPTER 2: HEMATOLOGY

15. 18.A. The pluripotent hematopoietic stem cell givesrise to lymphoid and myeloid progenitor cells. Thelymphoid progenitor produces cells destined tobecome lymphocytic cells, whereas the myeloidprogenitor cell produces progenitors committedto differentiation into granulocytic, erythrocytic,monocytic, or megakaryocytic lineages with appro-priate stimulus. The cells produced by progenitorcells can be demonstrated using in vitro culturetechniques; thus, the myeloid progenitor cell istermed CPU (colony forming unit)-GEMM basedon the cell colonies formed.

16.C. The mature megakaryocyte, the largesthematopoietic cell in normal bone marrow, hasa multilobed nucleus and abundant, granularcytoplasm. Plasma cells are characterized bya round, eccentric nucleus and intensely bluecytoplasm. Osteoblasts and osteoclasts are non-hematopoietic cells that may be present in nor-mal bone marrow. Osteoblasts are cells involvedin bone formation that resemble plasma cells butare larger and often found in groups. Osteoclastsreabsorb bone and are similar to megakary-ocytes in size but are multinucleated.

17.D. In normal adult marrow, about 50% is fat,40% is myeloid (granulocytic) cells, and 10% iserythroid cells. The M:E ratio is determined byperforming a differential count of marrow pre-cursor cells. The presence of 10% myeloblasts isan abnormal finding (reference range 0-2%),and a hematologic disease is likely. Megakary-ocytes should be seen when scanning and areusually reported as normal, increased, ordecreased in number. Marrow iron is assessedwith Perl's Prussian blue stain, and it is normalto see stainable iron in macrophages, as well asiron granules in the cytoplasm of developing redcell precursors.

B. The nucleus-to-cytoplasm ratio decreases asblood cell lines mature. With maturation, cellsgenerally become smaller, the nuclear chromatinbecomes clumpy and condensed, nucleoli disap-pear, and the cytoplasm loses its deep bluebasophilia when stained with Wright's stain.Exceptions include megakaryocytes (because ofendomitosis they grow larger as cytoplasm accu-mulates) and plasma cells (increased RNA andprotein synthesis produces a deep basophilia).

19.C. Thrombopoietin (TPO) is the major regula-tor of platelet production in the bone marrow byits action on committed progenitor and precursorcells of the megakaryocytic line. It is primarilyproduced by hepatocytes and possibly by thekidney. After marrow release, about 70% ofplatelets are in the blood circulation and 30% aresequestered in the spleen. Unlike erythropoietin,which is manufactured for routine therapeuticuse, recombinant TPO is still being evaluated.

20.B. Hematopoiesis within the medulla or innerpart of the bone marrow is termed medullary ormyeloid. Hematopoiesis that occurs in the liverand spleen (reactivation of fetal life) is calledextramedullary or myeloid metaplasia (organsmay enlarge). Cell production outside of the mar-row space takes place when the bone marrow isunable to meet its production demands. This mayoccur in severe hemolytic anemias when the max-imal capacity of the bone marrow to increaseactivity is exceeded. Myeloid metaplasia mayalso be an extension of a disease process suchas myelofibrosis. Myelophthisis refers to thereplacement of normal marrow hematopoietic tis-sue by fibrotic tissue or cancer cells, whereasmyelodysplasia describes abnormal maturationof erythrocytic, granulocytic, and/or megakary-ocytic cell lines. The period of intrauterine lifewhen cell production occurs in the yolk sac maybe termed mesoblastic.

Page 99: Hematology

ANSWERS & RATIONALES • 327

Erythrocytes 24.

21.D. Normal red blood cells survive about 4months, or 120 days. The entire life span of the redcell is spent inside the vascular tree, making iteasier to determine the rate of production anddestruction. Red cell survival depends upon anintact RBC membrane, sufficient cellular energy,and normal hemoglobin function. As red cells cir-culate for 120 days, enzymes are depleted and theability to deform decreases. Under normal condi-tions, red cell loss due to aging (-1%) is equal todaily replacement. Most destruction of aged redcells occurs extravascularly by macrophages ofthe reticuloendothelial system (spleen, liver).

22.B. The erythrocyte has a semipermeable mem-brane that allows water and some anions, such aschloride (CF) and bicarbonate HCOs , to enterthe cell rapidly. Sodium ions (Na+) enter the celland potassium ions (K+) leave the cell slowly butcontinuously. In order to maintain a high intra-cellular K+ concentration and remove excessNa+, ATP-dependent cationic pumps expel Na+

and take in K+. This regulation of intracellularcations allows the red cell to control its volumeand water content.

23.D. A molecule of hemoglobin is composed offour globular, protein subunits, and each subunitcontains a heme group bound within a convo-luted globin chain. Heme groups are identicaland consist of protoporphyrin IX with a centraliron atom, made largely in the mitochondria.Amino acids are sequenced on ribosomes to pro-duce four types of globin chains (alpha, beta,delta, and gamma) that combine in identicalpairs. A normal hemoglobin molecule consistsof two alpha-globin chains and two non-alpha-globin chains, each of which binds a hemegroup. The different globin chains determine thehemoglobin type (A, A2, or F).

D. Culling is the process of removing aged orabnormal red blood cells from the circulation bythe spleen. Red cells (7 (Jim) enter the spleenthrough the splenic artery and must squeezeback into active circulation through 2- to 4-jxmclefts in the venous sinusoids. Aged or abnor-mally shaped red cells with impaired membraneflexibility are trapped in the splenic microcircu-lation and ingested by macrophages. The spleenis the largest filter of blood in the body and has anessential role in the "quality control" of red cells.

25.C. Each hemoglobin molecule has four hemegroups located at its surface, and oxygen binds tothe central ferrous iron (Fe2+) in heme. Deoxyhe-moglobin (not carrying O2) and oxyhemoglobin(carrying up to four O2) are normal physiologicforms of hemoglobin with iron in the ferrousstate. Hemoglobin in which the ferrous iron

r\ *""̂ Q_l

(Fe ) has been oxidized to the ferric state (Fe )is known as methemoglobin and is unable tocarry O2. Carboxyhemoglobin is hemoglobinwith carbon monoxide (CO) attached to ferrousiron rather than O2. Both methemoglobin andCarboxyhemoglobin are reversible.

26.C. Of the total body iron present in a normaladult, approximately 70% is contained in hemo-globin (in red cells of the blood and marrow).Most of the remainder, -25%, is found in stor-age sites as ferritin or hemosiderin. A muchsmaller amount of iron is contained in musclemyoglobin (4%) and respiratory enzymes suchas peroxidase (1%). The structures of hemoglo-bin and myoglobin are similar (both consist ofglobin and heme), but myoglobin functions as anoxygen trap in the tissues.

Page 100: Hematology

CHAPTER 2: HEMATOLOGY

27. 30.

B. A senescent red blood cell is one that haslived its life span. Repeated passes through thespleen deplete the cells of glucose and decreasetheir surface area as membrane lipids are lost.The red cells are removed from the circulationby splenic macrophages that recognize subtleabnormalities in these cells, sequester them, anddestroy them.

28.B. "Poikilocytosis" is a general term that refersto deviations from the normal red cell shape(biconcave, discoid). "Anisocytosis" is the termused when differences in the sizes of red cellsare described. Color in red cells is designated asnormochromic (normal) or hypochromic (indi-cating a decreased hemoglobin concentration).Abnormally shaped red cells and red cell inclu-sions are associated with rigid red cells that havereduced deformability and shortened survival.

29.A. Howell-Jolly bodies are nuclear (DNA) rem-nants that remain in the red cell after the nucleushas been extruded and may represent nuclearinstability. These inclusions are associated withthe defective nuclear maturation found in mega-loblastic anemias and the rapid cell division thatoccurs in severe hemolytic anemias. Under nor-mal circumstances, the spleen effectively pitsthese bodies from the cell. Pitting is a processthat removes inclusions while leaving the restof the red cell intact. It may be that the pittingmechanism is overwhelmed and cannot keeppace with inclusion formation in hemolytic ane-mias. Howell-Jolly bodies can also be seen inindividuals after splenectomy who lack the nor-mal pitting function.

A. Spherocytes appear smaller and moredensely staining than normal red cells and lacka central pallor area. Because they are the resultof membrane loss, their surface area-to-volumeratio is decreased. Spherocytes should be dis-tinguished from acanthocytes, which also lack apallor area but have sharp, irregular projections.Echinocytes have a central pallor area and blunt,short projections. Red cells with intracellularrod- or bar-shaped crystals contain hemoglobinC crystals.

3LB. The presence of lead causes an inhibition ofseveral of the enzymes important in heme synthe-sis. Among these is pyrimidine 5'-nucleotidase,which is normally responsible for degradation ofribosomal ribonucleic acid (RNA). The lack of thisenzyme apparently allows aggregates of incom-pletely degraded RNA to remain in the cell cyto-plasm. It is this ribosomal material that appearson Wright's stain as punctate basophilic stippling.Precipitated hemoglobin forms Heinz bodies (notvisible with Wright's stain), nuclear fragments arecalled Howell-Jolly bodies, and iron deposits arePappenheimer bodies.

32.D. The presence of red cell rouleaux (coiningpattern) is a characteristic finding in multiplemyeloma because of the increased concentra-tion of immunoglobulins in the blood plasma(hypergammaglobulinemia). The excessive immu-noglobulins are produced by malignant plasmacells. Cold hemagglutinin disease is character-ized by red cell agglutination or clumping in anonspecific pattern. "Hypersplenism" refers toan enlarged, overactive spleen that destroys bothnormal and abnormal cells, possibly causingpancytopenia.

Page 101: Hematology

ANSWERS & RATIONALES • 329

33. 36.C. In viewing Color Plate 1 B, the inclusions inthe red blood cells are Howell-Jolly bodies. Dur-ing passage through the microvessels of thespleen, the red cell is examined for intracellularinclusions or membrane-bound antibodies, which,if present, are removed. Abnormal red cells circu-late longer, and inclusions such as Howell-Jollybodies or Pappenheimer bodies will be seen post-splenectomy (or in conditions with splenic atro-phy). The phagocytic removal of abnormal redcells is assumed by the liver, but the liver is not asefficient as the spleen. Howell-Jolly bodies are notassociated with iron deficient or iron overloadstates.

C. A "shift to the left" in the oxygen dissocia-tion curve of hemoglobin means that a higherpercentage of hemoglobin will retain more of itsoxygen at a given pressure. Thus affinity will begreater and oxygen delivery will be reduced. Ahigher or more alkaline pH and a lower tempera-ture are associated with decreased oxygen disso-ciation. With conditions in the lungs (increasedpH, decreased 2,3-BPG, decreased temperature),hemoglobin affinity for oxygen is increased,which favors oxygen uptake. With conditions inthe tissues (decreased pH, increased 2,3-BPG,increased temperature), hemoglobin affinity foroxygen is decreased, which favors release ofoxygen to the tissues.

C. One of the reasons for increased intestinalabsorption of iron is an accelerated rate of ery-thropoiesis (another is depletion of iron stores).Although the mucosal cell does act as a barrierin normal circumstances, this function is notabsolute and controls break down in the pres-ence of large amounts of iron, causing an excessto be absorbed. An acid pH is required for ironabsoiption, and sites of maximal absorption arethe duodenum and upper jejunum. The body hasno effective means for iron excretion.

35.D. The presence of iron granules or deposits canbe detected with Perl's Prussian blue iron stain.Siderocytes are mature red blood cells thatcontain stainable iron granules (abnormal). Sider-oblasts are bone marrow nucleated red cells (nor-moblasts) that contain small amounts of iron inthe cytoplasm (normal). Ringed sideroblasts aremarrow normoblasts that contain iron in the mito-chondria that forms a ring around the nucleus(abnormal). Siderocytes and ringed sideroblastsare associated with iron overload problems, par-ticularly sideroblastic anemia. Reticulocytes maycontain small amounts of unused iron that is nor-mally removed by the spleen.

37.

D. Erythropoietin (EPO) is a hormone producedby the kidney that increases erythropoiesis in thebone marrow in response to tissue hypoxia. TheCFU-E (colony-forming unit-erythroid) is a com-mitted erythroid progenitor cell with many recep-tors for erythropoietin. EPO stimulation of theCFU-E produces the recognizable pronormoblastand promotes differentiation of RBC precursors.The maturation time of erythrocyte precursors(5-7 days) can be reduced in times of increasedneed for red cells by the action of erythropoietin.

38.C. Increased binding of 2,3-BPG (2,3-bisphos-phoglycerate) decreases the affinity of hemoglo-bin for oxygen, which promotes oxygen releaseto the tissues, a compensatory mechanism inanemic patients. Increased pH (alkalinity)enhances oxygen affinity and thus inhibits deliv-ery to the tissues. Less oxygen is available athigher altitudes, and this affects blood saturationand delivery to tissues. An increase in erythro-poietin release will affect red cell production butdoes not have an immediate or direct impact onoxygen delivery.

Page 102: Hematology

330 • CHAPTER 2: HEMATOLOGY

39.A. After the nucleus is extruded, reticulocytesspend about 2 days in the bone marrow beforerelease into the blood, where maturation contin-ues for another day. Intense erythropoietin stim-ulus can cause early release of bone marrowreticulocytes. These reticulocytes are larger andcontain more filamentous reticulum than a moremature reticulocyte. These "shift" or "stress"reticulocytes exhibit diffuse basophilia on theWright's stained smear and will need more thanthe usual 1 day in circulation to mature (to loseRNA). A very high number of reticulocytes inthe blood circulation can increase the MCV. Thelevel of reticulocyte maturity is best assessed bythe immature reticulocyte fraction (IRF), anindex reported by automated cell counters that isbased on RNA content.

40.C. Heinz bodies do not stain with Wright's stainand appear as "normal" hemoglobin even thoughtheir presence causes cell rigidity and membranedamage. They can be visualized on wet prepswith phase microscopy or by using supravitalstains, such as crystal violet or brilliant green.Heinz bodies consist of intracellular globin orhemoglobin precipitate that results from hemo-globin denaturation (G6PD deficiency, unstablehemoglobin valiants) or excess globin chains(certain thalassemic syndromes). Basophilic stip-pling, Cabot rings, and Pappenheimer bodies arevisible with both Wright's and supravital stains.

anemias that result from defective bone marrowdelivery of red cells to the blood.

42.C. Tissue hypoxia associated with low erythro-cyte and hemoglobin levels causes increasedrenal release of erythropoietin to stimulate bonemarrow erythropoiesis. Depending on severity,the bone marrow responds by increasing itsactivity 6-8 times normal and becomes hyper-cellular because of an increase in RBC precur-sors (erythroid hyperplasia); and the M:E ratiofalls. Nucleated red cells may be released intothe blood along with the outpouring of reticulo-cytes. The number of nucleated red cells tends tocorrelate with anemia severity.

43.C. Millions of hemoglobin molecules are pro-duced in the red cell cytoplasm during matura-tion. When developing erythroid cells aredeprived of essential hemoglobin components,the result is the production of microcytic,hypochromic red cells. It is thought that duringmaturation, extra cell divisions occur until acertain hemoglobin concentration is reached.Impaired hemoglobin synthesis may be theresult of heme defects (involving iron or proto-porphyrin) or may be caused by globin defects.Impaired DNA synthesis is associated withmacrocytic red cells, and normocytic red cellsare characteristic of enzyme defects.

4LB. The presence of schistocytes (schizocytes) onthe smear indicates that red cells have been sub-jected to some form of physical trauma thatcauses damage. Red cell fragmentation can be theresult of impact with fibrin strands, mechanicaltrauma by artifical surfaces, injury by heat, partialphagocytosis, or damage by toxins and drugs.Schistocytes are characteristic of the increasedred blood cell destruction that occurs in severehemolytic anemias but are not associated with

44.C. The nucleated red cells seen in Color Plate 2 •would be staged as orthochromic normoblasts(metarubricytes) when in the bone marrow. This isthe last stage of red cell maturation that contains anucleus. The pyknotic, degenerated nucleus is nor-mally extruded out of the red cell in the marrow toyield the anucleate reticulocyte. The release ofnucleated red cells into the blood before reachingmaturity usually indicates a high demand for redcells.

Page 103: Hematology

ANSWERS & RATIONALES • 331

45. 48.

B. The red cell membrane consists of a proteinshell heavily coated with lipids. The membranelipid bilayer is maintained by constant inter-change with plasma lipids. Acanthocytes are theresult of abnormal plasma lipids that havealtered the lipid composition of the membrane,often involving increased cholesterol content.Acanthocytes (spur cells) are associated with acongenital form of acanthocytosis and with liverdisease, or are seen following splenectomy.

46.A. The mature red cell, which lacks mitochondriaand Kreb's cycle activity, depends on glucosemetabolism for cellular energy. The end product ofthe anaerobic Embden-Meyerhof pathway (BMP)is ATP, which is necessary for membrane mainte-nance and volume control (cation pumps). Thehexose monophosphate (HMP) pathway is aero-bic and reduces oxidants by providing NADPHand glutathione. The Rapoport-Luebering shuntcontrols the amount of 2,3-bisphosphoglyceratethat regulates hemoglobin affinity for oxygen. Oxi-dized hemoglobin (methemoglobin) is reduced tofunctional hemoglobin by the methemoglobinreductase pathway.

47.C. The polychromatophilic normoblast (rubri-cyte) is the last red cell stage capable of mitosis.With cellular divisions, each pronormoblastproduces up to 16 erythrocytes. The polychro-matophilic normoblast is also the stage in whichhemoglobin is first visible. The gray-blue colorof the cytoplasm when Wright's stained is due toa mixture of hemoglobin and RNA, hence thename "polychromatophilic." The reticulocyte isthe last stage able to synthesize hemoglobin.

A. The major adult hemoglobin, Hb A, consistsof two alpha- and two beta-globin chains. Theswitch from gamma chains (Hb F) to beta chainsoccurs 3-6 months after birth, and Hb A reachesadult levels (about 97%) around 6 months of age.Most globin chains produced in a normal adultare alpha and beta types (1:1 ratio) for hemoglo-bin A production. Hemoglobin A2 contains deltachains and comprises about 2% of hemoglobinin normal adults. Epsilon chains are found inearly embryonic hemoglobins only.

49.C. Impaired DNA synthesis results in nuclearmaturation that lags behind cytoplasmic devel-opment (asynchrony), decreased cellular divi-sions, and the production of macrocytic redcells. Defective nuclear maturation (megaloblas-tic) is almost always caused by a deficiencyof vitamin B12 or folic acid, which are DNAcoenzymes. Macrocytic red cells that are not dueto vitamin B12 or folic acid deficiency (non-megaloblastic) may be seen in liver disease orwhen reticulocytosis is pronounced. Microcytic,hypochromic red cells are the result of impairedhemoglobin synthesis.

5(K

A. When iron is removed from the heme ofdestroyed red blood cells, it is bound to transfer-rin and recycled for hemoglobin production orgoes to storage. The major storage form of ironis ferritin, which is a water-soluble iron complexbound in a protein shell called apoferritin.Hemosiderin is a water-insoluble complex ofiron aggregates and protein that is derived fromferritin. The main site of iron stores is the liver,but storage iron is also found in the bone marrowand spleen.

Page 104: Hematology

332 • CHAPTER 2: HEMATOLOGY

51.

B. RBC indices are average values, so they haveless meaning in heterogeneous RBC populationswith wide size variations. Because the MCV is amean red cell volume measurement, the presenceof both microcytes and macrocytes would yield afalsely normal MCV value. One would expectthe RDW (red blood cell distribution width) to behigh because it is an index of variation in red cellsize or anisocytosis. The RDW is low when redcells are of uniform size (a homogeneous popula-tion). The RDW is high when a heterogeneouspopulation of red cells is present.

52.B. Bilirubin is formed when hemoglobin degra-dation occurs in the reticuloendothelial system,primarily in the spleen. Unconjugated bilirubinis transported by the plasma to the liver, where itis conjugated. When excessive extravascular redcell destruction occurs, the plasma bilirubinlevel rises and is largely unconjugated bilirubin.When acute intravascular red cell destructionoccurs, hemoglobin is released into the plasmaand findings may include hemoglobinemia,hemoglobinuria, and hemosiderinuria.

53.D. Haptoglobin forms a 1:1 complex withalpha-beta dimers of hemoglobin. The large sizeof this complex prevents filtration of the hemo-globin through the kidneys, where it can causerenal damage. Haptoglobin can be depleted inthe plasma during major hemolytic events, suchas malarial attacks, transfusion reactions, andother causes of severe intravascular red celldestruction.

54.C. Codocytes (target cells) have an increased sur-face area-to-volume ratio and are associated withabnormal hemoglobin synthesis. They are foundin hemoglobinopathies, especially hemoglobin S

or C disorders, as well as thalassemias and irondeficiency. Target cells can also result from anincrease in membrane lipids and may be seen inliver disease. Discocytes are normal biconcave redcells, and elliptocytes (ovalocytes) can be found invarying sizes. The teardrop shape of daciyocytesmay occur when a red cell is stretched in thespleen and cannot regain its original shape.

55.B. Heme synthesis begins in the mitochondriawith the formation of aminolevulinic acid. For-mation of the pyrrole ring structure occurs in thecytoplasm, resulting in the synthesis of copro-porphyrinogen III. The final stages of porphyrinsynthesis occur again in the mitochondria, cul-minating in the formation of heme when ferrousiron is incorporated into protoporphyrin IX inthe presence of ferrochelatase.

56.

A. The red cell membrane consists of an outerbilayer of lipids with embedded, integral pro-teins and an underlying skeleton. Spectrin is thepredominant skeletal protein that forms acytoskeleton with other proteins, such as actin,protein 4.1, and ankyrin. The skeletal proteinsare responsible for cell shape, deformability, andstability. Any defect in structure or extensivedamage to the membrane cannot be repaired andmay lead to premature red cell death.

57.

C. Reduced glutathione (GSH) counteracts oxi-dants that accumulate in the red cell. Theseoccur as a result of normal metabolic activitiesand increase during infections or as a result oftreatment by certain drugs. In the absence ofGSH or as a result of enzyme deficiencies in thehexose monophosphate pathway (HMP), oxi-dant accumulation can lead to oxidation and pre-cipitation of hemoglobin.

Page 105: Hematology

ANSWERS & RATIONALES

58.C. Total iron-binding capacity (TIBC) repre-sents the amount of iron that circulating transfer-rin could bind when fully saturated. In this test,the amount of transferrin protein in the serum isindirectly measured by adding feme (Fe3+) ironto the serum and allowing it to bind to the unsat-urated sites on transferrin. Unbound iron is thenremoved and the sample analyzed for theremaining iron that is bound to transferrin. Theserum iron level measures iron bound to trans-ferrin. Under normal conditions, about one-thirdof the binding sites on transferrin are occupiedwith iron.

59.B. The amount of circulating ferritin indirectlyreflects the amount of storage iron in the tissues.A bone marrow exam is not essential to assessiron stores, except in complicated cases, becausethe serum ferritin test is considered a good indi-cator of iron storage status in most individuals.Because ferritin is an acute-phase reactant, itmay be increased in chronic inflammatory disor-ders regardless of iron stores. Therefore, theserum ferritin should be interpreted with otheriron tests. The percent transferrin saturation isthe serum iron divided by the serum TIBC.

60.B. Fetal hemoglobin can be distinguished fromadult hemoglobin in red blood cells by the acidelution technique of Kleihauer and Betke. Onlyhemoglobin F remains in red blood cells afterexposure to a citric acid-phosphate buffer solu-tion at pH 3.3. Hb F has a higher oxygen affinitythan Hb A (less binding of 2,3-BPG), so it car-ries oxygen well in utero. Hemoglobin F pro-duction decreases after birth, composing lessthan 1% of total hemoglobin in normal adults. Incertain conditions (thalassemias, hemoglo-binopathies), defective beta-chain productioncan be compensated for by increased production

of gamma chains and formation of hemoglobinF (two alpha and two gamma chains).

Erythrocyte Disorders

61.D. Deficiencies of folic acid (folate) and vita-min B12 result in abnormal DNA synthesis and aresultant delay in nuclear maturation in compar-ison to cytoplasmic development. These ane-mias are categorized as megaloblastic becauseof the giant red cell precursors observed in thebone marrow. The other anemias are character-ized by defects of heme (sideroblastic anemiaand iron-deficiency anemia) or globin synthesis(hemoglobin C disease).

62.

A. G6PD deficiency has a sex-linked inheritancepattern and is the most common enzyme defi-ciency in the hexose monophosphate (HMP)shunt. Individuals are asymptomatic unlessexposed to oxidants, which compromise the abil-ity of the glutathione reduction pathway to pre-vent the oxidation of hemoglobin. The oxidizedhemoglobin precipitates in the form of Heinzbodies, which cause acute intravascular hemoly-sis. In the most common G6PD variant, thehemolytic episode is self-limiting, with old redcells that lack enzyme being destroyed and youngred cells with some enzyme activity unaffected.

63. ___B. Hemoglobin nomenclature indicates a numberof things. The symbol a2 or a2A indicates thepresence of normal adult, or A, alpha chains. Thedesignation (3226 Glu~>Lys indicates that lysineresidues have replaced glutamic acid on position26 of the beta chains. All types of E hemoglobinshow a similar electrophoretic mobility andmigrate closely to hemoglobins C and A2 on cellu-lose acetate (alkaline pH). Hemoglobin E occurswith the greatest frequency in Southeast Asia.

Page 106: Hematology

I334 • CHAPTER 2: HEMATOLOGY

64. 66.C. The peripheral blood as seen in Color Plate 3mshows numerous elliptocytes (ovalocytes). Ifthey were artifact due to smear preparation, theywould be oriented in the same direction. Heredi-tary elliptocytosis (HE) is associated with symp-tomatic hemolytic anemia in only about 10-15%of the cases, but the presence of an enlargedspleen is evidence of ongoing extravasculardestruction. In patients with chronic hemolysis,gallstones are a common complication becauseof excess bilirubin catabolism. In most personswith HE, anemia does not develop because bonemarrow production of red cells compensates forthe mild shortening of red cell life span.

65.B. Cooley anemia, or beta-thalassemia major,would be the appropriate diagnosis in this case.In this condition, two beta-thalassemia genes areinherited that result in virtually no hemoglobinA production because no beta-globin chains areproduced. The primary hemoglobin made ishemoglobin F. The severe microcytic anemiaresults from the destruction of red cell precur-sors in the bone marrow (ineffective erythro-poiesis) and rigid red cells in the blood thatcontain unused alpha-globin chains. Nucleatedred blood cells and target cells, as seen in ColorPlate 4B, are common, as well as basophilic stip-pling. Infants with alpha-thalassemia major diein utero or shortly after birth. Hemoglobin H dis-ease (three-gene deletion alpha-thalassemia)results from deficient, alpha-chain synthesis thatleads to production of Hb H (four beta chains),an unstable hemoglobin that forms Heinz bodiesand causes chronic hemolysis. No clinical mani-festations are seen in patients with hereditarypersistence of fetal hemoglobin (HPFH).

C. As seen in Color Plate 5B, the presence ofnumerous target cells and SC crystals on theperipheral blood smear suggests the presence ofhemoglobin SC disease. These bizarre crystalsare distinguished by one or more blunt, finger-like projections that protrude from the cell mem-brane. Clinically, hemoglobin SC disease is notusually as severe as sickle cell disease, and elec-trophoresis shows equal amounts of Hb S andHb C. Codocytes, in varying numbers, are typi-cal of hemoglobin S, C, and E disorders.

67.C. Pica is a clinical finding seen in somepatients with iron deficiency. Pica is unusualcravings for nonfood items that may include dirt,clay, laundry starch, or, most commonly, ice.Among some cultures, pica is a custom (eatingdirt) that may contribute to iron deficiency. Inchildren, lead poisoning often results from theingestion of dirt or lead-based paint from toysand may be related to iron deficiency. Porphyriasare a group of inherited disorders characterizedby enzyme deficiencies and abnormal porphyrinmetabolism. The presence of pyridoxine (pyri-doxal-5'-phosphate or vitamin Bg) is importantto early porphyrin synthesis.

68.B. Folate deficiency is most commonly a resultof poor dietary intake of folate alone or in com-bination with increased requirements as duringpregnancy. Daily requirements for folate arehigh, and depletion of folate stores can occurwithin 4 months as compared to vitamin B12, inwhich deficiency takes at least 2 years to develop(there are high stores). Thus, dietary deficiencyof vitamin B12 is rare, but folate supplements arecommonly required during pregnancy or inhemolytic anemias with excess cell turnover.

Page 107: Hematology

ANSWERS & RATIONALES • 335

I

69. 72.

C. The hemoglobin solubility test can detect thepresence of hemoglobin S, which is insoluble inthe dithionite reagent, whereas normal hemoglo-bin A is soluble. A positive screening test, how-ever, does not distinguish between patients withhemoglobin AS trait, hemoglobin SC disease, andhemoglobin SS disease, so results must be con-firmed by electrophoresis. Sickle cell trait is clini-cally asymptomatic with target cells only.Disorders prevalent in the malarial belt (sickle celltrait, G6PD deficiency, hereditary ovalocytosis,thalassemia minor) are thought to impart resis-tance to falciparum malaria. Repeated splenicinfarctions by sickle cell masses in hemoglobin SSdisease cause autosplenectomy by adulthood.

70.D. The schistocytes in Color Plate 6u are foundin microangiopathic hemolytic anemia and causedby red cells shearing on fibrin strands depositedin small vessels. Widespread or localized (e.g.,kidney) fibrin deposition in DIG, HUS, and TTPresults in red cell fragmentation. In addition,thrombocytopenia is a usual feature of MAHA.ITP is characterized by severe thrombocytope-nia that results from destruction of platelets byautoantibodies, but it is not associated with redcell damage or anemia.

71.C. Ringed sideroblasts result from the accumu-lation of iron deposits in the mitochondria sur-rounding the nucleus of erythroid precursors.The deposits are secondary to a defect in hemesynthesis and a pathological finding in siderob-lastic anemia. Blocks in the protoporphyrinpathway required for heme synthesis may behereditary (rare) or acquired and result in ironoverload with increased marrow iron. Pappen-heimer bodies and basophilic stippling are fre-quent findings on the blood smear, and increasedserum iron, decreased TIBC, increased percenttransferrin saturation, and increased serum fer-ritin are usual.

D. Reticulocytosis is indicative of increasederythropoietic activity by the bone marrow. Thisis a normal response in conditions involving pre-mature red cell destruction in the circulation orfollowing blood loss due to acute hemorrhage.The reticulocyte count is consistently increasedin active hemolytic disease because the marrowspeeds up red cell production to supply replace-ment cells. Anemia develops when the rate ofred cell destruction exceeds the marrow's abilityto replace red cells (uncompensated hemolyticdisease). The reticulocyte count is not usuallyelevated in pernicious anemia even thoughincreased marrow erythropoiesis occurs. Thedefective cellular maturation that occurs inmegaloblastic anemias results in the death ofmany red cells in the bone marrow (ineffectiveerythropoiesis).

73.B. The major defect in hereditary stomatocyto-sis is altered permeability of the red cell mem-brane to Na+ and K+ ions. A net gain of sodiumwithin the cell leads to increased water entry andthe appearance of a swollen cell with a slit-likearea of pallor. This is a heterogeneous group ofdisorders, in that a number of specific membranedefects have been postulated, and anemia variesfrom mild to severe. One autosomal dominantdisorder is associated with Rh-null individuals.

74.D. Polycythemia vera (PV) belongs to thegroup of disorders that are hematopoietic stemcell defects and commonly characterized asmyeloproliferative disorders. Although themajor increase in PV is in red blood cells, thereis also an overproduction of granulocytes andplatelets, particularly in the early stages of thedisease. The increased production of red cells inPV is not due to the activity of erythropoietin.The production of erythropoietin is almost com-pletely suppressed in this malignant condition.

Page 108: Hematology

CHAPTER 2: HEMATOLOGY

75.

A. Aplastic anemia can be defined as blood pan-cytopenia resulting from bone marrow failure.This stem cell disorder results in a hypocellularmarrow with few developing precursors anddecreased production of all cell lines. The ane-mia is generally normocytic or slightly macro-cytic with reticulocytopenia. The "defect" alsoaffects resting hematopoietic cells in the liver andspleen, so extramedullary hematopoiesis doesnot occur to compensate for marrow failure.

76.A. Because red blood cells and plasma are losttogether, the hemoglobin and hematocrit will notreflect the severity of an acute hemorrhage untilthe lost blood volume begins to be replaced bythe formation of plasma. The restoration of anormal blood volume is usually complete by 24hours. It is then that the hemoglobin and hemat-ocrit will reach their lowest point and will beginto rise only with the release of newly formed redcells, usually within 3-4 days.

77.D. Elevated RBC, hemoglobin, and hematocritvalues in a newborn are a carryover fromintrauterine life, when a high number of red cellswere needed to carry oxygen. Erythropoiesis issuppressed in response to the marked increase inoxygenation of tissues after birth, and the reticu-locyte count, which is initially high, will fallalong with a slow decline in the hemoglobinlevel. A hemoglobin value below 140 g/L (14.0g/dL) is abnormal for a neonate. Newborn redcells are macrocytic and up to 10 nucleated redcells per differential may be seen.

78.D. The red blood cells with single elongatedprojections, seen in Color Plate ?•, are dacry-ocytes or teardrops. Dacryocytes are often seenin disorders of marrow replacement that affectbone marrow architecture, especially myelofi-brosis. Teardrops can also result from the splenic

removal of inclusions and may be present in avariety of anemias. Drepanocytes or sickle cellsare observed during a sickling crisis of sicklecell anemia. Acanthocytes, echinocytes, and/orcodocytes can be found in liver disease (pres-ence varies with disease severity).

79.C. Myelophthisic anemia is an anemia of bonemarrow failure. It is seen in patients who areexperiencing bone marrow replacement of nor-mal hematopoietic tissue by metastatic cancercells, fibrosis, or leukemia. The anemia is consid-ered a hypoproliferative anemia because there isno hemolysis involved and the cells are normo-cytic, normochromic. Disruption of the bone mar-row by abnormal cells can result in the release ofimmature cells (nucleated red cells and immatureneutrophils) into the blood and may involve bloodcell production in extramedullary sites.

80.A. Any idiopathic disorder is one for whichthere is no apparent cause. Ionizing radiation is awell-known cause of aplasia, as is chemicalexposure (pesticides, benzene). latrogenic disor-ders are those that result from treatments for adifferent disorder; for example, aplasia canresult from chloramphenicol treatment for bac-terial disease. Aplastic anemia may develop as acomplication from infections such as Epstein-Barr or hepatitis viruses.

81.C. Diamond-Blackfan anemia is a congenitaldisorder that depresses only red blood cell pro-duction. Fanconi anemia is a congenital form ofaplastic anemia that results in aplasia of all celllines and has a high risk of developing acutemyeloid leukemia or other cancers. The bonemarrow distinguishes Diamond-Blackfan fromthe hypocellular marrow seen in aplastic anemiabecause there is a lack of erythroid precursorsbut a normal number of myeloid and megakary-ocytic precursor cells.

Page 109: Hematology

ANSWERS & RATIONALES • 337

82.C. Dehydration is a cause of relative (pseudo)erythrocytosis due to plasma loss. High altitudeadjustment, cardiac or pulmonary disease, anddefective oxygen transport are all causes ofabsolute secondary erythrocytosis. Secondaryerythrocytosis (polycythemia) is a compensa-tory increase in red cells, produced in an attemptto increase the amount of oxygen available to thetissues.

83.B. In infants with homozygous alpha-thalassemia, no alpha-globin chains are produced(because of the deletion of all four alpha genes).Consequently, the infants have nearly 100%hemoglobin Bart's, which consists of fourgamma-globin chains. This hemoglobin migratesfarther toward the anode than Hb A. Because HbBart's has a very high oxygen affinity, it is use-less for delivery of oxygen to the tissues, makingits presence incompatible with life. HemoglobinH, composed of four beta chains, also migratesfarther than Hb A, but Hb H disease is not fatal.

84.C. Bart's hydrops fetalis (homozygous alpha-thalassemia major) is a lethal condition in whichall normal hemoglobins are absent and the pres-ence of Bart's hemoglobin results in death due tohypoxia. Severe cases of Rh hemolytic diseaseof the newborn/fetus (erythroblastosis fetalis)are characterized by hemolytic anemia, highnumbers of nucleated red blood cells, and hyper-bilirubinemia that can cause brain damage (ker-nicterus). The bilirubin level is elevated butanemia is mild, if present, in ABO hemolyticdisease of the newborn.

Howell-Jolly bodies, and hypersegmented neu-trophils. Clinical severity generally differenti-ates the heterozygous (mild) and homozygous(severe) conditions of thalassemic and sicklecell syndromes. The anemia of acute blood lossis usually normocytic, whereas the anemia ofchronic blood loss becomes microcytic due tothe development of iron deficiency.

86.C. The cause of the many spherocytes and poly-chromatophilic red cells seen in Color Plate 8nwould be best determined with the directantiglobulin test (DAT). The differential diagno-sis is hereditary spherocytosis (negative DAT)and warm autoimmune hemolytic anemia (posi-tive DAT). Both of these hemolytic disorders arethe result of membrane injury and would showan increased osmotic fragility result due to thespherocytes, elevated reticulocyte counts, andelevated urine urobilinogen, as well as elevatedserum bilirubin levels.

87.D. Schistocytes and spherocytes are associatedwith red cell destruction and would be found inclostridial septicemia (toxins), prosthetic heartvalves (mechanical trauma), and thermal burns(heat). Microspherocytes can also result fromthe direct membrane damage caused byclostridial toxins and heat. Aplastic anemia isnot a hemolytic anemia but is caused bydecreased bone marrow production. Aplasticanemia is usually normocytic, with no evidenceof red cell damage on the blood smear, and redcell destruction tests such as serum bilirubinwould be normal.

85.B. The blood profile alone cannot distinguishfolic acid and vitamin B12 deficiencies, becauseboth are characterized by macrocytic ovalocytes,

Page 110: Hematology

CHAPTER 2: HEMATOLOGY

88.D. Erythrocytosis (polycythemia) is eitherabsolute or relative. Absolute erythrocytosisoccurs when the RBC mass increases, taking up alarger than usual proportion of the blood volume.Relative polycythemia occurs when the RBCmass stays normal but the amount of fluid volumedecreases, thus increasing the proportion of theblood occupied by red cells as compared to thetotal blood volume which has decreased. Primarypolycythemia is a condition of erythrocytosiswithout an underlying or contributing condition.The body produces an increased number of redcells without an increase in erythropoietin (aninappropriate response). Secondary polycythemiaoccurs when some underlying condition causesan increase in erythropoietin, so erythrocytosisoccurs secondary to the condition (an appropriateresponse).

89.A. Hemochromatosis is an excessive depositionof iron in body tissues that results in iron-ladenmacrophages, expansion of storage sites, andserious damage to organs (heart, liver). Iron over-load can be hereditary or acquired as a complica-tion of severe hemolytic anemias, frequent bloodtransfusions, or sideroblastic anemia. Hereditaryhemochromatosis is caused by a mutation ofthe HFE gene that results in increased absorptionof iron from the gastrointestinal tract and leadsto iron overload. It is associated with low levelsof hepcidin, an iron regulator, which causesincreased iron absorption and release of ironfrom macrophages. The treatment for hereditaryhemochromatosis is phlebotomy, and moleculartesting is done for diagnosis.

90.A. Abetalipoproteinemia, or hereditary acan-thocytosis, is a rare autosomal recessive disorderof lipid metabolism. An absence of serum betalipoprotein, a transport protein, causes abnormalplasma lipids. The numerous acanthocytes (spurcells) are the result of an alteration in the lipid

content of the red cell membrane. The anemia ismild, but this disorder is associated with pro-gressive neurologic disease.

91.A. The majority of body iron is found in thehemoglobin of circulating erythrocytes. Thismeans that any form of bleeding will lead toexcessive iron loss. Iron balance is normallyvery tightly controlled through absorption ratherthan excretion. Iron deficiency in males is rarebut, if present, it is usually the result of chronicgastrointestinal bleeding (ulcers, cancer).

92.B. The severe transfusion-dependent anemia,which is typical of homozygous beta-thalassemia,is the result of unbalanced globin-chain synthesisand massive red cell destruction that far exceedsthe rate of production. Decreased or absent betachains lead to excess alpha chains that precipitatein red cells and subsequently are destroyed. Theresponse is intense marrow erythroid hyperplasia,bone expansion, and erythropoiesis in extra-medullary sites. A complication of continuous redcell hemolysis and repeated blood transfusions isiron overload. Patients require iron chelationtherapy to prevent liver and heart failure. Splenec-tomy may be needed to reduce blood require-ments, but it is not done before 4 years of agebecause of the increased risk of infection.

93.A. Serum iron is low in both iron-deficiency ane-mia and the anemia of chronic disorders. The totaliron-binding capacity (TIBC), which is an indirectmeasure of the amount of transferrin protein, islow in the anemia of chronic disease, whereas itis high in iron deficiency. Synthesis of transferrinis regulated by iron availability. Usually, whenstorage iron decreases, serum iron levels decreaseand transferrin levels (TIBC) increase. In the ane-mia of chronic disorders, storage iron is normal orincreased (but unavailable), and transferrin levels(TIBC) are decreased.

Page 111: Hematology

ANSWERS & RATIONALES

94.C. Although the punctate basophilic stipplingfound in lead poisoning in erythrocytes is con-sidered a classic finding, the anemia present isusually not severe unless accompanied by irondeficiency. The presence of lead inhibits severalenzymes involved in the formation of heme,with a consequent increase in erythrocyte proto-porphyrin and urinary aminolevulinic acid. Themost significant effect of lead toxicity is theresulting neurological deficit and impairment ofmental development.

the cells contribute collectively to the severity ofsickle cell disorders. The classification of "trait"versus "disease" is not based on the severity ofsymptoms. The absence of Hb A and the presenceof over 80% Hb S on electrophoresis would beclassified as homozygous sickle cell disease (SS),whereas the heterozygous condition (AS) wouldshow approximately 60% Hb A and 40% Hb S.Sickling is rare in the trait condition because ofthe lower concentration of Hb S. Sickle cell dis-ease typically shows increased levels of compen-satory Hb F, as does hemoglobin SC disease.

95.C. In Color Plate 9B, the dual population of redblood cells represented may also be termed dimor-phic. This blood picture could be seen in a patientwith microcytic, hypochromic anemia after thetransfusion of normal red cells or when new nor-mocytic, normochromic red cells are producedafter successful treatment for iron deficiency.Concurrent deficiencies, such as coexisting ironand folate deficiency during pregnancy, wouldresult in the production of both microcytic andmacrocytic red cells.

96.

B. The cells visualized in Color Plate 10B aresickle cells in the presence of target cells. Thesubstitution of a valine for the glutamic acidnormally found in the sixth position of the beta-globin chain causes red cells containing hemo-globin S to undergo the characteristic shapechange that gives the sickle cell its name. Adefect of both beta genes results in sickle celldisease, whereas a single gene mutation causesthe sickle cell trait. Hemoglobin C results fromthe substitution of lysine for glutamic acid in thesixth position of the beta-globin chain.

97.D. The number of irreversibly sickled cells(ISCs) and the proportion of S hemoglobin within

98.B. Hyperbaric oxygen will reverse the sicklingprocess, but it will also suppress erythropoietin,which stimulates the bone marrow to produceadequate replacement erythrocytes. Hydrox-yurea reduces sickling by increasing Hb F levelsand has been shown to improve the clinicalcourse of patients plagued by painful crises.Treatment is primarily supportive and sympto-matic, with efforts made to avoid those factorsknown to precipitate a crisis.

99.D. In response to premature red cell destruc-tion, the normal bone marrow can speed up redcell production. Hemolytic anemias typicallyhave high reticulocyte counts, because the mar-row can respond to the need for red cells. Gener-ally, anemias caused by defective maturation ordecreased production have inappropriately lowreticulocyte counts, because the marrow fails torespond due to injury or lack of essentialhematopoietic components. Low hemoglobinand hematocrit values reveal the presence ofanemia but do not indicate etiology. The reticu-locyte count is particularly useful in distinguish-ing hemolytic anemias from other normocyticanemias that are not hemolytic.

Page 112: Hematology

I 340 • CH

—Sv ioo7m r. P

CHAPTER 2: HEMATOLu'GY

C. Both iron deficiency and heterozygous tha-lassemia can present with a mild microcytic,hypochromic anemia. Target cells may be seenin both, but basophilic stippling is only found inthalassemia. The hemoglobin A2 is normal inheterozygous alpha-thalassemia but is fre-quently twice the normal level in heterozygousbeta-thalassemia, because these individualscompensate with increased delta-chain produc-tion due to deficient beta-globin chain synthesis.In this case, iron deficiency would likely beruled out first with iron tests. Beta-thalassemiawith hemoglobin S trait (Hb S/beta-thalassemia)produces a severe clinical picture similar tosickle cell anemia, with sickling of red cells.

101.D. Oxidative denaturation is the primarymechanism of the hemolytic process. Whenglucose-6-phosphate dehydrogenase (G6PD) isdeficient, the red blood cells cannot generatesufficient reduced glutathione (GSH) to detoxifyhydrogen peroxide. Hemoglobin is oxidizedto methemoglobin, denatures, and precipitates,forming Heinz bodies. The Heinz bodies causethe rigidity of the red cells, and hemolysisoccurs as the cells try to pass through the micro-circulation.

102.B. In hereditary spherocytosis, the rigid sphero-cytes are being destroyed in the splenic microcir-culation. Following splenectomy, the hemoglobinlevel should rise as the spherocytes circulatelonger. Consequently, there is less need forincreased red cell production by the bone marrow,and the number of reticulocytes released into theblood will fall. Approximately, 30% of plateletsare normally sequestered by the spleen, so a tran-sient increase in the platelet count occurs and redcell inclusions (Howell-Jolly and Pappenheimerbodies), normally pitted out of red cells by thespleen, will be observed.

103.D. Unconjugated bilirubin levels will rise wheneither excessive intravascular or extravascularhemolysis is occurring. When hemolysis isintravascular, the free hemoglobin released intothe circulation is bound by haptoglobin, and thecomplex is transported to the liver, where it ismetabolized to bilirubin. Depletion of the hapto-globin protein will occur if use exceeds produc-tion, and then hemopexin binds hemoglobin forremoval. When both haptoglobin and hemo-pexin are depleted, plasma hemoglobin levelswill increase. The serum lactate dehydrogenaserises when red cells are broken down and intra-cellular LD enzymes are released.

104.A. Hemolytic anemias can be classified by themode of transmission (hereditary or acquired)and by the type of defect (intrinsic or extrinsic).With the exception of paroxysmal nocturnalhemoglobinuria (PNH), intrinsic defects arehereditary, and the defect that shortens survivalis within the abnormal red cell. Hemolytic ane-mias due to extrinsic defects are acquired andcaused by external agents or extracorpuscularfactors that destroy the intrinsically normal redcell.

105.

C. Paroxysmal cold hemoglobinuria (PCH) iscaused by an IgG biphasic antibody with Pspecificity known as the Donath-Landsteinerantibody. This autoantibody fixes complementto the red cells in the cold, and the complement-coated red cells lyse when warmed. PCH can beidiopathic or follow a viral infection and is char-acterized by acute intravascular hemolysisand hemoglobinuria after cold exposure. Coldautoantibodies usually show I specificity,whereas warm autoantibodies are often directedagainst Rh antigens on the red cells.

Page 113: Hematology

ANSWERS & RATIONALES • 341

106.B. The hypersegmented neutrophils and macro-cytic ovalocytes seen in Color Plate 11 • suggestthe presence of megaloblastic anemia. The twomost common causes are lack of folic acid orvitamin Bj2, which are coenzymes required fornormal DNA synthesis. This patient's neurologi-cal symptoms are indicative of a vitamin Bj2deficiency, because that vitamin is also neededfor myelin synthesis (CNS).

107.B. Pappenheimer bodies observed with Wright'sstain can be confirmed with the Prussian bluestain and are composed of iron. The presence ofsiderotic granules in the red cells is associatedwith iron overload, and the serum ferritin test,which reflects the amount of storage iron, wouldbe elevated. The test for parietal cell antibodiescan be done to determine the cause of vitamin B12

deficiency.

108.D. Hemoglobinopathies are a hereditary groupof qualitative disorders in which genetic muta-tions cause the production of structurally abnor-mal globin chains. The three most commonvariant hemoglobins are Hb S, Hb C, and Hb E,all of which are due to an amino acid substitu-tion in the beta-globin chain. Hemoglobin C isthe second most common hemoglobin variant,after hemoglobin S, seen in the United States.Thalassemias are characterized by an absent orreduced rate of globin-chain synthesis.

109.C. The structurally abnormal red cells in hered-itary spherocytosis (HS) are deficient in spectrinand are abnormally permeable to sodium. Thebone marrow produces red cells of normalbiconcave shape, but HS cells lose membranefragments and become more spherical as they gothrough the spleen and encounter stress in the

blood circulation. The membrane defect isaccentuated by the passage of red cells throughthe spleen, where they are deprived of glucoseand are unable to generate sufficient ATP topump sodium out of the cell. Ultimately, the redcells are trapped and destroyed in the spleen.The osmotic fragility is increased because of themembrane loss (reduced surface area-to-volumeratio), and the MCHC value may be increased.

no.A. In hereditary elliptocytosis (HE), the red bloodcells show increased permeability to sodium andmay have one of several membrane defects linkedto this heterogeneous disorder. These include defi-ciencies in skeletal proteins such as protein 4.1 orspectrin. The characteristic oval or elliptical shapeis seen only in mature red blood cells, and it occursin the circulation when HE red cells cannot returnto a normal biconcave shape.

111.C. The anemia of chronic disease (ACD) is verycommon and develops in patients with chronicinfections (tuberculosis), chronic inflammatorydisorders (rheumatoid arthritis, systemic lupus),and malignant disease (cancer, lymphoma).ACD has a complex etiology that includesimpaired release of storage iron for erythro-poiesis and a reduced response to erythropoietin.The anemia may be normocytic or microcytic,and severity depends on the underlying disorder.

112.A. Hemoglobin C disease results from ahomozygous substitution of lysine for glutamicacid at position 6 of the beta-globin chains.Numerous target cells and occasional intracellu-lar C crystals will be found on the blood smear.Osmotic fragility is decreased (increased resis-tance) because of the many target cells, and elec-trophoresis will show an absence of Hb A andover 90% Hb C. The clinical severity of Hb CCis mild as compared to Hb SS or Hb SC diseases.

Page 114: Hematology

342 • CHAPTER 2: HEMATOLOGY

113. 116.D. Red cells that contain a high concentrationof hemoglobin S will assume the sickle shapewhen deprived of oxygen (which can bereversed if reoxygenated). After repeated sick-ling, reversion capabilities are lost and irre-versibly sickled cells (ISCs) are seen. Sicklecells are mechanically brittle, nondeformablecells that become impeded in circulation, caus-ing blocks that restrict blood flow in vessels andleading to organs (vascular occlusive disease).They are easily trapped in the small vessels ofthe spleen, leading to obstructive ischemia andeventual destruction of splenic tissue.

114.B. Aplastic anemia is bone marrow failurecharacterized by hypocellularity and decreasedproduction of all cell lines. The normal M:Eratio (4:1) does not change in aplasia, becausethe number of both myeloid and erythroid pre-cursors is decreased. In anemias such as sicklecell anemia, beta-thalassemia major, or mega-loblastic anemia, the marrow becomes hypercel-lular because of an increase in erythroidprecursors, and the M:E ratio falls.

115.B. The immune hemolytic anemia indicated bythe smear findings is warm autoimmunehemolytic anemia (WAIHA). Antibody-coatedred cells are being partially phagocytized bymacrophages (with receptors for IgG and com-plement), causing loss of membrane fragments.The spherocytes are ultimately destroyed, pri-marily in the spleen. The cause of the autoanti-body production may be unknown (idiopathic),develop secondary to a disease that alters theimmune response (chronic lymphocyticleukemia or lymphoma), or can be drug induced.Cold hemagglutinin disease is characterized byred cell agglutination due to a cold autoantibody.

C. Intrinsic factor is a glycoprotein secreted bythe parietal cells, along with HC1, that is needed tobind vitamin B12 for absorption. Pernicious ane-mia (PA), which is a megaloblastic anemia causedby the lack of intrinsic factor, is most commoncause of vitamin B12 deficiency (cobalamin). PAis characterized by atrophy of the gastric parietalcells and achlorhydria (absence of HC1). Autoim-mune factors are involved because a high percent-age of patients produce autoantibodies to intrinsicfactor (50%) and/or parietal cells (90%). Thebone marrow erythroid precursors exhibit mega-loblastic maturation, with nuclear maturationlagging behind cytoplasmic maturation (asyn-chrony is also seen in developing granulocytes andplatelets). Many fragile red cells die in the bonemaiTow, and those released into the circulationhave a very short survival, which causes a markedincrease in lactate dehydrogenase levels.

117.B. Hemoglobin D migrates in the same locationas hemoglobin S on cellulose acetate at alkalinepH but does not cause sickling. The negative solu-bility test rules out the presence of hemoglobin S.Target cells are seen in large numbers in homozy-gous hemoglobin D disease. The quantification of95% differentiates homozygous from heterozy-gous states where less than 50% hemoglobin Dwould be seen.

118.B. G6PD deficiency compromises the ability ofthe glutathione reduction pathway to prevent theoxidation of hemoglobin. Oxidative stress mayoccur from infections, ingestion of mothballs,ingestion of fava beans, and certain drugs,including primaquine or sulfonamides. The oxi-dized hemoglobin precipitates in the form ofHeinz bodies, which leads to a hemolytic crisischaracterized by intravascular red cell destruc-tion, removal of Heinz bodies by splenicmacrophages, and the presence of spherocytesand fragmented red cells on the smear.

Page 115: Hematology

119.B. Paroxysmal nocturnal hemoglobinuria(PNH) is an acquired defect of membrane struc-ture in which red cells have a high affinity forcomplement binding. PNH is characterized bypancytopenia and chronic intravascular hemoly-sis with hemoglobinuria and hemosiderinuria. Astem cell mutation causes production of redcells, white cells, and platelets that are sensitiveto complement lysis because of the loss of amembrane glycolipid (GPI). The sucrose hemol-ysis test can be used to screen for PNH red cells,but the Ham's acid serum test has been replacedby immunophenotyping for confirmation ofPNH. Paroxysmal cold hemoglobinuria (PCH)is characterized by intravascular hemolysis andhemoglobinuria after cold exposure that is dueto a complement-binding autoantibody. A tran-sient finding of hemoglobinuria following force-ful contact of the body with hard surfaces (asmay be seen in joggers and soldiers) describesMarch hemoglobinuria.

120.A. Secondary, acquired sideroblastic anemia isthe result of blocks in the protoporphyrin path-way that can be identified and, therefore, arereversible if the toxin or agent is removed.Alcohol inhibits vitamin B6 and anti-tuberculosisdrugs also interfere with vitamin B6 (pyridox-ine). Lead poisoning causes multiple blocks inprotoporphyrin synthesis, including inhibitionof ferrochelatase, which is needed for iron incor-poration into protoporphyrin to produce heme.Methotrexate is an antifolate chemotherapeuticagent that causes a drug-induced megaloblasticanemia.

121.A. Although iron deficiency may be the mostcommon cause of anemia in pregnancy, there is amild form of anemia that develops during thethird trimester in pregnant women with adequateiron levels. Although both erythrocytes and

ANSWERS & RATIONALES

plasma increase during pregnancy, the plasmaincreases in a higher proportion, causing a rela-tive (pseudo) anemia. This increased blood vol-ume actually increases oxygen delivery to boththe mother and the fetus.

122.A. The anemia of chronic renal failure resultsfrom decreased production and release of ery-thropoietin from the diseased kidney. The dropin erythropoietin results in decreased red bloodcell production by the marrow. Recombinanterythropoietin is of great value in treating ane-mia resulting from end-stage renal disease. Ironor folate supplements may be needed to maxi-mize the response, especially in patients on dial-ysis. Uremic metabolites may cause reduced redcell survival and impairment of platelet function.

123.D. Aplastic anemia is a stem cell defect thatleads to decreased production of erythrocytes,leukocytes, and platelets (pancytopenia). Thesurvival of red cells released into the circulationis normal. Infection is a serious problem becauseof the lack of neutrophils. The reduced numberof platelets is responsible for the bleeding oftenseen. Treatment includes blood and platelettransfusions, antibiotics, growth factors, andsteroids. Bone marrow transplantation may benecessary.

124.B. The fish tapeworm competes for vitaminBj2, and a macrocytic (megaloblastic) anemiamay develop. Hookworm infestation causeschronic blood loss and a microcytic anemia dueto iron deficiency. A variety of organisms areassociated with hemolysis, including malariaand clostridial infections. Viral hepatitis cancause marrow suppression and a normocytic,hypoproliferative anemia.

343 ̂ m

1 J

Page 116: Hematology

344 CHAPTER 2: HEMATOLOGY

125. 128.A. A need for the increased oxygen carryingcapacity provided by additional red blood cellsis found in conditions such as pulmonary dis-ease, where normal oxygenation is inhibited. Adecrease in the ability of the cardiovascular sys-tem to appropriately circulate cells is anotherreason for increased erythrocytes. Individualswith a high level of methemoglobin, such asheavy smokers or persons with genetic disor-ders, cannot effectively unload oxygen. Thisresults in a need for increasing the number of redblood cells to compensate. Renal tumors areassociated with excess production of erythropoi-etin, leading to an inappropriate polycythemia.

126.B. Thalassemias are a group of congenitaldisorders characterized by quantitative defects inglobin-chain synthesis. Alpha-thalassemias resultfrom gene deletions that cause a reduced rate ofalpha-globin chain production. Beta-thalassemiasresult from point mutations that cause a reducedrate of beta-globin chain synthesis. Normally,equal amounts of alpha- and beta-globin chainsare produced for Hb A synthesis. In alpha- or beta-thalassemias, synthesis of globin chains is imbal-anced, because a decreased production rate of onetype of globin chain causes an excess of the other(consequences will depend on the thalassemiatype).

127.C. Low serum iron and iron stores (representedby serum ferritin) characterize iron deficiencythat is severe enough to result in anemia. Theproduction of transferrin, the iron transport pro-tein, increases as iron stores decrease. Transfer-rin saturation decreases dramatically so thattransferrin is less than 15% saturated with iron.

D. The switch from gamma-globin chain pro-duction for Hb F to beta-globin chain synthesisfor Hb A occurs 3-6 months after birth. Clinicalsymptoms of a homozygous beta-globin chaindefect, such as sickle cell disease or homozy-gous beta-thalassemia, will not be evident untilabout 6 months of age or shortly after. Alpha-globin chain production is normally highthroughout fetal and adult life. A homozygousdefect involving the alpha-globin chain willaffect the infant in utero.

129.A. The hemolytic crisis of malaria results fromthe rupture of erythrocytes containing merozoites.This event becomes synchronized to produce thefever and chill cycles that are characteristic ofthis infection. In severe infections, particularlythose caused by Plasmodium falciparum, themassive intravascular hemolysis results in signifi-cant hemoglobinuria.

130.D. The incorrect ratio of blood to anticoagulantcaused the cells to shrink. This produced the cre-nated appearance of the red cells. This is an arti-fact as opposed to a significant clinical findingand can also be the result of prolonged bloodanticoagulation. Spur cells (or acanthocytes) lacka central pallor area and have sharp projections,as opposed to crenated cells (or echinocytes),which have a pallor area and blunt projections.

131.A. Pyruvate kinase (PK) is an enzyme of theEmbden-Meyerhof pathway (anaerobic glycoly-sis). A deficiency of PK results in decreased ATPgeneration, which causes impairment of thecation pump and a loss of normal membranedeformability. PK-deficient cells have a short-ened survival time, but clinical manifestationsvary widely.

Page 117: Hematology

ANSWERS & RATIONALES • 345

132.B. When iron use exceeds absorption, ironstores (serum ferritin) are depleted first. At thisearly stage, there is no anemia (normal hemo-globin) and the transferrin level is normal. Thisis followed by increased transferrin synthesis(TIBC) and decreased serum iron. Finally, amicrocytic, hypochromic anemia develops.

133.A. In the anemia of chronic disease (ACD), achronic illness causes impaired release of ironfrom storage. These patients have iron but areunable to use it for bone marrow erythropoiesis.Hepcidin, a hormone produced by the liver, playsa major role in the regulation of body iron byinfluencing intestinal absorption and release ofstorage iron from macrophages. Hepcidin levelsincrease during inflammation (positive acute-phase reactant), which causes decreased releaseof iron from stores. There is also impairedresponse of marrow red cell precursors to ery-thropoietin stimulation in ACD. The impairedresponse is thought to be related to the effects ofinflammatory cytokines. Recombinant erythro-poietin improves the anemia in some cases.

134.C. In the idiopathic or primary type of siderob-lastic anemia, the blocks in the protoporphyrinpathway (heme synthesis) that lead to iron over-load are unknown and, therefore, are irreversible.The anemia is refractory (unresponsive) to treat-ment other than transfusion. Ringed sideroblastsand increased stainable iron will be found in thebone marrow when stained with Prussian blue.This primary, acquired form of sideroblastic ane-mia is also known as refractory anemia withringed sideroblasts (RARS) and is classified as amyelodysplastic syndrome.

135.B. The demand for red blood cell replacement inbeta-thalassemia major during early childhood

development results in a hyperproliferative mar-row. Expansion of the marrow causes the bonesto be thin and narrow. This may result in patho-logic fractures. Facial bones have the Mongoloidappearance, with prominence of the forehead,cheekbones, and upper jaw.

136.D. Cold autoimmune hemolytic anemia (CAIHA),or cold hemagglutinin disease, is characterized bythe production of IgM cold autoantibodies thatoften show I specificity. The cause of the autoanti-body production may be unknown (primary) oroccur secondary to Mycoplasma pneumonia orlymphoma. Significantly high liters can result inagglutination of red cells in the extremities calledRaynaud's phenomenon (acrocyanosis).

137.A. The red blood cell distribution width (RDW)is an index of red cell size variation or anisocyto-sis. The RDW will be high when a heterogeneouscell population consisting of red cells with varyingsizes is present (sickle cell anemia with compen-sation). The RDW is low when a homogeneousor single population of red cells is present that areof uniform size (thalassemia minor, anemia ofchronic disease).

138. _____D. Splenomegaly is a common finding in hemo-lytic anemias, because the spleen is the major siteof extravascular red cell destruction. Patients withhereditary spherocytosis and hemoglobin SC dis-ease often have enlarged spleens for this reason.Patients with beta-thalassemia major exhibitsplenomegaly because of active splenic removalof red cells, but the spleen may also be a site ofextramedullary erythropoiesis. Splenomegaly canalso be due to extramedullary hematopoiesis inmalignant disorders such as polycythemia vera ormyelofibrosis. Splenomegaly would not be a char-acteristic finding in megaloblastic anemia.

1

Page 118: Hematology

346 • CHAPTER 2: HEMATOLOGY

Leukocytes

139.C. The major function of leukocytes is defense,either by phagocytosis or by immune mecha-nisms. The phagocytic cells are the granulocytesand monocytes. The immune response is medi-ated by lymphocytes; however, monocytes playa role in immunity as antigen-presenting cells.Leukocytes may be classified according to gran-ularity as granulocytes and nongranulocytes ordivided based on nuclear segmentation as poly-morphonuclears (PMNs) and mononuclears.

140.D. Monocytes have a diameter up to 20making them the largest cells in the peripheralblood under normal conditions. Eosinophils andneutrophils have diameters of about 12 |xm. Thesmall lymphocyte is 8-9 |xm in diameter, similarto the red blood cell, which has a diameter of6-8 (xm. Large lymphocytes range in size from11 to 16 jim in diameter.

141.A. After granulocytes are released from thebone marrow, they remain in the circulation oneday or less. Their major function takes place inthe tissues. They migrate through the vessel wallsto reach areas of inflammation very soon afterrelease. The life span of the granulocyte is short;however, eosinophils and basophils appear tosurvive longer in the tissues than neutrophils.

142.C. Approximately 50% of the neutrophils in theperipheral blood are found in the circulatingpool. This is the pool measured when a totalWBC count is done. Another 50% are foundadhering to vessel walls (marginal pool). Thesepools are in constant exchange. Emotional orphysical stimuli can cause a shift of cells fromthe marginating pool to the circulating pool,causing a transient rise in the total WBC count.

The total WBC count can double but returns tonormal within several hours.

143.A. Although some phagocytic activity has beenattributed to the eosinophil, it is the segmentedneutrophil and monocyte that have the greatestphagocytic activity. The neutrophil is the mostimportant because of numbers and its ability torespond quickly, especially against bacterialpathogens. Monocytes arrive at the site of injuryafter the neutrophil to "clean up."

144.B. The growth factor mainly responsible for reg-ulating the production of granulocytes and mono-cytes is granulocyte/monocyte colony-stimulatingfactor (GM-CSF), which acts on the committedbipotential progenitor cell CFU-GM (colony-forming unit-GM). GM-CSF stimulation of gran-ulocyte or monocyte production increases inresponse to need and can also affect the produc-tion of erythrocytic and megakaryocytic lineages.G-CSF induces granulocyte differentiation, andM-CSF supports monocyte differentiation. Ery-thropoietin (EPO) is a lineage-specific growth fac-tor responsible for stimulating erythrocyteproduction, and thrombopoietin (TPO) is mainlyresponsible for regulating platelet production.Interleukins, particularly IL-3, influence multiplecell lines, including granulocytes and monocytes.

145.A. The granulocyte mitotic pool contains thecells capable of division, which are themyeloblasts, promyelocytes, and myelocytes.The post-mitotic pool, or reserve, is the largestbone marrow pool and contains metamyelo-cytes, band and segmented forms. This pool isavailable for prompt release into the blood ifneeded (e.g., infection), and its early release isthe cause of a "left shift." If released, the bonemarrow mitotic pool can dramatically increaseits activity to replenish this reserve (cytokinestimulation increases).

Page 119: Hematology

ANSWERS & RATIONALES 347

146.B. A "shift to the left" means an increase inimmature neutrophilic cells in the blood causedby bone marrow release of cells in response toinfection or tissue damage. A redistribution ofthe blood pools because of emotional or physicalstimuli is characterized by an increased WBCcount without a left shift. A cell "hiatus" refersto a population of cells in which there is a gap inthe normal maturation sequence. A cell hiatus ismost often seen in acute leukemia, in whichthere are many blasts and a few mature cells butno intermediate stages.

147.D. "Agranulocytosis" refers to an absence ofgranulocytes in both the peripheral blood andbone maiTow. A deficiency of granulocytes isfound in cases of aplastic anemia, in which defi-ciencies in red cells and platelets also occur. Theearly release of cells from the bone marrow willresult in immature cells in the blood but is notreferred to as agranulocytosis. Neutrophils thatexhibit little or no granulation may be calledhypogranular or agranular and are a sign ofabnormal growth (dyspoiesis).

148.B. Antibodies are synthesized by plasma cells,which are end-stage B lymphocytes that havetransformed to plasma cells following stimula-tion by antigen. An end product of T cell activa-tion is the production of cytokines (lymphokines)such as interleukins and colony-stimulating fac-tors. T cells are surveillance cells that normallycomprise the majority (about 80%) of lympho-cytes in the blood. T cells regulate the immuneresponse by helping (T helper or inducer cells) orsuppressing (T suppressor cells) the synthesis ofantibody by plasma cells.

149.C. Absolute values for cell types are obtainedby multiplying the percentage of the cell type by

the total number of cells. In this case, 4000/mm3 X0.65 = 2600/jJiL or 2.6 X 109/L. Although refer-ence ranges vary, the normal absolute count forlymphocytes is from 1.0 to 4.0 X 109/L and thenormal percentage of lymphocytes is 20-44%.In this case, there is a relative lymphocytosis(increase in percentage), but the absolute lym-phocyte value is normal. Percentages can bemisleading, so the absolute number of a particu-lar cell type should always be evaluated.

150.B. Auer rods are seen in the cytoplasm of malig-nant cells, most often myeloblasts, and are com-posed of fused primary (nonspecific, azurophilic)granules. Hypersegmented neutrophils have fivelobes or more and are associated with vitaminB12 or folate deficiency. Toxic granules are pri-mary granules with altered staining characteris-tics that stain in late-stage neutrophils due totoxicity. Dohle bodies are agranular patches ofRNA present in neutrophil cytoplasm and associ-ated with toxic states.

151.D. The total white blood cell count referenceranges for males and females are equivalent.WBC counts do change with age, being higherin newborns and children than in adults. Anychange from basal conditions, such as exerciseor emotional stress, will cause a transient leuko-cytosis due to a redistribution of blood pools.WBC values are lower in the morning andhigher in the afternoon (diurnal variation).

152. __C. Neutropenia is associated with a risk of infec-tion. The degree of neutropenia correlates with theinfection risk from high susceptibility (<1.0 X109/L) to great risk (<0.5 X 109/L). Infectionincreases with the degree and duration of the neu-tropenia. Shortness of breath and bleeding tenden-cies are clinical symptoms associated with severeanemia and thrombocytopenia, respectively.

Page 120: Hematology

348 CHAPTER 2: HEMATOLOGY

153.B. Basophils and tissue mast cells have receptorsfor IgE and complement components, which trig-ger degranulation when appropriate antigens arepresent and are responsible for severe hypersensi-tivity reactions (anaphylaxis). Basophils and tis-sue mast cells have morphologic similarities butrepresent distinct cell types. Basophils possesswater-soluble granules that contain, among othersubstances, heparin and histamine (a vasodilatorand smooth muscle contractor). Basophils have asegmented nucleus, and the granules, althoughoften scanty, overlie the nucleus. The mast cellhas a single round nucleus, contains many moregranules than the basophil, and can be found inthe bone marrow.

154.C. The last stage in the granulocytic series thatdivides is the myelocyte. Cells before and includ-ing this stage constitute the bone marrow mitoticpool and undergo multiple cellular divisions.Nuclear chromatin progressively clumps andnucleoli are no longer present in the nondividingmetamyelocyte stage that follows the myelocyte.

155.C. The precursor cell that can first be recognizedas granulocytic is the myeloblast and has nogranules. Primary or nonspecific granule produc-tion begins and ends during the promyelocytestage. The granules are distributed betweendaughter cells as mitotic divisions occur. Sec-ondary or specific granule production beginswith the myelocyte stage and continues duringsucceeding cell stages with the synthesis of prod-ucts specific to the function of the particulargranulocyte (neutrophil, eosinophil, or basophil).

156.B. Eosinophils lack lysozyme, which is presentin neutrophils and monocytes, and contain a dis-tinctive peroxidase that differs biochemicallyfrom the myeloperoxidase of neutrophils andmonocytes. Major basic protein is a component

of the granules and is very important to the abil-ity of eosinophils to control parasites. In addi-tion, eosinophils play a role in modifying theallergic reactions caused by degranulation ofbasophils. Basophils release eosinophil chemo-tactic factor of anaphylaxis (ECF-A), whichcalls eosinophils to the site.

157.

A. Primary granules, which appear in thepromyelocyte stage, may be called azurophilic ornonspecific granules. Specific or secondary gran-ules (neutrophilic, eosinophilic, basophilic)appear in the myelocyte stage. Primary granulescontain hydrolytic enzymes (e.g., myeloperoxi-dase, lysozyme, acid phosphatase) and are coatedwith a phospholipid membrane. Lactoferrin is acomponent of neutrophil granules. Primary gran-ules are visible in the myelocyte stage, but in laterstage cells the primary granules, although present,are less visible by light microscopy under normalconditions.

158.A. The presence of toxic granules, Dohle bod-ies, and/or vacuoles in the cytoplasm of neu-trophils (segmented, band, metamyelocyte, andmyelocyte stages) is indicative of a neutrophilicresponse to inflammation. The changes observedin the "toxic" neutrophil may occur in patientswith severe burns, some malignancies, exposureto toxic drugs and chemicals, and acute infection(most often bacterial). A Barr body is a "drum-stick"-shaped body of nuclear material found inthe neutrophils of females that represents theinactive X chromosome and is of no signifi-cance. Auer rods are seen in malignant myeloidcells, usually blasts. Hypersegmented neu-trophils are associated with megaloblastic ane-mias but may be seen in long-term chronicinfections. Pyknotic cells and vacuoles may beseen in overwhelming sepsis or in a degenerat-ing blood specimen. Russell bodies are globularinclusions found in plasma cells that are com-posed of immunoglobulin.

Page 121: Hematology

ANSWERS & RATIONALES

159. 162.

3491 I

- IA. Diapedesis is the movement of cells (usuallyreferring to neutrophils) from the blood streaminto the tissues by squeezing through endothelialcells of the vessel wall. Chemotaxis is the move-ment of cells directed by chemotactic stimulisuch as bacterial products, complement compo-nents, or injured tissue. Opsonization is the coat-ing of an organism or foreign particle by IgG orcomplement for recognition and phagocytosisby neutrophils or monocytes. The ingestion ofred cells, often coated with IgG or complement,is called erythrophagocytosis. Margination is theattachment of neutrophils to the endothelial lin-ing of the blood vessels.

160.D. Basophil granules contain histamine, apotent vasodilator and smooth muscle contrac-tor, that is responsible for the systemic effectsseen in immediate hypersensitivity reactions(type I), which are also termed anaphylaxis.Degranulation occurs when basophils are coatedwith an IgE type of antibody that recognizes aspecific allergen, such as bee venom, certainplant pollens, or latex. The resulting anaphylac-tic shock can be life threatening.

161.B. Morphologic criteria such as cell size,nuclear shape, and chromatin pattern or cyto-plasmic granularity cannot be used to identifylymphocyte subtypes. Monoclonal antibodies(CD surface markers) to specific surface andcytoplasmic antigens can distinguish lympho-cyte subpopulations and identify the develop-ment stage. For example, blood lymphocytesthat are B cells express CD 19 and CD20 mark-ers, T cells express CD2 and CDS (and eitherCD4 or CDS markers), and NK cells expressCD56. Natural killer cells often exhibit largegranular lymphocyte morphology (LGLs).

B. Plasma cells are the mature end stage of theB lymphocyte, producing immunoglobulins(antibodies) in response to activation by a spe-cific antigen (humoral immunity). The antibodyproduced by a single plasma cell is of oneimmunoglobulin type. Natural killer (NK) cellsrecognize and kill tumor cells or cells infectedwith virus through direct contact. Virocytes arereactive lymphocytes, and thymocytes areimmature T cells. T lymphocytes provide cellu-lar (cell mediated) immunity.

163.A. A function of the eosinophil is to modify thesevere allergic reactions caused by degranulationof the basophil. Neutrophils have receptors for theopsonins IgG and complement and are the mostimportant cell in the initial defense against acutebacterial infection. Neutrophils are nonspecificphagocytes, ingesting bacteria, fungi, dead cells,etc., and they contain hydrolytic enzymes, includ-ing muramidase (lysozyme) and alkaline phos-phatase. Neutrophils die in the performance oftheir function and are removed by macrophages.

164.

C. The nucleus in both monocytes and reactivelymphocytes can be irregular in shape, withindentations, although a monocyte nucleus oftenhas folds and lobulations. Reactive lymphocytescharacteristically have an increased amount ofdark blue cytoplasm, whereas monocyte cyto-plasm is usually a blue-gray color. Lymphocyteslack the many fine granules that give monocytesa typical "ground glass" appearance of the cyto-plasm, but monocytes can occasionally havelarger granules. Sharp indentation of the cyto-plasm by adjacent red cells and an increasednumber of large granules are features of reactivelymphocytes. Vacuoles, although more com-monly present in monocytes, can also be seen inreactive lymphocytes.

Page 122: Hematology

350 • CHAPTER 2: HEMATOLOGY

165. 168.B. Indentation of the nucleus (kidney shape) isthe feature that characterizes the metamyelocytestage. Specific granules begin forming in themyelocyte and persist through later stages.Cytoplasmic color is not a reliable feature,because it is variable and may not differ signifi-cantly from the myelocyte or band stage. Nucle-oli are absent in metamyelocytes and may not bevisible in myelocytes (they may be indistinct).

166.A. Young children have the highest peripherallymphocyte concentrations, ranging from 4.0 to10.5 X 109 cells/L at 1 year of age and decliningto 2.0-8.0 X 109 cells/L by 4 years of age. Lym-phocyte counts decrease with age because of adecrease in lymphocyte stimulation and process-ing of antigens, ranging from 1.0 to 4.0 X 109

cells/L in adults. In addition to the difference inlymphocyte number in children, the normalmorphology of children's lymphocytes differsfrom that of adults. Patient age should be consid-ered when deciding between normal and abnormallymphocytes.

167.A. Early B cell precursors would be expected toexpress TdT, CD 10, and CD34. TdT, the enzymemarker for terminal deoxynucleotidyl trans-ferase, and the stem cell marker CD34 are pres-ent on the earliest B or T lymphoid cells. Surfaceimmunoglobulin (SIgM) can only be detected onB cells at later stages of development. TdT canbe used to differentiate the leukemic cells ofacute lymphoid leukemia from acute myeloidleukemia. CALLA (CD 10 or common ALL anti-gen) is a marker found in precursor types of Bcell ALL.

C. Acid hydrolases and the number of lysosomesincrease as the blood monocyte matures into a tis-sue macrophage. Macrophages are widely dis-persed in body tissues and organs of thereticuloendothelial (RE) system (also known asthe mononuclear phagocyte system). Macro-phages have receptors for IgG and complement,and they serve as phagocytes by ingesting debrisand dead cells (usually neutrophils) at sites ofinflammation. Macrophages act in the immuneresponse as antigen-presenting cells by ingestingand exposing antigens for recognition by lympho-cytes. Monocytes/macrophages secrete comple-ment components and cytokines, includingcolony stimulating factors and interleukins.

169.B. Antigen-independent lymphopoiesis occursin primary lymphoid tissue located in the thymusand bone marrow. The formation of immuno-competent T and B cells from the lymphoid pro-genitor cell is influenced by environment(thymus, marrow) and several interleukins. Anti-gen-dependent lymphopoiesis occurs in second-ary lymphoid tissue (spleen, lymph nodes,Peyer's patches) and begins with antigenic stim-ulation of immunocompetent T and B cells. Lym-phocytes are the only white cells that recirculate(i.e., return to the blood from the tissues).

170.D. Myeloperoxidase is an enzyme present in theprimary granules, regardless of the phagocyticactivity of the cell. The products produced dur-ing the respiratory burst, many of them shortlived, are generated in response to chemotacticactivation and ingestion of microbes. Generationof oxygen metabolites is necessary for microbialkilling.

Page 123: Hematology

ANSWERS & RATIONALES

Leukocyte Disorders

171.C. The Epstein-BaiT vims (EBV) attaches toreceptors on B lymphocytes, and the virus isincorporated into the cell. The infection generatesan intense immune response of T cells directedagainst infected B cells. It is the activated T lym-phocytes that comprise the majority of reactivelymphocytes seen in the blood of patients withinfectious mononucleosis. Other B cells producenonspecific polyclonal (heterophile) antibody inresponse to the EBV infection.

172.C. The malignant cells of hairy cell leukemia(HCL) stain positive with acid phosphatase inthe presence of tartaric acid; that is, hairy cellscontain tartrate-resistant acid phosphatase(TRAP). Normal cells stain acid phosphatasepositive, but staining is inhibited by the additionof tartrate. HCL is a chronic disorder, mainlyconfined to the elderly. The spleen usuallyshows marked enlargement, but enlarged lymphnodes are very uncommon. Hairy cells aremalignant B cells, and pancytopenia is usual atpresentation.

lymphocytes, and diagnosis is based on the pre-dominant site of involvement. Mycosis fun-goides and Sezary syndrome are different stagesof a cutaneous T cell lymphoma in which theskin is the early site of involvement, with subse-quent progression to the bone marrow andblood.

174.D. A leukoerythroblastic blood profile, whichrefers to the presence of both immature neu-trophils and nucleated red cells, is most com-monly associated with conditions involvingbone maiTOw infiltration by malignant cells(leukemia, cancer) or replacement by fibrotictissue. A neutrophilic left shift is defined as thepresence of increased numbers of immature neu-trophils in the blood without nucleated red cells.A regenerative left shift and a neutrophilicleukemoid reaction are characterized by varyingdegrees of leukocytosis and a neutrophilic leftshift, most often found in response to infection.In contrast, a degenerative left shift refers toleukopenia and a left shift that may occur if mar-row pools are depleted in an overwhelminginfection (use exceeds the bone marrow's capac-ity to replace).

173.A. The lymphoid cells of B cell acute lym-phoblastic leukemia (FAB type L3) are morpho-logically identical to the malignant B cells ofBurkitt lymphoma (large cells with basophiliccytoplasm and cytoplasmic lipid vacuoles).Although the site of origin is the bone marrow inB cell ALL and the tissues in Burkitt lymphoma,the World Health Organization (WHO) classifiesthem as the same disease entity with differentclinical presentations (Burkitt leukemia/lym-phoma). Both chronic lymphocytic leukemia(CLL) and small lymphocytic lymphoma (SLL)are malignant proliferations of small, mature

175.

A. "True" Pelger-Huet anomaly is a benignautosomal dominant trait characterized byhyposegmentation of the granulocytes, coarsenuclear chromatin, and normal cytoplasmicgranulation. The cells have no functional defect.It is of practical importance to recognize thisanomaly so that it is not confused with a shift tothe left due to infection. Acquired or "pseudo"Pelger-Huet is commonly associated withmyeloproliferative disorders, myelodysplasticsyndromes, or drug therapy. Pelgeroid cells arehyposegmented and the cytoplasm is frequentlyhypogranular.

Page 124: Hematology

352 • CHAPTER 2: HEMATOLOGY

176.A. Eosinophils are decreased in Gushing syn-drome, in which the adrenal glands secrete largeamounts of adrenocorticosteroids. Eosinophilsare increased in allergic disorders, various skindiseases, and certain types of parasitic infections(especially those due to intestinal and tissue-dwelling worms). Eosinophilia is also seen inchronic myelogenous leukemia and Hodgkinlymphoma.

177.D. Chronic granulomatous disease (CGD) is ahereditary disorder in which neutrophils are inca-pable of killing most ingested microbes. Thedisease is usually fatal because of defective gener-ation of oxidative metabolism products, such assuperoxide anions and hydrogen peroxide, whichare essential for killing. Chemotaxis, lysosomes,phagocytosis, and neutrophil morphology are nor-mal. Several variants of CGD have beendescribed, with specific enzyme defects and dif-ferent modes of inheritance. The more commontype of CGD has a sex-linked inheritance pattern.

178.B. A drug-induced megaloblastic blood profilewith macrocytic ovalocytes and hypersegmentedneutrophils is shown in Color Plate 111. This isa common finding in patients receiving antifo-late chemotherapeutic drugs such as methotrex-ate. Recombinant erythropoietin is associatedwith a reticulocyte response and used to treat avariety of conditions, such as renal disease, ane-mia of chronic disease, or anemia caused bychemotherapy. Chloramphenicol is an antibioticwith a known association for aplasia due to mar-row suppression.

179.C. Primary or essential thrombocythemia (ET) isa chronic myeloproliferative disorder in whichthe main cell type affected is the platelet. An

extremely high number of platelets are produced,but abnormal platelet function leads to bothbleeding and clotting problems. The bone marrowshows megakaryocytic hyperplasia. The hemo-globin value and platelet count are increased inpolycythemia vera, and CML is characterized bya high WBC count. Malignant thrombocythemiamust be differentiated from a reactive thrombocy-tosis seen in patients with infection or followingsurgery. In reactive causes, the platelet count israrely over 1 million X 109/L, platelet function isnormal, and thrombocytosis is transient.

180.B. The bone marrow is progressively replacedby fibrotic tissue in myelofibrosis, a chronicmyeloproliferative disorder. Attempts to aspiratebone marrow usually result in a "dry tap." Abiopsy stain demonstrates increased fibrosis(fibroblasts are thought to be stimulated bymegakaryocytes). The presence of teardrop-shaped red blood cells is an important feature ofmyelofibrosis. In addition, abnormal platelets, aleukoerythroblastic blood profile and myeloidmetaplasia in the spleen and liver are often asso-ciated with this disease. A high LAP score (ref-erence range 13-160) and increased RBC massare found in polycythemia vera, but the LAPscore is low in chronic myelogenous leukemia.

181.B. A striking lymphocytosis may be seen inchildren with pertussis, but normal lymphocytes,rather than reactive lymphocytes, are present. Arelative and/or absolute lymphocytosis withreactive lymphocytes in various stages of activa-tion, as seen in Color Plate 121, is characteristicof infection caused by Epstein-Barr virus(EBV), cytomegalovirus (CMV), and toxoplas-mosis. A positive heterophile antibody test canhelp distinguish infectious mononucleosiscaused by EBV from conditions with a similarblood picture. Epstein-Barr virus is also linkedto Burkitt and Hodgkin lymphomas.

Page 125: Hematology

ANSWERS & RATIONALES • 353

182. 185. IA. B cell chronic lymphocytic leukemia (CLL)is by far the most common type found in theUnited States. Immune dysfunction because ofhypogammaglobulinemia occurs in later stagesof the disease, as does thrombocytopenia. Devel-opment of warm autoimmune hemolytic anemiais a frequent occurrence in patients with CLL.Treatment for B cell CLL is conservative andaimed at controlling symptoms. T cell CLL is arare and is a more aggressive disease.

183.D. Waldenstrom macroglobulinemia is causedby a proliferation of transitional B lymphocytes(lymphoplasmacytic or plasmacytoid lymphs)that secrete high amounts of monoclonal IgM.Because IgM is a macroglobulin, blood hyper-viscosity is the cause of many of the symptomsfound in this disease (bleeding and visualimpairment). Plasmapheresis can reduce the IgMprotein concentration. Hepatosplenomegaly iscommon in Waldenstrom disease (rather thanbone lesions).

184.C. The elevated WBC count and toxic neu-trophils seen in Color Plate 13B indicate anextreme response to severe infection (bacterialsepticemia, fungal) or treatment with recombi-nant myeloid growth factors. GM-CSF and G-CSF are used to increase cells for peripheralstem cell transplant and reduce infection inpatients after high-dose chemotherapy or duringtransplant. A leukemoid reaction is one thatmimics the type of blood picture seen inleukemia. It is associated with extremely highleukocyte counts (often greater than 50 X 109

cells/L) and is usually found in severe infection.The most common type of leukemoid reaction isneutrophilic, but lymphocytic leukemoid reac-tions also occur. HIV infection is associatedwith leukopenia and lymphocytopenia.

B. The blood shown in Color Plate 14B is froman elderly patient with chronic lymphocyticleukemia (CLL), which is characterized by anabsolute lymphocytosis and a predominance ofsmall, mature lymphocytes with hyperclumpednuclear chromatin. Elevated leukocyte countsare usual, as are fragile, smudged lymphocytes.Acute lymphoblastic leukemia (ALL) typicallyoccurs in children and is characterized byimmature lymphoid cells. Plasmacytoid lym-phocytes and red cell rouleaux may be found inthe blood of individuals with Waldenstrom dis-ease. Viral infections are associated with a lym-phocytosis and the presence of reactivelymphocytes that are heterogeneous in mor-phology. Reactive lymphocytes exhibit a varietyof forms with regard to size and cytoplasmicstaining intensity as compared to the homoge-neous cell populations present in malignant dis-orders such as CLL and ALL.

186.D. Progression to acute leukemia is a veryunlikely event for patients with chronic lympho-cytic leukemia, even though there is no cure.Patients with chronic myelogenous leukemiatypically progress to "blast crisis," most often ofmyeloid type, unless treated with imatinib mesy-late (Gleevec®) in the chronic phase. Refractoryanemia with excess blasts (RAEB) is the mostlikely type of myelodysplastic syndrome todevelop acute myelogenous leukemia. Refrac-tory anemia with ringed sideroblasts (RARS) is"preleukemic" but fairly stable.

Page 126: Hematology

354 • CHAPTER 2: HEMATOLOGY

187.A. Gaucher disease is a lipid storage disorder inwhich there is an accumulation of glucocere-broside in the macrophages because of a geneticlack of glucocerebrosidase, an enzyme requiredfor normal lipid metabolism. Gaucher cells arefound in the liver, spleen, and bone marrow.Niemann-Pick disease is caused by a deficiencyof sphingomyelinase in which "foamy" macro-phages, called Niemann-Pick cells, are filled withsphingomyelin. Normal macrophages may con-tain iron and other cellular debris.

188.D. The presence of Reed-Sternberg cells is thediagnostic feature of Hodgkin disease (lym-phoma). The Reed-Sternberg giant cell is usu-ally binucleated, and each lobe has a prominentnucleolus. Studies suggest that this neoplasticcell is of B cell lineage. It is not found in theblood but only in the tissues. Circulating T cellswith a convoluted nucleus describe the Sezarycells seen in Sezary syndrome, the leukemicphase of mycosis fungoides. A monoclonal pop-ulation of large lymphoid cells or immature Bcells with nuclear clefts is most descriptive oflymphoma cells, present in certain types ofperipheralized non-Hodgkin lymphoma, thathave spread from the tissues to the bone marrowand blood.

189.B. The overall reaction of the body to tissueinjury or invasion by an infectious agent isknown as inflammation. This response bringsleukocytes to the site of infection or tissue dam-age and is associated with activation of inflam-matory mediators, including cytokines (IL-1),molecules released by cells (histamine), and by-products of plasma enzyme systems (comple-ment, kinins, fibrin). The plasma concentrationof positive acute-phase reactants (APRs), such

as C-reactive protein (CRP) and fibrinogen, canincrease dramatically in response to inflamma-tion, and levels of albumin and transferrin willfall (negative APRs). The erythrocyte sedimen-tation rate (ESR) will be elevated, primarilybecause of the rise in fibrinogen.

190.C. The abnormal cells found in acute promyelo-cytic leukemia (FAB type M3) contain largenumbers of azurophilic granules. These granulescontain procoagulants that on release hyperacti-vate coagulation, resulting in disseminatedintravascular coagulation. Although other acuteleukemias may trigger DIG, M3 is the one mostfrequently associated with this life-threateningbleeding complication. If DIG is resolved, manypatients with acute promyelocytic leukemiarespond favorably to therapy with retinoic acid,which causes maturation of the malignantpromyelocytes. The presence of t(15;17) hasdiagnostic and prognostic significance, andacute promyelocytic leukemia is classified with"acute myeloid leukemias with recurrent cytoge-netic translocations" by the World Health Orga-nization (WHO). Acute myeloblastic leukemiawith t(8;21) is also included in this WHO cate-gory (con-elates with FAB type M2).

191.D. Although a hallmark of acute lymphoblasticleukemias (ALL), lymphadenopathy is not asso-ciated with acute myelogenous leukemias. ALLis also more likely to have central nervous sys-tem involvement, and the CNS is a potential siteof relapse. Hepatomegaly and splenomegaly areassociated with both types of acute leukemia, aswell as with the presence of anemia, neutrope-nia, and thrombocytopenia. Common presentingsymptoms are fatigue, infection, or bleeding. Ifuntreated, both acute myelogenous and lym-phoblastic leukemias have a rapidly fatal course.

Page 127: Hematology

ANSWERS & RATIONALES

192. 195.B. The blast cells shown in Color Plate 15 • arefrom a child with CALLA positive, precursor Bacute lymphoblastic leukemia. The malignantcells would be expected to express CD 10, the com-mon ALL antigen marker; the B cell lineage markerCD 19; and TdT (terminal deoxynucleotidyl trans-ferase), a marker on early lymphoid cells. Precur-sor T acute lymphoblastic leukemia would expressTdT and the T cell markers CD2 and CD7. CD 13and CD33 are myeloid markers, and CD14 is amarker for monocytic cells.

193.A. The "packed" bone marrow with predomi-nantly immature blast cells and few normal pre-cursor cells, as seen in Color Plate 16>, is mostindicative of a patient with acute leukemia.Although chronic leukemias usually have a hyper-cellular marrow, the malignant cells are moremature or differentiated (i.e., able to maturebeyond the blast stage). Myelodysplastic syn-dromes are associated with a hypercellular bonemarrow, but the marrow blast percent is less than20% (using WHO criteria). Aplastic anemia ischaracterized by a hypocellular bone marrow withfew cells.

194.C. The secretion of large amounts of monoclonalIgG or other immunoglobulin light chains by amalignant clone of plasma cells produces a char-acteristic M spike on serum and urine proteinelectrophoresis. In some cases, only the lightchains are produced in excess. Because the lightchains are easily cleared by the kidneys, they mayappear only in the urine (Bence-Jones protein-uria). Renal impairment in multiple myeloma isassociated with the toxic effects of filtered lightchains. High levels of serum beta microglobulincorrelate with the myeloma tumor burden. Cryo-globulins are proteins that precipitate in the coldand may be seen in multiple myeloma andWaldenstrom macroglobulinemia.

IC. Multiple myeloma is a malignant lympho-proliferative disorder characterized by a clonalproliferation of plasma cells and multiple bonetumors. Myeloproliferative disorders are charac-terized by a proliferation of bone marrow cells(granulocytic, monocytic, erythrocytic, megakary-ocytic), with usually one cell type primarilyaffected. For example, the main cell type affectedin polycythemia vera is the erythrocyte, and theplatelet is mainly affected in essential thrombo-cythemia. Transformation among the myeloprolif-erative disorders is frequent.

196.

B. The Philadelphia chromosome, t(9;22), isdetected in almost all cases of CML (depends ondetection method) and results in a mutatedBCR/ABL fusion gene. The resulting fusion pro-tein causes increased tyrosine kinase activity,which promotes cell proliferation. Imatinib mesy-late (Gleevec®) is a therapeutic agent that targetsthe molecular defect by blocking tyrosine kinaseactivity and is now a first-line drug used in thechronic phase of CML. The t(15;17) that is diag-nostic of promyelocytic leukemia results in aPML/RARA (retinoic acid receptor alpha) fusiongene that blocks maturation. Many PML patientsrespond to retinoic acid therapy, which inducespromyelocyte differentiation. Nearly all cases ofBurkitt lymphoma have t(8; 14), which is a translo-cation of the MYC gene from chromosome 8 tothe Ig heavy chain (IgH) region on chromosome14. JAK2 (Janus kinase) is a point mutation in agene regulating cell proliferation, and it is presentin over 90% of polycythemia vera cases andapproximately 50% of those with essential throm-bocythemia and myelofibrosis. Detection of cyto-genetic and molecular mutations has diagnosticand prognostic significance and is an importanttool in monitoring response to treatment.

Page 128: Hematology

CHAPTER 2: HEMATOLOGY

197.B. The French-American-British (FAB) classi-fication of acute leukemias, myeloproliferativedisorders, and myelodysplastic diseases wasoriginally based on cellular morphology andcytochemistry (immunophenotyping was lateradded). Using FAB criteria, acute leukemia wasdefined as greater than 30% bone marrow blasts.The diagnostic criteria used by the World HealthOrganization (WHO) includes morphologic,cytochemical, immunologic, cytogenetic, andmolecular features, as well as clinical findings,to better characterize all hematologic malignan-cies (myeloid and lymphoid) and predict diseasecourse. The WHO classification defines acuteleukemia as the presence of 20% or more bonemarrow blasts and includes diagnostic cate-gories with recurrent cytogenetic abnormalities.According to the WHO classification, lymphoiddisorders are grouped into B cell, T/NK cell, andHodgkin lymphoma. Further division of the Band T cell neoplasms considers site of involve-ment and precursor cell versus mature cell con-ditions.

are not seen in lymphoblasts, and their presencecan be diagnostic of acute myelogenous leukemia,such as acute myeloblastic leukemia (FAB typesMl and M2) or acute myelomonocytic leukemia(FAB type M4). Multiple Auer rods may be seenin acute promyelocytic leukemia (FAB type M3).Auer rods stain negatively with LAP, whichdetects the enzyme alkaline phosphatase in neu-trophil granules.

200.D. At presentation, patients with chronicleukemia (e.g., CLL or CML) consistently haveelevated leukocyte counts, whereas individualswith acute leukemia may present with low, nor-mal, or high leukocyte counts. The hallmarkfindings of anemia, thrombocytopenia, and neu-tropenia are often found in patients with acuteleukemia at the time of diagnosis and are due toreplacement of normal marrow hematopoieticcells by blasts. Patients with chronic leukemiamay have few symptoms at onset, with anemiaand thrombocytopenia developing during pro-gression of the disease.

198.A. The blood profile of both chronic myeloge-nous leukemia (CML) and a neutrophilic leuke-moid reaction is characterized by extremely highleukocyte counts with immature neutrophils.Splenomegaly is a manifestation of the malig-nant disease process and associated with CMLrather than a leukemoid reaction. The presenceof toxic granules and Dohle bodies would betypical of a leukemoid reaction caused by asevere bacterial infection. The LAP score is lowin CML and high in a neutrophilic leukemoidreaction.

199.C. In Color Plate 17«, the malignant blast cellcontains an Auer rod, composed of fused pri-mary granules, which stains positive with bothmyeloperoxidase and Sudan black B. Auer rods

201.D. Leukocyte alkaline phosphatase (LAP)scores are usually low in patients with chronicmyelogenous leukemia (CML). The LAPreflects alkaline phosphatase activity in neu-trophils, and the score is usually elevated in con-ditions where neutrophils are activated and/orincreased in number, such as late pregnancy,bacterial infection, and polycythemia vera. Theprimary use of the LAP is to distinguish betweenthe malignant cells of CML and a severe bacte-rial infection (leukemoid reaction). It may alsobe used to distinguish between CML and otherchronic myeloproliferative disorders such aspolycythemia vera. The LAP may be called NAP(neutrophil alkaline phosphatase) stain.

Page 129: Hematology

ANSWERS & RATIONALES • 357

202.C. Acute viral hepatitis is associated with lym-phocytosis. The major causes of neutrophilia arebacterial infection, neoplastic tumors, and inflam-matory responses to tissue injury. "Toxic" neu-trophils may be present (toxic granulation, Dohlebodies, vacuolization). Infection with organismsother than bacteria (fungi, some parasites, certainviruses) may also cause neutrophilia.

203.D. Monocytes must be distinguished from reac-tive lymphocytes, which are the characteristicfeature of infectious mononucleosis. Monocyto-sis occurring in the recovery stage of acute infec-tions is considered a favorable sign. An increasein monocytes is associated with collagen disor-ders (e.g., rheumatoid arthritis), tuberculosis, andmalignant conditions such as myelodysplasticsyndromes and monocytic leukemias.

204.B. The periodic acid-Schiff (PAS) stain can beused to detect intracellular glycogen deposits inthe lymphoblasts of acute lymphoblasticleukemia (ALL), in which coarse clumps of PASpositive material may be observed. Myeloblastsand monoblasts usually show a faint stainingreaction. The immunophenotype has a muchgreater diagnostic value for ALL than the cyto-chemical stain results. The PAS may also beused to distinguish the malignant erythroid pre-cursors of acute erythroleukemia, which showstrong PAS positivity, from normal erythrocyticcells that stain negative.

205.D. Myelodysplastic syndromes (MDSs) arecharacterized by a hypercellular bone marrowand up to 20% marrow blasts that distinguishMDS from acute leukemia (using WHO crite-ria). The blood and bone marrow blast percent-ages differ, and the risk of transformation to

acute leukemia varies with the types of MDSs.These disorders are characterized by one ormore peripheral blood cytopenias along withfeatures of abnormal growth (dyspoiesis) in thebone marrow. A consistent feature in all types ofmyelodysplasia is unexplained and refractoryanemia. Abnormalities may be morphologicand/or functional. Criteria that help define thetypes of myelodysplastic syndromes includemegaloblastoid maturation of erythroid precur-sors, presence of multinucleated red cells, ringedsideroblasts, hypogranular and/or hyposeg-mented neutrophils, monocytosis, abnormalplatelet morphology, circulating micromegakary-ocytes, and degree of dyspoiesis.

206.A. Naphthol AS-D chloroacetate esterase (spe-cific) reacts strongly in granulocytic cells, andalpha-naphthyl acetate esterase (nonspecific)stains positively in monocytic cells. The esterasestains are used to distinguish between subtypesof acute myelogenous leukemia. The cells ofacute myeloblastic leukemia (FAB types Ml andM2) will stain positive with specific esterase andnegative with nonspecific esterase. The cells ofacute monocytic leukemia (FAB type M5) willstain positive with nonspecific esterase and neg-ative with specific esterase. The cells of acutemyelomonocytic leukemia (FAB type M4) willshow positivity with both specific and nonspe-cific esterase. Stain results are correlated withcell morphology, immunophenotype, and kary-otype for diagnosis.

207.A. May-Hegglin anomaly is an autosomal dom-inant disorder in which large blue cytoplasmicstructures that resemble Dohle bodies are foundin the granulocytes and possibly the monocytes.Leukocytes are normal in function. Platelets aredecreased in number and abnormally large.About one-third of patients have mild to severebleeding problems because of abnormal plateletfunction.

Page 130: Hematology

358 CHAPTER 2: HEMATOLOGY

208. 211.D. Alder-Reilly anomaly is a hereditary auto-somal recessive disorder caused by a deficiencyof enzymes involved in the metabolism of muco-polysaccharides. Partially degraded mucopolysac-charides accumulate in various tissues, organs,and the leukocytes that are characterized by thepresence of large azurophilic granules resem-bling toxic granulation. The inclusions do notaffect leukocyte function and are referred toas Alder-Reilly bodies. The anomaly is oftenassociated with facial and skeletal abnormali-ties, such as those seen in Hunter syndromeand Hurler syndrome. Lysosomal fusion withimpaired degranulation is the defect in Chediak-Higashi syndrome and is associated with earlydeath due to abnormal leukocyte function.

209.C. Serum and urine protein electrophoresisdetects the presence of an M spike, the first essen-tial step in establishing the disorder as a mono-clonal gammopathy such as multiple myeloma orWaldenstrom disease. This can be followed byimmunoelectrophoresis to determine the class ofimmunoglobulin or chain type. Immunologicmarkers, cytochemical stains, and/or cytogeneticsare used in conjunction with cell morphology todiagnose malignant conditions.

210.C. The prognosis is poor for patients with stageIV Hodgkin disease, in which there is widespreaddisease including bone marrow involvement.Stage I or II Hodgkin disease has a very goodprognosis for cure. The clinical course and treat-ment varies with the extent of disease and mor-phologic subtype (Rye classification). The peakincidence for Hodgkin lymphoma occurs in youngadults (late twenties). Men have a 50% higherincidence of the disease than women. The CRPlevel and ESR are increased during active diseaseand can be used to monitor remission status.

C. The acute leukemia indicated by these resultsis acute myelomonocytic leukemia (AMML),which has both granulocytic and monocytic fea-tures. Note the monocytic characteristics of theblast cells in Color Plate 18«. CD 14 is a mono-cytic marker and CDS 3 is a marker for primitivemyeloid cells. The SBB shows positive stainingin both granulocytic and monocytic cells, thespecific esterase stains positive in granulocyticcells, and the nonspecific esterase is positive inmonocytic cells.

212.B. Acute lymphoblastic leukemia (ALL) ofchildren has the best prognosis. Other favorablefactors include children between ages 3 and 7,mild to moderate increases in the peripheralwhite blood count prior to treatment, and precur-sor B ALL, CALLA positive type (rather than Tcell ALL). Certain cytogenetic and molecularabnormalities are also associated with a betterprognosis. Acute leukemia in adults is lessfavorable because remissions are shorter andmore difficult to induce, especially in those over70 years of age. Prognosis is poor in adults withALL.

213.B. The test that would be the most beneficial forthe diagnosis of Hodgkin lymphoma is a lymphnode biopsy. Lymphadenopathy is the major clin-ical presentation of Hodgkin disease, and earlystages do not have bone marrow involvement. Askin biopsy would be indicated for diagnosis ofmycosis fungoides, a T cell lymphoma of theskin. A bone marrow exam and spinal tap areimportant to the diagnosis of acute leukemias.

Page 131: Hematology

ANSWERS & RATIONALES • 359

214. 217.

A. A hypercellular bone marrow and high M:Eratio are most characteristic of the excessive gran-ulocyte production that occurs in chronic myel-ogenous leukemia. Polycythemia vera typicallyhas a hypercellular marrow with panhyperplasiaand a normal or low M:E ratio. Beta-thalassemiamajor is a severe hemolytic anemia in which RBChyperplasia of the marrow is pronounced and alow M:E ratio is usual. Aplastic anemia is associ-ated with a hypocellular marrow with a reductionof all cell lines and normal M:E ratio.

C. Primary polycythemia (vera) is a malignantmyeloproliferative disorder characterized byautonomous marrow production of erythrocytesin the presence of low erythropoietin levels.Usual findings include increased RBC masswith elevated hemoglobin values and variabledegrees of leukocytosis and thrombocytosis(pancytosis). Splenomegaly, a high LAP score,thrombotic tendencies, and problems caused byblood viscosity are typical. Phlebotomy is doneto reduce red cell mass.

215. 218.B. Refractory anemia with ringed sideroblasts(RARS) is a myelodysplastic syndrome (MDS)that may also be referred to as primary or idio-pathic sideroblastic anemia. The main findingsthat characterize this type of MDS includerefractory anemia with a heterogeneous popula-tion of red cells, a hypercellular bone marrowwith <5% blasts, and the presence of >15%ringed sideroblasts in the marrow (demonstratedwith Paissian blue stain). RA and RARS are theleast likely MDS types to progress to acutemyelogenous leukemia.

216.C. Recent strenuous exercise or other physicaland emotional stimuli cause a transient increasein the leukocyte count. This is due to a redistribu-tion of the blood pools. Marrow injury to stemcells or marrow replacement by malignant cellscauses neutropenia of varying degrees. Neutrope-nia may be caused by immune mechanisms (anti-bodies) or an overactive spleen that sequestersneutrophils. Chemotherapeutic drugs also sup-press bone marrow production of neutrophils.

D. Basophilia (and eosinophilia) is a typicalfinding in patients with chronic myelogenousleukemia (CML). A progressive increase inbasophil number suggests transformation of thedisease to a more accelerated phase. Myelopro-liferative disorders such as CML, polycythemiavera, or AML are often associated with periph-eral basophilia, which is not a feature of lym-phoproliferative disorders such as acutelymphoblastic leukemia, hairy cell leukemia, orplasma cell leukemia.

219.

C. More than 50% of the marrow cells are ery-throid in acute erythroleukemia (FAB type M6).Giant erythroid precursors, bizarre and multinu-cleated red cells, and increased myeloblasts arefound in the marrow and may appear in theblood. Acute erythroid leukemia is rare, and thedisease typically evolves into acute myeloblasticleukemia (FAB types Ml or M2).

Page 132: Hematology

CHAPTER 2: HEMATOLOGY

220.D. Plasma cell myeloma is a clonal diseaseinvolving malignant end-stage B cells, in whichoverproduction of immunoglobulin is a hallmarkand the presence of red cell rouleaux is a charac-teristic rinding on the blood smear, as shown inColor Plate 20«. Excessive amounts of a mono-clonal immunoglobulin result in the depositionof proteins on circulating red cells that causesred cell "coining." The erythrocyte sedimenta-tion rate is extremely elevated because of spon-taneous rouleaux formation. Multiple myelomais characterized by bone pain and spontaneousbone fractures caused by tumors of plasma cells.Bone destruction leads to elevated calciumlevels, and renal impairment can result fromdamage by excess light chains. Plasma cells pro-gressively crowd out normal bone marrow pre-cursors and may be found in the bloodcirculation in advanced disease. Treatment withthalidomide has improved survival.

221.C. The production of hematopoietic cells insites outside of the bone marrow can be referredto as myeloid metaplasia or extramedullaryhematopoiesis. Hematopoiesis, with the excep-tion of lymphopoiesis, is normally confined tothe bone marrow during postnatal life. Produc-tion of erythroid, myeloid, and megakaryocyticelements can be established in the liver andspleen, similar to that which occurs duringembryonic development. Myeloid metaplasia isfrequently associated with myelofibrosis, a con-dition in which the marrow is gradually replacedby fibrotic tissue.

222.A. Prominent lymphadenopathy is the mostconsistent finding in non-Hodgkin types of lym-phoma at presentation, but lymphoma may alsoarise in the spleen, liver, or GI tract (abdominaltumor). Lymphomas begin as localized tumorsinvolving lymphoid tissue that spread to the

bone marrow and blood (depends on type). Themalignant lymphoid cells are immunologicallyclassified as B cell (most common) or T/NK cell.Clonality can also be established by demonstrat-ing gene rearrangements via molecular analysis.Some common subtypes of non-Hodgkin lym-phoma are small lymphocytic, Burkitt, follicu-lar, and mantle cell lymphomas. Leukemias areinitially systemic disorders primarily involvingthe bone marrow and blood at onset. Bonelesions are associated with multiple myeloma.

Methodology

223.

B. The standard assay for hemoglobin utilizespotassium ferricyanide. This solution, formerlycalled Drabkin's reagent, is now called cyan-methemoglobin (HiCN) reagent. The ferri-cyanide oxidizes hemoglobin iron from ferrous(Fe2+) to ferric (Fe3+), and the potassium cyanidestabilizes the pigment as cyanmethemoglobin forspectrophotometric measurement.

224.B. The band containing hemoglobin A2 is theslowest-migrating, staying closest to the cath-ode. The band containing hemoglobin A has anet negative charge at an alkaline pH, and itmoves the farthest toward the anode. An adultpatient without a hemoglobinopathy will haveonly these two bands appearing on a celluloseacetate electrophoresis.

225.

C. Hemoglobins S, D, and G all migrate to thesame location on the hemoglobin electrophoresisgel at an alkaline pH. However, because hemo-globins D and G are nonsickling hemoglobins,tests based on sickle formation under decreasedoxygen tension will have negative results. Thesehemoglobins can be further differentiated by theirmovement on agar gel at an acid pH, whereashemoglobins D and G will migrate with hemoglo-bin A, not with hemoglobin S.

Page 133: Hematology

ANSWERS & RATIONALES • 361

226. 230.C. When the sample is deoxygenated, reducedhemoglobin S polymerizes, resulting in a cloudysolution. A false negative can be obtained if thequantity of hemoglobin S is below the sensitivityof the method, which can be seen in newbornsand anemic patients. Although this procedure isa screening test for hemoglobin S detection, it ispositive in the presence of any sickling hemo-globin, such as hemoglobin

227.A. A slanted column increases the ESR. A clot-ted sample, which lacks fibrinogen, causes afalsely decreased ESR. Fibrinogen is the plasmaprotein that most greatly affects the ESR. TheEDTA tube for ESR must be at least half-full, andthe test must be set up within 4 hours of draw;failure to follow these guidelines results in poik-ilocytosis that will inhibit rouleaux formation.

228.C. Some patients develop EDTA-dependentplatelet agglutinins caused by an IgM or IgGplatelet-specific antibody. To correct for this, thesample can be redrawn in sodium citrate andrerun. The dilution factor of blood to anticoagu-lant in sodium citrate is 9:1. To compensate forthe 10% dilutional loss of platelets, the plateletcount obtained must be multiplied by 1.1 (300 X109/LX1.1=330X109/L).

229.B. Blood smears should be made within 5 hoursof collection from blood anticoagulated withEDTA. Although some of the blood cells maystill be normal in blood kept longer, others(especially granulocytes) may deteriorate. Vac-uolation of neutrophils can appear as an artifactin blood kept past this time. The age of the bloodmay also affect the visual quality when the slideis stained.

A. Decreasing the angle will produce a longer,thinner smear. Increasing the angle or using asmaller drop of blood will produce a shorter,thicker smear. The angle normally used for thespreader slide when making a smear is 30^5degrees.

231.D. One type of Romanowsky stain is theWright's stain. It is a polychrome stain consist-ing of methylene blue and eosin. This combina-tion causes multiple colors to appear on staining.Another commonly used Romanowsky stain isthe Wright's-Giemsa stain. Brilliant green andneutral red are used in a supravital stain forHeinz bodies. Crystal violet and safranin areused in Gram's stain for bacteria.

232.C. When red blood cells are stained correctlywith Wright's stain, their color is pink to orange-red. They will appear bright red in the presence ofan acid buffer and stain. Staining elements such aswhite cells, which stain with a more basic pH, willnot take up the stain adequately in this instance.Inadequate washing and an alkaline stain or buffermixture results in a smear that is excessively blue.

233.C. The formula for calculating a reticulocytecount in percent is

X 100Number of reticulocytes counted

Total number of RBCs counted

In the case described in question 233,

60%Reticulocytes = X 100 = 6.0%

1000

Because the error in reticulocyte counts is high,it is desirable to count a larger number of cells oruse a standardized counting method such as theMiller disk.

Page 134: Hematology

362 • CHAPTER 2: HEMATOLOGY

234. 238.C. The formula used to calculate the absolutereticulocyte count is

Reticulocyte percent

100X RBC (1012/L) X 1000

Multiplication by 1000 is done to report theresults in SI units of 109/L.

In this case,6.0 X 3.00

100X 1000 = 180 X 109/L

180 X 103/fJiL is not expressed in SI units.

235.B. As visualized in Color Plate 19«, Sudanblack B is a cytochemical stain for lipids, includ-ing steroids, phospholipids, and neutral fats. It iswidely used as a tool to differentiate the blasts ofacute lymphoblastic leukemia (ALL) from thoseof acute myelogenous leukemia (AML). Blastsin ALL are SBB negative, whereas those inAML will show some degree of positivity.

236.C. An LAP score is determined by first multi-plying the number of cells found by the degreeof positivity (i.e., 20 X 1 = 20). These numbersare then added together to obtain a final score. Inthis instance, 0 + 20 + 60 + 60 + 60 = 200.

237.C. When stained with a mixture of potassiumferricyanide and hydrochloric acid, nonhemeiron stains bright blue. This is the most commonstain used for storage iron. It can be used onbone marrow to identify sideroblasts, peripheralblood to identify the presence of siderocytes, orurine to perform hemosiderin testing.

C. Pappenheimer bodies are iron deposits associ-ated with mitochondria, and they stain with bothPerl's Prussian blue and Wright's stain. A cell thatcontains Pappenheimer bodies is called a sidero-cyte. Howell-Jolly bodies and basophilic stipplingcan be visualized with Wright's stain, whereasHeinz bodies require a supravital stain to be seen.

239.B. Depth on a standard counting chamber is0.10 mm. The formula to calculate volume is V=A X D, where V is volume, A is area, and D isdepth. When the counting chamber is used, thearea may change, depending on the number ofruled squares counted, but the depth remainsconstant.

240.B. The standard formula for hemacytometercounts expressed in mm3 is

Total number cells counted X dilution factor

Area counted X depth

In this instance,

308 X 20 _ 6160

8 mm2 X 0.10 mm 0.8 mm3= 7700/mm3

= 7.7 X 109/L

241.B. The Rule of Three states that RBC X 3 = Hgband Hgb X 3 = Hct ±3 in error-free results.These rules apply only for normocytic, nor-mochromic erythrocytes. One check to deter-mine if an error has occurred is to determine theMCHC. An MCHC should be less than 37.0g/dL in error-free results. The MCHC is calcu-lated by dividing hemoglobin by hematocrit andmultiplying by 100. In instance (B), the MCHCis 38.3 g/dL and the Rule of Three is broken. Allother answers follow the Rule of Three.

Page 135: Hematology

ANSWERS & RATIONALES • 363

242. 246.A. The only true cause of a high MCHC is thepresence of spherocytes, as may be seen inhereditary spherocytosis. Because the MCHC isa calculation using the hemoglobin and hemat-ocrit, anything causing those parameters to bewrong will affect the MCHC. The occurrence ofa falsely high MCHC is much more commonthan the presence of spherocytes, and specimentroubleshooting procedures must be undertakento obtain reportable results.

243.C. Blood cells are nonconductors of electricalcurrent; they create a resistance/impedance ofcurrent in a diluent solution that is conductive.When the suspension is forced through a smallaperture, the current flow is interrupted by thepresence of the cells. A pulse is generated. Thenumber of pulses generated is proportional tothe number of particles present, and the sizeof the pulse generated is proportional to the sizeof the cell.

244.C. Using ±2 s, 95% confidence limits areachieved; 95% confidence limits predict a rangethat values should fall within 95% of the time.For example, if a WBC count is 12.0 X 109/Lwith 2 s of ±0.5, then a succeeding count mustbe less than 11.5 X 109/L or greater than 12.5 X109/L to be considered significantly different.

245.B. Side angle scatter of a laser beam increaseswith granularity of the cytoplasm. Forwardangle scatter is used to determine relative size.The number of signals is proportional to thenumber of cells. The presence of specific anti-gens in the cytoplasm or on the cell surface isdetermined by immunofluorescence after reac-tions with appropriate antibodies.

A. An impedance counter cannot differentiatebetween the nucleus of a white blood cell andthe nucleus of an nRBC. Both will be counted asWBCs. The presence of 5 or more nRBCs/100WBCs can result in a falsely elevated whiteblood cell count, and a correction must be madeas follows:

Corrected WBC count =

Observed count X100

100 + # nRBCs/100 WBCs

247.D. Hemoglobin A2 values up to 3.5% are con-sidered normal. Values between 3.5 and 8.0%are indicative of beta-thalassemia minor. Hemo-globins C, E, and O have net electrical chargessimilar to hemoglobin A2. They elute off withhemoglobin A2 using anion exchange (column)chromatography, causing an invalid hemoglobinA2 result. If the hemoglobin A2 quantificationusing column chromatography yields a resultgreater than 8.0%, one of these interferinghemoglobins should be considered.

248.A. Any condition with spherocytes can causean increased osmotic fragility, dependent onthe number of spherocytes present. Spherocytesare seen in hereditary spherocytosis, immunehemolytic anemias, and severe burns. Targetcells, associated with thalassemias and hemoglo-binopathies, have an increased surface area-to-volume ratio and a decreased osmotic fragility.

249.B. The solubility test for hemoglobin S is notquantitative; it is reported as positive or nega-tive. A clotted specimen will not affect theresult. A clotted specimen will falsely decreasethe other tests listed: ESR due to low fibrinogen,hematocrit due to a false low RBC count, andplatelets are trapped in the clot.

Page 136: Hematology

CHAPTER 2: HEMATOLOGY

250.A. Hematology reference intervals are availablein many textbooks. They are influenced by patientpopulation, instrumentation, and reagents used.Therefore, it is ideal for each laboratory to estab-lish its own reference intervals. The referenceinterval excludes the upper and lower 2.5% of thevalues. The remaining 95% represent the refer-ence interval.

251.C. Standards are commercially available togenerate a hemoglobin concentration curve. Theabsorbance of each solution is read against areagent blank at 540 nm on a spectrophotometer.Patient blood samples and commercial controlmaterials can be used to assess precision andother quality control parameters.

252.A. Anything that causes an increase inabsorbance will cause a hemoglobin that is readspectrophotometrically to be falsely high. It isnecessary to correct for this type of error, such asmaking a plasma blank in the case of lipemia oricterus. WBCs are present in the hemoglobindilution and usually do not interfere. When theWBC count is extremely high, their presencewill cause cloudiness, increasing the absorbancein the hemoglobin measuring cell and resultingin a falsely high hemoglobin concentration.Excessive anticoagulant does not affect hemo-globin readings.

253.A. When a microhematocrit is spun at10,000-15,000 X g for 5 minutes, maximum pack-ing is achieved. Spinning a longer time has noaffect on the result. A tube that is not full causesRBC shrinkage and a falsely decreased hemat-ocrit. Hemolysis damages the RBC membrane

and allows for more packing. Trapped plasma ispresent when optimal packing is not achieved dueto inadequate speed or time of centrifugation,causing a falsely high hematocrit.

254.

C. The erythrocyte sedimentation rate (ESR)measures the rate of fall of red cells throughplasma. ESR increases when cells becomestacked (rouleaux, as seen in Color Plate 20B).ESR decreases when cells are not normal disco-cytes. Larger cells (macrocytes) and fewer cells(anemia) fall faster. Plasma containing increasedproteins, such as fibrinogen and globulins, pro-mote rouleaux formation and an elevated ESR.Hemoglobin content does not affect the ESR.

255.

A. Impedance counters measure RBCs andplatelets using the same dilution. To differentiatethe two, sizing thresholds are used. Particlesbetween 2 and 20 fL are counted as platelets,and particles larger than 35 fL are counted asRBCs. Small RBCs, clumped platelets, andgiant platelets fall in the overlap area betweenplatelets and RBCs, generating a warning flag.Nucleated RBCs are larger than normal RBCsand are not mistaken for platelets.

256.C. A platelet estimate is obtained by multiply-ing the average number of platelets per oilimmersion field (in an erythrocyte monolayer)by 20,000. The reference range for a plateletcount is 150^50 X 109/L. Approximately 8-20platelets per oil immersion field will represent anormal platelet concentration of approximately160^00 X 109/L. This method assumes the redblood cell count is normal. If it is not, alternateplatelet estimate procedures may need to beperformed.

Page 137: Hematology

ANSWERS & RATIONALES • 365

257. 261.D. A living cell stain using new methylene blueis performed for reticulocyte counts. Reticulocytesshould not be stained for less than 5 minutes.Howell-Jolly bodies, Pappenheimer bodies, cre-nated cells, and refractile artifact can be mistakenfor reticulocyte inclusions. Two or more particlesof reticulum constitute a reticulocyte.

258.B. All accredited laboratories are required toperform calibration with commercially availablecalibrators at least once every 6 months. Calibra-tion must be checked if any major part isreplaced or if optical alignment is adjusted. Acalibration procedure can be verified using com-mercially available controls.

259.A. An MCHC >37.0 g/dL is most likely causedby an error in measurement. In this instance, theRule of Three shows that the RBC X 9 matchesthe hematocrit, but the RBC X 3 does not matchthe hemoglobin. The hemoglobin does notmatch either the RBC or hematocrit. This indi-cates a hemoglobin problem, and it can be cor-rected with a saline replacement procedure. Thisspecimen may be lipemic or icteric. Warmingthe specimen is useful in troubleshooting a highMCHC due to a cold agglutinin. A microhemat-ocrit would be indicated if the hematocrit resultwas invalid.

260.A. Hemoglobin is valid on a hemolyzed speci-men, because RBC lysis is the first step in thecyanmethemoglobin method. The red blood cellcount depends on the presence of intact redblood cells. Red blood cell fragments caused byhemolysis may be as small as platelets and affectinstruments that use sizing criteria to differenti-ate the two. Therefore, samples for these proce-dures should be re-collected.

D. Heparin is recommended for osmoticfragility and red cell enzyme studies, because itresults in less lysis and less membrane stressthan other anticoagulants. Heparin inducesplatelet clumping and is unacceptable for theplatelet count. Heparin is unacceptable for coag-ulation test procedures because it binds withantithrombin to neutralize many enzymes, espe-cially thrombin. This would cause very longcoagulation test results. EDTA is recommendedfor most routine hematology procedures, espe-cially for Wright's-stained smears. Sodium citrateor EDTA can be used for sedimentation rates.

262.

A. When counting platelets, the center square(1 mm2) is counted on each side of the hemacy-tometer. Platelets appear round or oval and mayhave dendrites. These characteristics can helpdistinguish them from debris, which is irregu-larly shaped and often refractile. White cells arenot lysed; they may be counted, using a differentruled area of the hemacytometer. Platelets willbe easier to count if allowed to settle for 10 min-utes, because they will have settled into oneplane of focus.

263.

B. "Precision" is the term used to describe thereproducibility of a method that gives closelysimilar results when one sample is run multipletimes. An accurate method is one that givesresults that are very close to the true value. Lab-oratories must have procedures that are bothaccurate and precise.

Page 138: Hematology

366 • CHAPTER 2: HEMATOLOGY

Case Histories

264.C. WBC counts done by an impedance cellcounter must be corrected when nucleated redblood cells (nRBCs) are present (see Color Plate4B), because such instruments do not distin-guish between white and red nucleated cells.This correction is done according to the follow-ing formula:

Corrected WBC count =

Observed count X100

100 + # nRBCs per 100 WBCs

In this instance,

35.0 X100

100 + 110= 16.7 X 109/L

265.A. The appearance of red cells on a differentialsmear may be predicted by calculating the redcell indices.

MCV =Hct X 10 16% X 10

127RBC 2.50 X 10U/L= 64.0 fL

MCHHgb X 10 4.5 g/dL X 10

RBC 2.50 X 10"/L= 18.0 pg

MCHC =Hgb X 100 4.5 g/dL X 100

Hct 16%- 28.1 g/dL (281 g/L)

The mean corpuscular volume (MCV), mean cor-puscular hemoglobin (MCH), and mean corpuscu-lar hemoglobin concentration (MCHC) are allbelow the reference range. This indicates a cellthat is small (microcytic) with a reduced hemoglo-bin concentration (hypochromic). These indicesrefer to averages and do not necessarily reflect theactual appearance of cells in which there is greatdiversity in size and shape.

266.D. Children with beta-thalassemia major, alsoknown as Cooley anemia, do not use iron effec-tively to make heme. This occurs because of agenetic defect that causes a decreased rate ofproduction of structurally normal globin chains.In addition, these children receive frequenttransfusions due to the severe hemolytic anemia.The result is hemochromatosis with a highserum iron and storage iron. None of the otheranemias listed would elicit the bone marrowresponse seen by the high number of nucleatedRBCs, because they are not hemolytic.

267.D. Beta-thalassemia major is characterized by aninability to produce beta-globin chains, resultingin a decrease or complete absence of hemoglobinA. Hemoglobin F, a compensatory hemoglobinthat contains two alpha- and two gamma-globinchains, is frequently the only hemoglobin present.Hemoglobin A2 is classically increased in het-erozygous beta-thalassemia, but it is variable inhomozygous beta-thalassemia.

268.B. The predominant hemoglobin present atbirth is hemoglobin F, which consists of twoalpha- and two gamma-globin chains. It is notuntil about 6 months of age that beta-chain pro-duction is at its peak. At this point, hemoglobinA (two alpha and two beta chains) replaceshemoglobin F as the predominant hemoglobin.A deficiency in the production of these chainswill not be apparent until this beta-gammaswitch has occurred.

Page 139: Hematology

ANSWERS & RATIONALES • 367

269.D. The formula for calculation of transferrinsaturation is as follows:

Transferrin saturation % =Serum iron Og/dL) X 100

TIBC ((ig/dL)

In this case,

22 X 100Transferrin saturation % = = 15%

Because the reference range for saturation is20-45%, this is a low saturation.

270.D. In the anemia of chronic disease, patientshave iron but are unable to utilize it. Hepcidin, ahormone produced by the liver, plays a role inbody iron regulation. Intestinal iron absorptionand release of iron from macrophages bothdecrease in response to increased hepcidinlevels. Hepcidin is a positive acute-phase reac-tant, so increased levels are seen in anemia ofchronic disease due to inflammation. Thisadversely affects iron availability.

271.A. The most common anemia among hospital-ized patients is anemia of chronic disease.Patients with chronic infections, inflammatorydisorders, and neoplastic disorders develop thistype of anemia. The typical presentation is a nor-mocytic, normochromic anemia, but microcyticand hypochromic anemia can develop in long-standing cases. Chronic blood loss can causeiron deficiency and microcytic/hypochromicanemia.

272.A. Both megaloblastic anemias and some non-megaloblastic anemias are characterized by the

presence of macrocytic, normochromic redcells. Vitamin B12 and folic acid are coenzymesnecessary for DNA synthesis. Lack of either onecauses megaloblastic anemia. Maturation asyn-chrony is evident in both the peripheral bloodand bone marrow. The bone marrow examina-tion is done after vitamin B]2 and folate levelsbecause of the test's invasive nature. Vitamin B12

and folic acid levels are normal or increased innonmegaloblastic anemias. Iron studies are use-ful in the diagnosis of microcytic/hypochromicanemias.

273.

C. The common causes of megaloblastic anemiaare pernicious anemia and folic acid deficiency.Neurological symptoms are not associated withfolic acid deficiency. Folic acid is a water-solublevitamin for which there are low body stores. Adiet low in green vegetables and meat products orhigh in alcohol can result in folate deficiency in

months. Alcohol is a folate antagonist.

274.A. The general classification of anemiadescribed here is megaloblastic anemia. A defi-ciency of vitamin B12 or folic acid affects DNAproduction. All dividing cells will show nuclearabnormalities, resulting in megaloblastic changes.In the neutrophil, as seen in Color Plate 1 !•, thistakes the form of hypersegmentation (five lobesor more). Enlarged, fragile cells are formed,many of which die in the bone marrow. Thisdestruction leads to increased LD, bilirubin, andiron levels. Oval macrocytes and teardrop cellsare seen. Pancytopenia and inclusions are com-mon findings. One cause of a nonmegaloblasticmacrocytic anemia, which has round cells suchas target cells instead of oval cells, is liverdisease.

Page 140: Hematology

CHAPTER 2: HEMATOLOGY

275. 278.B. The pancytopenia and red blood cell mor-phologic findings are all consistent with mega-loblastic anemia. Further investigation of serumfolate and vitamin B^ levels is warranted. Perni-cious anemia (PA) is noted for neurologicalcomplications and is seen more commonlyamong people of British and Scandinavianancestry. PA is caused by a lack of intrinsic fac-tor production in the stomach, which is neces-sary for the absorption of vitamin B12. Becausethere are large body stores of vitamin 612, ittakes from 1 to 4 years for the deficiency to man-ifest itself. Aplastic anemia is also associatedwith pancytopenia, but not the red cell morpho-logic changes seen in this patient.

276.A. Alcohol is a folic acid, not a vitamin Bj2,antagonist. Patients with pernicious anemia (PA)are incapable of absorbing vitamin Bj2 due to alack of intrinsic factor or antibodies to intrinsicfactor or parietal cells. PA is characterized byachlorhydria and atrophy of gastric parietal cellsthat secrete intrinsic factor. Achlorhydria is notdiagnostic for PA, because it may occur in otherdisorders (such as severe iron deficiency), but itis confirmatory evidence of the problem. D.latum competes for B^ in the intestines.

277.B. Because intrinsic factor is necessary forabsoiption of vitamin B^ from the ileum, intra-muscular injections of vitamin B^ are used totreat PA. Although oral doses of folic acid willcorrect the megaloblastic blood profile seen inPA, the neurological symptoms will notimprove. For this reason, correct diagnosis iscrucial. Methotrexate is a folic acid antagonist.Because body folic acid stores are low, a defi-ciency can develop quickly.

A. The adult red blood cell in glucose-6-phos-phate dehydrogenase (G6PD) deficiency is sus-ceptible to destruction by oxidizing drugs. Thisoccurs because the mechanism for providingreduced glutathione, which keeps hemoglobin inthe reduced state, is defective. The anti-malarialdrug primaquine is one of the best-known drugsthat may precipitate a hemolytic episode. Inges-tion of fava beans can also elicit a hemolyticepisode in some patients.

279.C. G6PD deficiency, a sex-linked disorder, is themost common enzyme deficiency in the hexosemonophosphate shunt. Most patients are asymp-tomatic and go through life being unaware of thedeficiency unless oxidatively challenged. Pyru-vate kinase, an enzyme in the Embden-Meyerhofpathway, is necessary to generate ATP. ATP isneeded for red blood cell membrane mainte-nance. Patients with a pyruvate kinase deficiencyhave a chronic mild to moderate anemia.

280.B. Reduced glutathione levels are not main-tained due to a decrease in NADPH production.Methemoglobin (Fe3+) accumulates and dena-tures in the form of Heinz bodies. Heinz bodiescause rigidity of the RBC membrane, resultingin red cell lysis. Dohle bodies are composed ofRNA; Howell-Jolly bodies are composed ofDNA; Pappenheimer bodies are iron deposits.

Page 141: Hematology

ANSWERS* RATIONALES

281.C. Iron-deficiency anemia (IDA) causes amicrocytic, hypochromic anemia. It is the mostcommon anemia found in children. IDA devel-ops quickly in children because of rapid growthwith increased dietary iron requirements. Hered-itary spherocytosis results in RBCs that are nor-mal to low-normal in size, with an MCHCpossibly greater than 37.0 g/dL. Folic acid defi-ciency causes a macrocytic/normochromic ane-mia. Erythroblastosis fetalis is a hemolyticdisease of the newborn caused by red blood celldestruction by antibodies from the mother; suchantibodies are no longer in the circulation of a15-month-old child.

282.C. The development of iron deficiency occurs instages: the iron depletion stage, the iron-deficienterythropoiesis stage, and the iron-deficiency ane-mia stage. Iron stores are the first to disappear, sothe serum ferritin level is the earliest indicator ofiron-deficiency anemia. This is followed bydecreased serum iron and increased TIBC. Thelast abnormality seen is microcytic, hypochromicred blood cells.

283.C. In iron-deficiency anemia, red blood cellproduction is restricted because of lack of iron,and the reticulocyte absolute value reflects thisineffective erythropoiesis. The formula used tocalculate the absolute reticulocyte count is

Absolute reticulocytes =Reticulocytes %

100X RBC (10U/L) X 1000

The 1000 in the calculation is to convert to SIunits (109/L).In this case,

0.2

100

The reference interval for the absolute reticulo-cyte count is approximately 18-158 X 109/L.

284.A. Petechiae and ecchymoses (bruises) are pri-mary hemostasis bleeding symptoms seen inquantitative and qualitative platelet disorders.Although estimates vary, spontaneous bleedingdoes not usually occur until platelet numbers areless than 50 X 109/L. The malignant disorderrepresented in this case is noted for thrombocy-topenia.

285.A. The bone marrow blast percent indicates thepresence of an acute leukemia. The triad of symp-toms seen in acute leukemia is neutropenia, ane-mia, and thrombocytopenia. Acute lymphoblasticleukemia is the leukemia most likely to be foundin this age group. Hairy cell leukemia does notpresent with blasts. Myelodysplastic syndromepresents with less than 20/30 (WHO/FAB,respectively) percent marrow blasts.

286.C. Periodic acid-Schiff stains glycogen in lym-phoblasts. The myeloperoxidase stain is positivein myeloid cells. Monocytes show a positivereaction to the nonspecific esterase stain. Leuko-cyte alkaline phosphatase is useful in the diag-nosis of chronic myelogenous leukemia.

287.A. Terminal deoxyribonucleotidyl transferase(TdT) is a nuclear enzyme (DNA polymerase)found in stem cells and precursor B and T lym-phoid cells. High levels of TdT are found in 90%of ALLs. TdT has been found in up to 10% ofcases of AML (FAB MO and Ml), but in lowerlevels than are present in ALL. This enzyme isnot found in mature lymphocytes.

I

X 2.70 (10U/L) X 1000 - 5 X 10y/L

Page 142: Hematology

370 m CHAPTER 2: HEMATOLOGY

288. 292.B. There are now more than 200 recognizedhuman leukocyte antigens, each of which hasbeen given a CD (cluster designation) number.CDs 2, 5, and 7 are seen on T cells. CALLA, thecommon acute lymphoblastic leukemia antigen,is seen in early pre-B cells. Distinct CD markershave been identified for cells of both lymphoidand myeloid stem cell lineage.

289.B. The bone marrow blast percent is highenough to indicate an acute leukemia. Sudanblack B, myeloperoxidase, and specific esterasestains are positive, indicating the presence of themyeloid cell line. The nonspecific esterase stainis negative, indicating the absence of a inono-cytic cell line. The bone marrow blast percent istoo low for FAB Ml, but it is in the range forFAB M2.

290.A. Chromosome analysis is an important diag-nostic tool in clinical medicine. Nonrandomchromosome abnormalities are recognized inmany forms of cancer. t(8;21) is associated withacute myelogenous leukemia FAB M2; t(15;17)is only seen in FAB M3. The Philadelphia chro-mosome, t(9;22), is seen in at least 90% ofpatients with chronic myelogenous leukemia;t(8;14) is associated with Burkitt lymphoma.

291.B. When a diagnosis of AML or myelodysplasticsyndrome is suspected, a bone marrow examina-tion is performed. The WHO approach to thediagnosis of acute leukemia requires the presenceof >20% blasts in the bone marrow; the FABclassification requires >30%. The referenceinterval for bone marrow blast percent is 0-2%.Myelodysplastic syndromes have increased bonemarrow blast percentages, but <20% using WHOcriteria and <30% using FAB criteria.

B. HTLV-I is implicated in T cell leukemia andlymphoma in Japan. The Epstein-Barr virus isassociated with Burkitt lymphoma in Africa.Chronic bone marrow dysfunction can be causedby exposure to radiation, drugs, and chemicalssuch as benzene. Myelodysplastic syndromes andmyeloproliferative disorders are "preleukemic"because they have a high incidence of terminatingin acute leukemia. Paroxysmal nocturnal hemo-globinuria, aplastic anemia, multiple myeloma,and lymphoma are stem cell disorders that areparticularly noted for transformation into acuteleukemia. Genetic susceptibility is associatedwith Klinefelter and Down syndromes, both ofwhich have chromosomal abnormalities. It islikely that more than one factor is responsible forthe evolution of an acute leukemia.

293.A. The myelodysplastic syndromes (MDSs) arepluripotential stem cell disorders characterizedby one or more peripheral blood cytopenias.Bone marrow examination is necessary for diag-nosis. There are prominent maturation abnor-malities in all three cell lines in the bonemarrow. Megaloblastoid erythrocyte maturationis present that is not responsive to Bi2 or folicacid therapies. Although many of the red bloodcell inclusions noted in this case can be seen in amegaloblastic anemia such as pernicious ane-mia, this patient has a normal vitamin B12 andfolate level. This patient has hyposegmentationof neutrophils, whereas megaloblastic anemiaspresent with hypersegmentation of neutrophils.Of the disorders listed, the only one associatedwith dyshematopoiesis of all cell lines ismyelodysplastic syndrome.

Page 143: Hematology

ANSWERS & RATIONALES • 371

294. 297. 1D. In most cases of MDS, the bone marrow ishypercellular with erythroid hyperplasia. MDSis considered a disease of the elderly. Becausenormal cellularity decreases with age, interpre-tation of cellularity must take the age of thepatient into account. WHO criteria for a diagno-sis of MDS are related to bone marrow blast per-cent, which must be <20%.

295.D. In RAEB at least two cell lines exhibitcytopenia, and all cell lines show evidence ofdyshematopoiesis. Poor granulation and pseudo-Pelger-Huet anomaly is seen. There are less than5% blasts in the peripheral blood, and between5 and 19% blasts in the bone marrow. Plateletsexhibit poor granulation, giant forms, and theabnormal maturation stage of micromegakary-ocytes. The five FAB classifications ofmyelodysplastic syndrome are RA, RARS,RAEB, CMML, and RAEB-t. CML is a myelo-proliferative disorder, not a myelodysplasticsyndrome.

296.C. The myelodysplastic syndromes are refrac-tory to treatment, and patients are supportedusing blood products dependant on their cytope-nias. The median survival rate for all types ofMDSs is less than 2 years. RAEB and RAEB-thave the highest percentage of blasts and thelowest survival rates. At this time, bone marrowtransplant offers the only chance for a cure, andit is the treatment of choice in patients less than50 years old. Studies have shown that the inci-dence of MDS is greater than the incidence ofAML in the 50-70-year-old age group. Up to40% of the myelodysplastic syndromes trans-form into acute leukemia.

B. Chronic myelogenous leukemia (CML) is amyeloproliferative disorder, a malignant prolif-eration of leukocytes not in response to infec-tion; the leukocyte count is often greater than100.0 X 109/L. No toxic changes are present inCML. Thrombocytosis is seen in more than halfof the patients with CML. A neutrophilic leuke-moid reaction represents a normal bodyresponse to a severe infection. It is a benign pro-liferation of WBCs with a high leukocyte countbut usually less than 50.0 X 109/L. Toxicchanges to the neutrophils such as toxic granula-tion, vacuoles, and Dohle bodies are present.These two disorders both display a "left" shift,and they can be confused with each other.

298.D. Leukocyte alkaline phosphatase activity isincreased in severe infections such as the neu-trophilic leukemoid reaction and polycythemiavera, and during the last trimester of pregnancy.It is greatly reduced in chronic myelogenousleukemia, although it may increase during blastcrisis of this disease. Periodic acid-Schiff andSudan black B are used to differentiate ALLfrom AML. The TRAP stain is useful in thediagnosis of hairy cell leukemia.

299.C. The Philadelphia chromosome, t(9;22), isfound in the precursor cells for erythrocytes,granulocytes, and platelets in at least 90% of thecases of CML. It is an acquired chromosomeabnormality that results from a reciprocaltranslocation between chromosomes 9 and 22,and it can be detected even when the patient is inremission. The BCR/ABL oncogene is alsoassociated with CML.

Page 144: Hematology

372 « CHAPTER!: HEMATOLOGY

300.B. Patients who have the Philadelphia chromo-some have a less aggressive disease and betterprognosis than the rare cases that do not have the

abnormality. Some cases of CML terminate inan acute lymphoblastic leukemia. However, thisoutcome cannot be predicted by the presence orabsence of the Philadelphia chromosome.

REFERENCES

Anderson, S. C., and Poulsen, K. B. (2003). Atlas of Hematology, Philadelphia:Lippincott Williams & Wilkins.

Can, J. H., and Kodak, B. F. (2008). Clinical Hematology Atlas, 3rd ed. St. Louis:Elsevier.

Harmening, D. M. (Ed.) (2002). Clinical Hematology and Fundamentals ofHemostasis, 4th ed. Philadelphia: F. A. Davis.

Hoffbrand, V., Moss, P., and Pettit, J. (2006). Essential Haematology, 5th ed. Maiden,MA: Wiley-Blackwell.

McKenzie, S. B. (2004). Clinical Laboratory Hematology, Upper Saddle River, NJ:Pearson Prentice Hall.

Rodak, B. F., Fritsma, G. A., and Doig, K. (2007). Hematology Clinical Principles andApplications, 3rd ed. St. Louis: Elsevier.